Download as pdf or txt
Download as pdf or txt
You are on page 1of 124

Otorhino

laryngology
Diseases of Ear, Nose and
Throat
&
Head and Neck surgery

Previous years
question-answer

©shovandas 1
©shovandas 2
Contents

Part Page no.


First part : Ear 5
Second part : Nose 41
Third part : Throat 65

©shovandas 3
©shovandas 4
First part

Ear

©shovandas 5
©shovandas 6
Group – A
01. Describe the aetiology, clinical features and managements of Acute Suppurative Otitis Media. [2018](3+4+3)
➢ Acute suppurative otitis media (ASOM): Inflammation of middle ear cleft of rapid onset & infective origin associated with
local and systemic signs.
➢ Aetiology:
o More common especially in infants and children of lower socioeconomic group.
o The disease follows viral infections of upper respiratory tract but soon pyogenic organisms invade the middle ear.
o Viruses:
▪ RSV, Influenza A, Parainfluenza, Rhinovirus, Adenovirus.
▪ Associated with 60-90% of acute otitis media (AOM).
o Bacteria:
▪ S. pneumoniae, ▪ Moraxella catarrhalis.
▪ H. influenzae,
➢ Clinical features:
o Stage of tubal occlusion:
▪ Oedema and hyperaemia of nasopharyngeal end of eustachian tube.
▪ Blockage of tube + absorption of air = negative intratympanic pressure.
▪ Symptoms:
✓ Mild earache.
✓ Generally no fever.
✓ Deafness is not marked.
▪ Signs:
✓ Retracted tympanic membrane.
✓ Loss of light reflex.
✓ Tuning fork test: conductive deafness.
o Stage of presuppuration:
▪ Pyogenic invasion.
▪ Exudate/ effusion in middle ear cleft.
▪ Serum, fibrin, PMNL, RBC are present.
▪ Symptoms:
✓ Marked earache.
✓ High grade fever in children.
✓ Deafness and tinnitus in adults.
▪ Signs:
✓ Congestion of pars tensa.
✓ Leash of blood vessels appear along the handle of malleus and at the periphery of TM, imparting it a
cartwheel appearance.
✓ Later, whole of TM including pars flaccida becomes uniformly red.
✓ Tuning fork test: conductive deafness.
o Stage of suppuration:
▪ Pus in middle ear and mastoid air cells.
▪ Symptoms:
✓ Excruciating earache.
✓ Fever of 102°-103°F.
✓ Deafness increases.
✓ Vomiting and convulsion may be present.
▪ Signs:
✓ Red, bulging TM with loss of landmark.
✓ TM may perforate.
✓ Yellow spot may be seen on the tympanic membrane where rupture is imminent.
✓ X-rays of mastoid: clouding of air cells because of exudate.
o Stage of resolution:
▪ Release of pus thorough ruptured TM & subsidence of symptoms.
▪ If proper treatment is started early or if the infection was mild, resolution may start even without rupture of
tympanic membrane.
▪ Symptoms:
✓ With evacuation of pus, earache is relieved.
✓ Fever comes down.
▪ Signs:
✓ External auditory canal may contain blood tinged discharge, later becomes mucopurulent.
©shovandas 7
✓ Small perforation in anteroinferior quadrant of pars tensa.
✓ TM returns to normal colour and landmarks.
o Stage of complications:
▪ Causes of complications:
✓ Highly virulent organism.
✓ Poor resistance by the patient.
✓ Resolution may not take place & disease spreads beyond middle ear.
▪ Complications:
✓ Acute mastoiditis, subperiosteal abscess, facial paralysis, labyrinthitis, petrositis, abscess in brain,
extradural abscess, meningitis or the lateral sinus thrombophlebitis.
➢ Management:
o In 66% otalgia subsides in 24 hrs.
o Symptomatic relief in 88% by 4-7 days.
o Medical management:
▪ NSAIDs help to relieve pain and bring down temperature.
▪ Antimicrobial:
✓ If < 2 years, antimicrobial in first visit.
✓ If > 2 years, antimicrobial only if infection is persisting.
✓ Antibiotics: Amoxycillin, Amoxy-Clav., Erythromycin,
Cephalosporins.
✓ Topical antibiotic or 1% acetic acid ear drops.
o No role of antihistaminics and decongestants.
o Ear toileting can be done.
o Surgical management:
▪ Myringotomy: limited role.
▪ Cortical mastoidectomy: only in case of complications.
o Vaccination:
▪ Influenza A vaccination.
▪ Bacterial vaccination ineffective.
o All cases of ASOM should be carefully followed , till drum membrane returns to its normal appearance & conductive
deafness disappears.

02. Describe anatomy of tympanic membrane. Write different type of eardrum perforations with their clinical significance.
[2004, 2006, 2008](6+4)
➢ Tympanic membrane or the eardrum:
o It forms the partition between the external acoustic canal & the middle ear.
o Obliquely set.
o 9–10 mm tall, 8–9 mm wide and 0.1 mm thick.
o Two parts:
▪ Pars tensa:
✓ It forms most of tympanic membrane.
✓ Annulus tympanicus: Its periphery is thickened to form a fibrocartilaginous ring, called as Annulus
Tympanicus, which fits in tympanic sulcus.
✓ Umbo: The central part of pars tensa is tented inwards at level of tip of malleus and is called umbo.
✓ Cone of light: Can be seen radiating from the tip of malleus to the periphery in the antero-inferior
quadrant.
▪ Pars flaccida:
✓ Also called shrapnell’s membrane.
✓ This is situated above lateral process of
malleus between the notch of Rivinus and
anterior posterior malleal folds.
✓ It is not so taut and may appear slightly
pinkish.
➢ Lndmarks on lateral surface of TM:
o Shadow of incudostapedial joint,
o Shadow of round window,
o Anterior malleal fold,
o Posterior malleal fold,
o Lateral process of malleus.

©shovandas 8
➢ Layers of tympanic membrane:
o Three layers:
▪ Outer epithelial layer: Continuous with skin lining meatus.
▪ Inner mucosal layer: Continuous with mucosa of middle ear.
▪ Middle fibrous layer:
✓ Encloses the handle of malleus.
✓ 3 types of fibre: radial, circular and parabolic.
➢ Nerve supply:
o Anterior half of lateral surface: Auriculotemporal (V3).
o Posterior half of lateral surface: Auricular branch of vagus (CN X).
o Medial surface: Tympanic branch of CN IX (Jacobson’s nerve).
➢ Blood supply:
o Arterial supply: Anterior tympanic branch of maxillary artery.
o Venous drainage: Veins drain into pterygoid venous plexus and superio-petrosal sinus.
➢ Eardrum perforations:
o Central perforation: Perforation in central part of pars tensa.
▪ Types:
✓ Anterior: Anterior to handle of malleus.
✓ Posterior: Posterior to handle of malleus.
✓ Inferior: Inferior to handle of malleus.
✓ Subtotal: Very large perforation of pars tensa where part of pars tensa and/or annulus of TM are still
preserved.
▪ Clinical significance:
✓ Central perforations are considered safe, as cholesteatoma is usually not associated with them.
✓ Seen in tubotympanic type CSOM.
o Marginal perforation: Perforation in marginal part of pars tensa and destroys even the annulus and reaches sulcus
tympanicus.
▪ Types:
✓ Postero-superior (most common),
✓ Anterior,
✓ Inferior,
✓ Total.
▪ Clinical significance:
✓ Stratified squamous epithelium from the external auditory canal can grow into the middle ear in any
type of marginal perforation by immigration & form a cholesteatoma. Therefore all marginal
perforations are considered dangerous.
✓ Seen in atticoantral type CSOM.
o Attic perforation: Perforation in pars flaccida.
▪ Clinical significance:
✓ Often associated with cholesteatoma.
✓ Seen in atticoantral type CSOM.

03. What is middle ear cleft? Describe anatomy of medial wall of middle ear. [2013](3+7)
➢ Middle ear cleft:
o The middle ear cavity together with eustachian tube, aditus, antrum & mastoid air cells is called middle ear cleft.
o Mucous membrane of nasopharynx is continuous with of middle ear
cleft mucous.
o Filled with air.
o Structures:
▪ Middle ear cavity:
✓ The middle ear extends much beyond the limits of
tympanic membrane and is divided into 3 parts.
✓ Mesotympanum: Lying opposite the pars tensa.
✓ Epitympanum or attic: Lying above pars tensa but,
medial to pars flaccida and bony lateral attic wall.
✓ Hypotympanum: Lying below the level of pars tensa.
▪ Eustachian tube: Also called pharyngotympanic tube. The por-
tion around the tympanic orifice of the eustachian tube near
the hypotympanum is called the protympanum.
▪ Aditus ad antrum: Opening through which attic communicates to antrum.
▪ Mastoid antrum: Large air containing space in the upper part of mastoid. Its roof is formed by the tegmen
antri, which is a continuation of tegmen tympani and separates it from the middle cranial fossa.
©shovandas 9
▪Mastoid air cells: Mastoid consists of bone cortex with honeycomb of air cell underneath. Antrum is basically
the largest air cell present in the mastoid.
➢ Anatomy of medial wall:
o Common wall between middle ear and inner ear.
o Promontory:
▪ Bulge produced by basal turn of cochlea.
▪ Senses sound of high frequency.
o Oval window:
▪ Covered by foot plate of steps.
▪ Connects to scala vestibuli.
o Round window:
▪ Covered by secondary TM.
▪ Connects to scala tympani.
o Lateral semi circular canal: Forms a bulge in medial wall.
o Processus cochleariformis:
▪ Projection of bone from cochlea.
▪ Just anterior to oval window.
▪ The tendons of tensor tympani
muscle take a turn here , move
laterally to attach to the handle of malleus.
▪ Acts as landmark for 1st genu of facial nerve, which is important for surgery.
o Facial nerve:
▪ 1st genu: enters medial wall.
▪ Horizontal or tympanic segment:
✓ Above oval window.
✓ Below lateral semi circular canal.
✓ Present in the canal for facial nerve.
✓ Its bony coverings may sometimes be congenitally dehiscent and thenerve may lie exposed, making
it vulnerable to injuries or infection.

04. Describe the anatomy of middle ear cavity. [2009](10)


➢ Middle ear cavity:
o The middle ear extends much beyond the limit of tympanic
membrane and is divided into 3 parts.
▪ Mesotympanum: Lying in opposite to pars tensa.
▪ Epitympanum or attic: Lying above pars tensa , but medial to pars flaccida and the bony lateral attic wall.
▪ Hypotympanum: Lying below the level of pars tensa.
o Protympanum: The portion around tympanic orifice of eustachian tube near the hypotympanum.
o Middle ear can be likened to a six sided box with a roof and a floor; and medial, lateral, anterior and posterior walls.
➢ Roof:
o Formed by a thin plate of bone called tegmen tympani.
o It also extends posteriorly to form the roof of the aditus and antrum (tegmen antri).
o Separates tympanic cavity from the middle cranial fossa of temporal lobe.
➢ Floor:
o a thin plate of bone, which separates tympanic cavity from the jugular bulb.
o 9th, 10th and 11th cranial nerves are present below the floor.
o Jacobson's nerve enters middle ear through floor and forms the tympanic plexus in promontory.
➢ Medial wall: Question no. 03
➢ Lateral wall:
o Common wall between external ear and middle ear.
o Most of the part is formed by tympanic membrane.
o Scutum: The bone above paras flaccida forming the lateral wall of attic.
o Prussak's space: Area medial to pars flaccida.
➢ Anterior wall:
o Thin plate of bone, which separates the cavity from internal carotid artery.
o Three openings:
▪ The lower one for the eustachian tube.
▪ The middle one for chorda tympani (the canal of huguier).
▪ The upper one for the canal of tensor tympani muscle.

©shovandas 10
©shovandas 11
➢ Posterior wall:
o Separates middle ear from the mastoid.
o Pyramid:
▪ Bony projection.
▪ Stapedius muscle originates from the summit of the pyramid.
▪ Inserted into the neck of stapes.
o Aditus:
▪ Opening, through which attic communicates with the antrum.
▪ Lies above the pyramid.
o Nerves:
▪ Facial nerve: Horizontal segment from medial wall → takes a turn at junction of medial and posterior wall
→ runs vertically as vertical /mastoid segment of facial nerve.
▪ Chorda tympani: Opens at posterior wall → passes through middle ear → exits through posterior wall.
o Facial recess:
▪ Depression in the posterior wall.
▪ Also called:
✓ Supra pyramidal recess,
✓ Posterior sinus.
▪ Boundaries:
✓ Medially: Vertical segment of facial nerve.
✓ Laterally: Entry for chorda tympani.
✓ Superiorly: Fossa incudis , depression produced by the short process of incus.
o Sinus tympani:
▪ Also called infra pyramidal recess.
▪ Hidden area of middle ear.
▪ Deep recess medial to pyramid.
▪ Bounded by the subiculum below and the ponticulus above.

05. Describe anatomy of medial and posterior wall of middle ear. Describe relation of the facial nerve in the above walls.
[2011](4+4+2)
➢ Anatomy of medial wall: Question no. 03.
➢ Anatomy of posterior wall: Question no. 04.
➢ Facial nerve relations:
o Last two segments of intra-temporal part of the facial nerve , are present in middle ear & are in relation with the
structures of medial and posterior wall.
o Tympanic or horizontal segment:
▪ 11.0 mm.
▪ In relation with the structures of medial wall.
▪ From geniculate ganglion to just above the pyramidal eminence.
▪ Above the oval window.
▪ Below the lateral semicircular canal.
▪ Present in the canal for facial nerve.
▪ Its bony covering may sometimes be congenitally dehiscent & the nerve may lie exposed, making it vulnerable
to injuries or infection.
o Takes a turn at junction of medial and posterior wall.
o Mastoid or vertical segment:
▪ 13.0 mm.
▪ In relation with the structures of posterior wall.
▪ From just behind the pyramid to stylomastoid foramen.
▪ Facial recess is present lateral to the facial nerve.
▪ Between tympanic and mastoid segments is the second genu of the nerve.

06. Auditory pathway up to cerebral cortex. Draw a labelled diagram of Organ of Corti. [2020](5+5)
➢ Auditory pathway:
o Inner hair cells
o Afferent nerve →
o Cochlear nerve →
o Spiral ganglion, exits through internal acoustic meatus →
o Dorsal and ventral cochlear nuclei in brain stem →
o Superior olivary nucleus →
o Nucleus of lateral lemniscus →

©shovandas 12
o Inferior colliculus →
o Medial geniculate body →
o Auditory cortex.
➢ Important points:
o The auditory pathway of both ear are interconnected:
▪ Each ear is represented in both cerebral hemisphere.
▪ Important for localization of sound.
▪ Interconnection starts at the level of superior olivary
nucleus.
o The area of cortex, concerned with hearing is situated in sup-
erior temporal gyrus (Brodmann’s area 41).
o Superior olivary nucleus is the starting point for localization of
sound.
➢ Organ of Corti:
o Organ of Corti is the sense organ of hearing & is situated on
the basilar membrane.
o Important components:
▪ Tunnel of Corti:
✓ Formed by the inner and outer rods.
✓ Contains a fluid called cortilymph.
✓ The exact function of the structures are not known.
▪ Hair cells:
✓ Receptor cells of hearing.
✓ Transduce sound energy into electrical energy.
▪ Supporting cell:
✓ Deiters’ cells: Situated between the outer hair cell & provide support to the latter.
✓ Cells of Hensen: Lie outside the Deiters’ cells.
▪ Tectorial membrane:
✓ Consists of gelatinous matrix with delicate fibres.
✓ Overlies the organ of Corti.
✓ The shearing force between hair cells & tectorial membrane produces the stimulus to hair cells.

07. Define Cholesteatoma. Describe the pathogenesis & management of the CSOM with cholesteatoma. [2017](2+3+5)
➢ Cholesteatoma:
o Normally middle ear cleft is lined by different type of epithelium:
▪ Ciliated columnar in the anterior and inferior part,
▪ Cuboidal in the middle part,
▪ Pavement-like in the attic.
o The middle ear is nowhere lined by squamous epithelium.
o The presence of cystic bag like structure lined by stratified squamous epithelium on a fibrous matrix in middle ear or
mastoid is called cholesteatoma.
o Contains desquamated squamous epithelium.
o In other words, cholesteatoma is a “skin in the wrong place.”
➢ CSOM with cholesteatoma:
o Atticoantral type of CSOM is associated with cholesteatoma.
o Involves posterosuperior part of middle ear cleft (attic, antrum, posterio tympanum and mastoid).
©shovandas 13
o Bcause of its bone eroding properties, causes risk of serious complications. For this reason, the disease is also called
unsafe or dangerous type.
➢ Pathogenesis:
o Cholesteatoma: Theories explaining appearance of sq. epithelium in middle ear:
▪ Presence of congenital cell rests.
▪ Wittmaack's theory: Retraction pocket.
▪ Ruedi's theory: Papillary in-growth.
▪ Habermann's theory: Epithelial migration.
▪ Sade's theory: Metaplasia or epidermidisation.
o Osteitis and granulation tissue:
▪ A mass of granulation tissue surrounds the area of an osteitis & may even fill the posterosuperior part of the
middle ear cleft.
▪ A fleshy red polypus may be seen filling the meatus.
o Ossicular necrosis:
▪ Destructions may be limited to the long process of incus or may also involve
stapes superstructure, handle of malleus or the entire ossicular chain.
▪ Cholesteatoma hearer: Occasionally, cholesteatoma bridges the gap caused by destroyed ossicles & hearing
loss is not apparent.
o Cholesterol granuloma:
▪ Mass of granulation tissue with foreign body giant cells surrounding the cholesterol crystals.
▪ It is a reaction to long-standing retention of secretions or haemorrhage, and may or may not co-exist with
cholesteatoma.
▪ When present in mesotympanum, behind an intact drum, latter appear blue.
➢ Management:
o No medical line of management.
o Aim of surgery:
▪ Primary aim: To remove the disease and render the ear safe.
▪ Secondary aim: To preserve or reconstruct hearing.
o Surgical management:
▪ Intact canal wall procedure:
✓ Posterior tympanotomy.
✓ To remove disease part from middle ear and mastoid →
opening made in facial recess area.
▪ Canal wall down procedure:
✓ Procedure:
Mastoid filled with cholesteatoma → removed until antrum →
remove wall between mastoid and middle ear →
middle ear and mastoid area converted into common cavity.
✓ Radical mastoidectomy:
Exenteration of mastoid air cells →
posterior canal lowered →
all mucosa and ossicles except foot plate of stapes removed →
eustachian tube is closed (to prevent further infection).
✓ Radical mastoidectomy is only done when there dead ear.
✓ Modified radical mastoidectomy:
Exenteration of mastoid air cells →
posterior canal lowered →
only unhealthy tissues are removed & reconstruction is done.
o Reconstructive surgery:
▪ Hearing can be restored by myringoplasty or tympanoplasty.
▪ It can be done at the time of primary surgery or as a second stage procedure.
o Conservative treatment:
▪ Done in selected cases, when cholesteatoma is small and easily accessible to suction clearance under an ope-
rating microscope.
▪ Repeated suction clearance and periodic check-ups are essential.
▪ Polyps and granulations can also be surgically removed by the cup forceps or cauterized by chemical agents
like silver nitrate or trichloroacetic acid.
▪ Other measures like aural toilet and dry ear precautions are also essential.

©shovandas 14
Group – b
01. What is deafness? Management of a case of conductive deafness in 25 year female patient. [2008]
➢ Deafness:
o The term deafness is used, when there is little or no hearing at all.
o WHO: In 1980 recommended that the term ‘deaf ’ should be applied only to those individuals whose hearing impair-
ment is so severe that they are unable to benefit from any type of amplification.
o Ministry of Social Welfare, Government of India - Scheme of Assistance to Hearing Handicap: ‘The deaf are those
in whom the sense of hearing is nonfunctional for ordinary purposes of life'.
o The cases included in the category will be those having hearing loss more than 90 dB in the better ear (profound
impairment) or total loss of hearing in both ears.
➢ Management of conductive deafness: Most cases of conductive hearing loss can be managed by medical or surgical means
with respect to their underlying causes.
o Removal of external canal obstructions:
▪ Procedure:
✓ Syringing. ✓ Forceps removal.
✓ Suction. ✓ Microscopic removal.
▪ Indications:
✓ Impacted wax. ✓ Keratotic mass.
✓ Foreign body. ✓ Benign/ malignant tumours.
✓ Osteoma or exostosis. ✓ Meatal atresia.
o Removal of fluid from middle ear:
▪ Procedure: Incision of TM i.e. myringotomy.
✓ To drain suppurative or nonsuppurative effusion of middle ear.
✓ To provide aeration in case of malfunctioning eustachian tube.
▪ Indications:
✓ Suppurative otitis media.
✓ Serous otitis media.
✓ Haemotympanum.
✓ Aero-otitis media to drain the fluids and to unlock the eustachian tube.
✓ Atelectatic ear.
o Removal of mass from middle ear:
▪ Procedure: Incision of TM i.e. myringotomy and removal of the mass by surgical excision.
▪ Indications:
✓ Small middle ear tumours.
✓ Cholesteatoma behind intact TM.
o Stapedectomy:
▪ Procedure: Fixation of stapes footplate.
▪ Indications: Otosclerosis.
o External ear reconstruction:
▪ Procedure:
✓ Rib cartilage graft reconstruction.
✓ Reconstruction using polythene plastic implant.
✓ Ear prosthesis.
▪ Indications: Congenital conductive hearing loss due to meatal atresia.
o Middle ear reconstruction:
▪ Procedure: Type of middle ear reconstruction depends on the damage present in the ear.
✓ Myringoplasty: Only repair of tympanic membrane.
✓ Ossiculoplasty: Reconstruction of ossicular chain.
✓ Tympanoplasty: Repair of both TM and ossicular chain.
▪ Indications:
✓ Myringoplasty: Perforation of tympanic membrane, traumatic or infective.
✓ Ossiculoplasty: Traumatic ossicular disruption, fixation of ossicles or necrosis.
✓ Tympanoplasty: Conditions affecting both TM and bone chain.
o Prosthesis:
▪ Incus prosthesis: When incus is missing but handle of malleus and stapes with superstructure are present
and functional.
▪ Incus-stapes prosthesis: When incus and stapes superstructure are missing. Malleus and stapes footplate
are functional.
▪ Partial ossicular replacement prosthesis (PORP): When malleus and incus are absent. Stapes is present and
mobile. PORP is placed between tympanic membrane and stapes head.
©shovandas 15
▪ Total ossicular replacement prosthesis (TORP): When malleus, incus and stapes superstructure are absent.
Only the stapes footplate is present and is mobile.
o Hearing aid: In cases, where surgery is not possible, refused or has failed.
o Medical management:
▪ Steroids in cases of inflammation.
▪ Antibiotics in cases of bacterial infective aetiology and eustachian tube blockage.

02. A middle aged male patient reported with bilateral conductive deafness. Diagnosis and management of case. [2016]
➢ Diagnosis:
o External canal obstruction: Wax, foreign body, furuncle, acute inflammatory swelling, benign or malignant tumour
or atresia of canal.
▪ History of recent manipulation of ear canal or insect in ear canal indicates foreign body ear.
▪ History of recent swimming indicates otitis externa.
▪ Physical examination of the auricle, periauricular area and external ear canal.
▪ Detailed otoscopic examination.
▪ Biopsy is essential for histological typing if there is any tumour.
o Perforation of tympanic membrane: Traumatic or infective.
▪ History of recent trauma.
▪ Detailed otoscopic examination.
▪ Fluid coming out from the ear from perforated TM in cases of CSOM.
o Fluid in the middle ear: Acute otitis media, serous otitis media or haemotympanum.
▪ Examination under microscope.
▪ X-ray mastoid or CT scan of temporal bone to see the extent of infection or presence of bone destruction.
▪ Culture and sensitivity of ear discharge.
o Mass in middle ear: Benign or malignant tumour.
▪ Combined MRI and CT scan of head.
▪ Four-vessel angiography is necessary when CT head shows involvement of jugular bulb, carotid artery or
intradural extension.
▪ Biopsy is essential for histological typing of the tumour.
o Disruption of ossicles: Trauma to ossicular chain, chronic suppurative otitis media, cholesteatoma.
▪ History of recent trauma.
▪ History of recent upper respiratory tract infection.
▪ Examination under microscope.
o Fixation of ossicles: Otosclerosis, tympanosclerosis, adhesive otitis media, congenital stapes footplate fixation,
congenital fixation of malleus head.
▪ Air-bone gap during audiometry indicates ossicular fixation.
▪ Carhart's notch during audiometry indicates otosclerosis.
o Eustachian tube blockage: Retracted tympanic membrane, serous otitis media.
▪ Direct otoscopic examination shows retracted tympanic membrane.
▪ History of recent upper respiratory tract infection.
▪ Rhinoscopy, rigid nasal endoscopy or flexible nasopharyngoscopy to see the pharyngeal end of the tube.
▪ Microscopic or endoscopic examination to see the tympanic end.
▪ Assessment of function of the tube can be made by Valsalva, politzerization, Toynbee and other tests.
o Congenital causes: Meatal atresia, ossicular discontinuity.
▪ Meatal atresia can be identified by direct physical examination.
▪ Air bone gap indicates ossicular discontinuity.
▪ CT scan confirms the discontinuity of ossicular chain.
➢ Management: Group B, question no. 01.

03. Aetiopathology, clinical features, management of otitis media with effusion. [2014]
➢ Otitis media with effusion:
o Also called as serous otitis media, Secretory otitis media, mucoid otitis media or ‘glue ear’.
o It is an insidious condition characterized by accumulation of nonpurulent nearly sterile effusion in the middle ear
cleft, which is naturally thick and viscid but may be thin and serous.
➢ Pathogenesis:
o Malfunctioning of eustachian tube: Fails to aerate the middle ear and drain the fluid.
o Increased secretory activity of middle ear mucosa: Increase in number of mucus or serous-secreting cells.
➢ Aetiology:
o Malfunctioning of eustachian tube:
▪ Adenoid hyperplasia.
▪ Chronic rhinosinusitis.
▪ Chronic tonsillitis.
©shovandas 16
▪ Benign and malignant tumours of nasopharynx.
▪ Palatal defects, e.g. cleft palate, palatal paralysis.
o Allergy: Obstructs eustachian tube by oedema and increases secretory activity of middle ear mucosa.
o Unresolved otitis media: Inadequate antibiotic therapy in acute suppurative otitis media may inactivate infection
but fail to resolve it completely, which can stimulate mucosa to secrete more fluid along with increase in number of
goblet cells and mucous glands.
o Viral infections: Adeno and rhinoviruses invade middle ear mucosa and stimulate it to increased secretory activity.
➢ Clinical features:
o Symptoms:
▪ Generally affects children of 5–8 years of age.
▪ Hearing loss: Insidious in onset and rarely exceeds 40 dB.
▪ Delayed and defective speech: Because of hearing loss.
▪ Mild earaches: Due to viral infections.
o Otoscopic findings:
▪ Dull retracted TM.
▪ Fluid level seen.
▪ Air bubbles present.
▪ Glue ear: Due to precipitation of fluid inside TM.
▪ Yellow, green or bluish colour of TM.
▪ Features of retracted TM:
✓ Prominent lateral process.
✓ Handle of malleus is shortened.
✓ Sickling of malleolar fold.
✓ Loss of light reflex.
o Hearing tests:
▪ Rinne test: Negative.
▪ Weber's test: Lateralized to worst ear.
▪ ABC test: Normal.
▪ Schwabach test: Lengthened.
▪ AB gap: Present.
▪ Tympanogram: B type of curve, best investigation.
o X-ray mastoids: Clouding of air cells due to fluid.
➢ Management:
o Aim:
▪ Removal of fluid.
▪ Prevention of its recurrence.
o Medical management:
▪ Decongestants: Nasal drops, sprays or systemic decongestants to relieve oedema of eustachian tube.
▪ Antiallergic measures: Antihistaminics or steroids.
▪ Antibiotics: In cases of upper respiratory tract infections or unresolved acute suppurative otitis media.
▪ Middle ear aeration: Repeated Valsalva manoeuvre, politzerization or eustachian tube catheterization to
ventilate middle ear & promote drainage of fluid. For children chewing gum is given to encourage repeated
swallowing which opens the tube.
o Surgical management: When fluid is thick and medical treatment alone is not enough.
▪ Myringotomy and aspiration of fluid: Incision on TM and fluid aspiration by suction.
▪ Grommet insertion: If myringotomy and aspiration combined with medical measures have not helped and
fluid recurs, a grommet is inserted to provide continued aeration.
▪ Tympanotomy or cortical mastoidectomy: Removal of loculated thick fluid or other associated pathology
such as cholesterol granuloma.
▪ Surgical treatment of causative factor: Adenoidectomy, tonsillectomy and/or wash-out of maxillary antrum
may be required. This is usually done at the time of myringotomy.

04. Describe the intratemporal course of the facial nerve. Enumerate the intratemporal branches of the facial nerve and their
site of origin. What is Bells Palsy? [2018](5+3+2)
➢ Infratemporal course of facial nerve: Group C, question no. 14.
➢ Branches with origin: Group C, question no. 14.
➢ Bells palsy: Group C, question no. 15.

05. A middle aged patient presents in the ENT OPD with intermittent mucopurulent discharge from one ear. How will you
investigate & treat the case? [2015, 2003](5+5)
➢ Investigation:
o History taking:
©shovandas 17
▪ Odour of discharge:
✓ Odourless discharge indicates tubotympanic type of CSOM.
✓ Foul smelling discharge indicates atticoantral type of CSOM.
▪ Blood in discharge: Generally indicates rupture of TM.
▪ Time of discharge: To find out acute or chronic onset.
▪ Associated history:
✓ Previous onset of acute otitis media.
✓ Recent upper respiratory tract infection.
o Clinical examinations:
▪ Examination under microscope:
✓ Presence of TM perforation.
✓ Presence of granulations, ingrowth of squamous epithelium from the edges of perforation, status
of ossicular chain, tympanosclerosis and adhesions indicate tubotympanic type CSOM.
✓ Presence of cholesteatoma, evidence of bone destruction, granuloma indicate atticoantral CSOM.
▪ Audiogram: Gives an assessment of degree of hearing loss and its type. Usually, the loss is conductive but a
sensorineural element may be present.
▪ Tuning fork tests: Can also be done to know the degree and type of hearing loss.
o Radiological examinations: Mastoid X ray or CT scan of temporal bone.
▪ Mastoid is usually sclerotic but may be pneumatized with clouding of air cells. There is no evidence of bone
destruction indicates tubotympanic type CSOM.
▪ Presence of bone destruction is a feature of atticoantral type CSOM. Cholesteatoma causes destruction in
the area of attic and antrum, better seen in lateral view.
o Culture and sensitivity of ear discharge: It helps to select proper antibiotic for local or systemic use.
➢ Treatment:
o Aural toilet: By dry mopping with absorbent cotton buds, suction clearance under microscope or irrigation with
sterile normal saline. Ear must be dried after irrigation.
o Ear drops:
▪ Antibiotic ear drops containing neomycin, polymyxin, chloromycetin or gentamicin are used.
▪ Antibiotics are combined with steroids which have local anti-inflammatory effect.
▪ Acid pH helps to eliminate pseudomonas infection, and irrigations with 1.5% acetic acid are useful.
o Systemic antibiotics: Only in acute exacerbation of chronically infected ear.
o Precautions: Water should be kept out of the ear during bathing, swimming and hair wash. Rubber inserts can be
used. Hard nose blowing can also push the infection from nasopharynx to middle ear and should be avoided.
o Treatment of contributory causes: Infected tonsils, adenoids, maxillary antra and nasal allergy.
o Surgical treatment:
▪ Aural polyp or granulation, if present, should be removed before local treatment with antibiotics. It will
facilitate ear toilet and permit ear drops to be used effectively. An aural polyp should never be avulsed as
it may be arising from stapes, facial nerve or horizontal canal and thus lead to facial paralysis or labyrinthitis.
▪ If there is cholesteatoma in case of atticoantral CSOM, canal wall up or canal wall down surgery is done.
o Reconstructive surgery: Once ear is dry, myringoplasty with or without ossicular reconstruction can be done to re-
store hearing. Closure of perforation will also check repeated infection from the external canal.

06. How will you investigate a case of persisting unilateral aural discharge with hearing loss for last three years in a 35 year old
male patient. [2019]
➢ Group B, question no. 05.

07. How do you investigate a case of long standing foul smelling ear discharge of a child of 8 years? [2008]
➢ Group B, question no. 05.

08. A 65 years old male patient presents with pain in right ear. How will you investigate the case. [2014, 2002, 1999, 1994]
➢ Differential diagnosis of ear pain:
o Local causes:
▪ External ear pathology: Furuncle, impacted wax, otitis externa, otomycosis, myringitis bullosa, herpes zos-
ter and malignant neoplasms.
▪ Middle ear pathology: Acute otitis media, eustachian tube obstruction, mastoiditis, extradural abscess,
aero-otitis media and carcinoma middle ear.
▪ Mastoid pathology: Mastoiditis, CSOM.
▪ Inner ear pathology: Labyrinthitis.
o Referred causes:
▪ Via Vth cranial nerve:
✓ Dental: Caries tooth, apical abscess, impacted molar, malocclusion and Costen syndrome.
✓ Oral cavity: Benign or malignant ulcerative lesions of oral cavity or tongue.
©shovandas 18
✓ TMJ disorders: Bruxism, osteoarthritis, recurrent dislocation, ill-fitting denture.
✓ Sphenopalatine neuralgia.
▪ Via IXth cranial nerve:
✓ Oropharynx: Acute tonsillitis, peritonsillar abscess, tonsillectomy. Benign or malignant ulcers of
soft palate, tonsil and its pillars.
✓ Base of tongue: Tuberculosis or malignancy.
✓ Elongated styloid process.
▪ Via Xth cranial nerve: Malignancy or ulcerative lesion of vallecula, epiglottis, larynx or laryngopharynx and
oesophagus.
▪ Via C2 and C3 spinal nerves: Cervical spondylosis, injuries of cervical spine and caries spine.
➢ History taking:
o Site: The patient should point with one finger. This is important to know that, the pain is in external canal, middle
ear or inner ear.
o Onset: Acute or chronic.
o Character of the pain
o Radiation of the pain.
o Referred site if any.
o Associated feature: Otologic (discharge, tinnitus, hearing loss and vertigo) and systemic.
▪ Discharge: Otitis externa, otitis media with TM perforation.
▪ Tinnitus: Noise trauma, whiplash, head injuries, ear wax or medication side effects.
▪ Vertigo: Labyrinthitis.
o Timing.
o Aggravating factors: Pain in aggravated by chewing in cases of TMJ disorders.
o Severity.
o Risk factors for tumour:
▪ Age older than 50 years.
▪ Tobacco addiction.
▪ Alcohol abuse.
o Special history:
▪ History of recent upper respiratory tract infection → otitis media.
▪ History of recent swimming → otitis externa.
▪ History of recent manipulation of ear canal or insect in ear canal → foreign body ear.
▪ History of recent airplane travelling or scuba diving → barotrauma.
▪ History of recent trauma (eg. noise trauma) to the ear.
➢ Physical examination:
o Examination of auricle and periauricular area: Tenderness during traction on the auricle or pressure on the tragus
indicates pathology in external auditory canal, usually otitis externa.
o Otoscopic examination: Detailed otoscopic examination is done to diagnose foreign body ear, impacted wax, otitis
externa, abnormal or perforated TM, middle ear examination in cases of perforated TM.
o Tests for hearing loss: Simple audiometric tests and tuning fork tests are done to assist type and extend of hearing
loss associated with earache for better diagnosis.
o Examination of TMJ: Referred pain due to Vth cranial nerve. TMJ should be palpated for tenderness and crepitus as
the patient opens and closes the mouth.
o Inspection of oral cavity: Laryngoscopy, pharyngoscopy for referred pain due to Vth, IXth, Xth cranial nerve.
o Inspection of teeth: Referred pain due to Vth cranial nerve.
o Examination of head neck: Referred pain via C2 and C3 spinal nerve.
o Facial nerve tests: Done to investigate any case of nerve compression.
o Radiological examination: Mastoid X ray or CT scan of temporal bone to diagnose mastoid pathologies.

09. Define CSOM. Discuss the aetiology, pathology, clinical features & management of the mucosal type of CSOM. (2+2+3+3)
➢ Aetiology:
o It is the sequela of acute otitis media usually following exanthematous fever and leaving behind a large central
perforation. The perforation becomes permanent and permits repeated infection from the external ear. Also the
middle ear mucosa is exposed to the environment and gets sensitized to dust, pollen and other aeroallergens causing
persistent otorrhoea.
o Ascending infections via the eustachian tube from tonsils, adenoids and infected sinuses may be responsible for
persistent or recurring otorrhoea.
o Persistent mucoid otorrhoea is sometimes the result of allergy to ingestants such as milk, eggs, fish, etc.
➢ Pathology: This remains localized mostly to the mucosa of anteroinferior part of the middle ear cleft. The pathological changes
seen in this type of CSOM are:
o Perforation of pars tensa: Central perforation with variable size and position.

©shovandas 19
o Middle ear mucosa:
▪ Normal when disease is quiescent or inactive.
▪ Oedematous and velvety when disease is active.
o Polyp: A polyp is a smooth mass of oedematous and inflamed mucosa which has protruded through a perforation
and presents in the external canal. It is usually pale in contrast to pink, fleshy polyp seen in atticoantral disease.
o Ossicular chain: Usually intact & mobile but may show some degree of necrosis, particularly of long process of incus.
o Tympanosclerosis: It is hyalinization and subsequent calcification of subepithelial connective tissue. It is seen in
remnants of TM or under the mucosa of middle ear. It is seen as white chalky deposit on the promontory, ossicles,
joints, tendons and oval and round windows. Tympanosclerotic masses may interfere with the mobility of these
structures and cause conductive deafness.
o Fibrosis and adhesions: They are the result of healing process and may further impair mobility of ossicular chain or
block the eustachian tube.
➢ Clinical features:
o Ear discharge:
▪ Nonoffensive.
▪ Mucoid or mucopurulent.
▪ Constant or intermittent.
o Hearing loss:
▪ Type: Conductive hearing loss.
▪ Severity: Varies but rarely exceeds 50 dB.
▪ Paradoxical hearing: Sometimes, the patient reports of a paradoxical effect, i.e. hears better in the presence
of discharge than when the ear is dry. This is due to ‘round window shielding effect’ produced by discharge
which helps to maintain phase differential. In the dry ear with perforation, sound waves strike both the
oval and round windows simultaneously, thus cancelling each other’s effect.
▪ SNHL in long standing cases: Cochlea may suffer damage due to absorption of toxin from the oval and round
windows and hearing loss becomes mixed type.
o Perforation:
▪ Type: Always central.
▪ Position: Anterior, posterior or inferior to the handle of malleus.
▪ Size: Small, medium or large or extending up to the annulus, i.e. subtotal.
o Middle ear mucosa:
▪ Seen in cases of large perforation.
▪ Normally pale pink and moist.
▪ When inflamed it looks red, oedematous and swollen.
o Occasionally, a polyp may be seen.
➢ Management: Group B, question no. 05.

10. A patient aged 30 years presented with deafness of both ears. Investigate to diagnose type of deafness, manage the case.
[1997](5+5)
➢ Investigation:
o Indicating conductive deafness:
▪ Negative Rinne test, i.e. BC > AC.
▪ Weber lateralized to poorer ear.
▪ Normal absolute bone conduction.
▪ Low frequencies affected more.
▪ Audiometry shows bone conduction better than air conduction with air-bone gap. Greater the air-bone gap,
more is the conductive loss.
▪ Loss is not more than 60 dB.
▪ Speech discrimination is good.
o Indicating SNHL:
▪ A positive Rinne test, i.e. AC > BC.
▪ Weber lateralized to better ear.
▪ Bone conduction reduced on Schwabach and absolute bone conduction tests.
▪ More often involving high frequencies.
▪ No gap between air and bone conduction curve on audiometry.
▪ Loss may exceed 60 dB.
▪ Speech discrimination is poor.
▪ There is difficulty in hearing in the presence of noise.
➢ Management:
o Conductive deafness:
▪ Removal of canal obstructions: Impacted wax, foreign body, osteoma or exostosis, keratotic mass, benign
or malignant tumours, or meatal atresia.
©shovandas 20
▪ Removal of fluid: Myringotomy with or without grommet insertion.
▪ Removal of mass from middle ear: Tympanotomy & removal of small middle ear tumour or cholesteatoma
behind intact tympanic membrane.
▪ Stapedectomy: In otosclerotic fixation of stapes footplate.
▪ Tympanoplasty: Repair of perforation, ossicular chain or both.
▪ Hearing aid: In cases, where surgery is not possible, refused or has failed.
o SNHL:
▪ Syphilis of the inner ear is treatable with high doses of penicillin and steroids with improvement in hearing.
▪ Hearing loss of hypothyroidism can be reversed with replacement therapy.
▪ Serous labyrinthitis can be reversed by attention to middle ear infection.
▪ Early management of Ménière’s disease can prevent further episodes of vertigo and hearing loss.
▪ SNHL due to perilymph fistula can be corrected surgically by sealing the fistula in the oval or round window
with fat.
▪ Ototoxic drugs should be used with care and discontinued if causing hearing loss. In many such cases, it
may be possible to regain hearing, total or partial, if the drug is stopped.
▪ Noise induced hearing loss can be prevented from further deterioration if the person is removed from the
noisy surroundings.

©shovandas 21
©shovandas 22
GROUP – C
01. Basal cell carcinoma. [2020]
➢ BCC is the most common, slow-growing invasive cancer that rarely metastasizes.
➢ Sites:
o Occur at sun-exposed sites and in fair-skinned people.
o Usual site is above a line drawn from angle of mouth to the pinna of the ear.
o Commonest sites are the helix and the tragus.
o Can rarely arise from external acoustic meatus.
➢ Origin:
o Have cytological similarity of tumor cells to the normal basal cells of epidermis.
o It is believed to be primitive ‘adnexal’ carcinomas probably of indubitable hair follicle derivation.
➢ Etiology and pathogenesis:
o More common in men.
o Age: more than 50 years of age.
o The incidence is more in patients with immunosuppression.
o Genetics:
▪ Mutation in tumor suppressor gene:
✓ Mutation in PTCH:
• Sporadic form:
30% .
May be due to UV damage.
• Germline mutation:
Nevoid basal cell carcinoma or basal cell nevus or Gorlin syndrome.
Autosomal dominant.
✓ TP53 mutations: 40–60% of cases.
▪ Defect in DNA repair genes: Xeroderma pigmentosum.
➢ Clinical Presentation:
o Appear as pearly papules containing prominent dilated subepidermal blood vessels.
o Advanced tumors may ulcerate, and locally invade and erode the underlying bone or facial sinuses like a rodent and
are known as rodent ulcers.
o Lymph node metastasis is very rare.
➢ Diagnosis:
o Made only on biopsy and microscopic examination.
o Microscopic appearance:
▪ Resemble the normal basal cell layer of the epidermis.
▪ Show peripheral palisading.
➢ Treatment:
o Superficial lesions, not involving cartilage can be irradiated & cosmetic deformity can be avoided.
o Lesions involving cartilage may require surgical excision and post-op radiation.

02. Sensory supply of external ear. [2020]


➢ Pinna:
o Greater auricular nerve (C2,3): Supplies most of the medial surface of pinna and only posterior part of lateral surface.
o Lesser occipital (C2): Supplies upper part of medial surface.
o Auriculotemporal (V3): Supplies tragus, crus of helix & the adjacent part of the helix.
o Auricular branch of vagus (CN X): Also called Arnold’s nerve, supplies the concha and corresponding eminence on
the medial surface.
o Facial nerve: Distributed with fibres of auricular branch of vagus supplies concha and retroauricular groove.

©shovandas 23
➢ External auditory canal:
o Anterior wall and roof: auriculotemporal (V3).
o Posterior wall and floor: auricular branch of vagus (CN X).
o Posterior wall of auditory canal also receives sensory fibres of CN VII through the auricular branch of vagus.
➢ Tympanic membrane:
o Anterior half of lateral surface: auriculotemporal (V3).
o Posterior half of lateral surface: auricular branch of vagus (CN X).
o Medial surface: tympanic branch of CN IX (Jacobson’s nerve).

03. Fistula test. [1994, 2014]


➢ Principal: To induce nystagmus by producing pressure changes in the external canal which are then transmitted to the laby-
rinth by a fistula. Stimulation of the labyrinth results in nystagmus and vertigo.
➢ Procedure: The test is performed by applying intermittent pressure on tragus or by using Siegel’s speculum.
➢ Observation:
o Negative:
▪ For normal person.
▪ Pressure change in external canal can’t be transmitted to labyrinth.
o Positive:
▪ Fistula in the horizontal semicircular canal:
✓ In cholesteatoma.
✓ Surgically created window in fenestration operation.
▪ Fistula in the oval window: Post-stapedectomy fistula.
▪ Fistula in the round window: Rupture of the round window membrane.
▪ Fistula in the promontory.
▪ A positive fistula also implies that the labyrinth is still functioning.
o False negative:
▪ Dead labyrinth.
▪ Fistula covered by cholesteatoma because, the cholesteatoma does not allow pressure changes to be trans-
mitted to the labyrinth.
o False positive:
▪ Movements of stapes result in stimulation of the utricular macula.
▪ Congenital syphilis: Stapes footplate is hypermobile.
▪ Ménière’s disease: Fibrous band connecting the utricular macula to stapes footplate (Hennebert's sign).

04. Caloric test. [2012]


➢ Principal: To induce nystagmus by thermal stimulation of the vestibular system.
➢ Advantage:
o Each labyrinth can be tested separately.
o Patient is also asked that whether vertigo induced by caloric test is qualitatively similar to the type experienced by
him during the episode of vertigo. If yes then it proves labyrinthine origin of vertigo.
➢ Modified Kobrak test:
o Procedure:
▪ Pt. is seated & head tilted 60° backwards to horizontal canal be vertical →
▪ Ear is irrigated with ice water for 60s with 5 mL →
▪ If there is no response, 10, 20 and 40 mL.
o Observation:
▪ Normal labyrinth: Nystagmus in opposite ear seen with 5 mL of ice water.
▪ Hypoactive labyrinth: Response with increased water between 5 and 40mL.
▪ Dead labyrinth: No response to 40mL of water.
➢ Fitzgerald Hallpike test (bithermal caloric test):
o Procedure:
▪ Patient is supine & head tilted 30° forward to horizontal canal be vertical →
▪ Ears are irrigated for 40s alternately with water at 30°C & 44°C →
▪ Eyes are observed for appearance of nystagmus till its end point →
▪ Time taken from start of irrigation to end point of nystagmus is recorded →
▪ Charted on calorigram →
▪ If negative, with water at 20°C for 4 min before labelling labyrinth dead →
▪ A gap of 5 min should be allowed between two ears.
o Cold water induces nystagmus to opposite side & warm water to the same side.
o Observation:
▪ Canal paresis:

©shovandas 24
✓ It indicates that the response elicited from a particular canal after stimulation is less than that from
the opposite side.
✓ Indicating depressed function of ipsilateral:
• Labyrinth,
• Vestibular nerve or
• Vestibular nuclei.
✓ Seen in:
• Ménière’s disease,
• Acoustic neuroma,
• Postlabyrinthectomy,
• Vestibular nerve section.
▪ Directional preponderance:
✓ If the nystagmus is 25–30% or more on one side than other, it is called directional preponderance
to that side.
✓ Directional preponderance occurs towards the side of central lesion and away from the side of a
the peripheral lesion.
▪ Both:
✓ Canal paresis on one side with directional preponderance to opposite side is seen in case of unila-
teral Ménière’s disease.
✓ Canal paresis with directional preponderance to ipsilateral side seen in acoustic neuroma.
➢ Cold-air caloric test:
o This test is done when there is perforation of TM because irrigation with water in such a case with perforation is
contraindicated.
o The test employs Dundas Grant tube, a coiled copper tube wrapped in cloth.
o The air in the tube is cooled by pouring ethyl chloride & then blown into the ear.
o A rough qualitative test.

05. Cerumen. [2007]


➢ Ear wax.
➢ Composition:
o Secretion of sebaceous glands:
▪ Modified sweat glands.
▪ Open into the place of hair follicles.
▪ Fluid rich in fatty acids.
o Secretion of ceruminous glands:
▪ Modified sweat glands.
▪ Open into the place of hair follicles.
▪ Fluid rich in lipids and pigment granules.
o Hair, o Keratin and
o Desquamated epithelial debris, o Dirt.
➢ Beneficial effects:
o Entraps any foreign material that happens to enter the canal.
o Due to acidic pH it is bacteriostatic and fungistatic.
➢ Plug formation:
o Cerumen may dry up and form a hard impacted mass.
o Causes:
▪ Hyperactive sweat glands & modified sweat gland in some people,
▪ Narrow and tortuous ear canal,
▪ Stiff hair,
▪ Obstructive lesion of the canal, e.g. exostosis.
o Symptoms:
▪ Acute: When water enters ear canal & the wax sells up.
✓ Impairment of hearing or sense of blocked ear.
✓ Tinnitus and giddiness may result from impaction of wax against TM.
✓ Reflex cough due to stimulation of auricular branch of vagus.
▪ Chronic: Wax may ulcerate meatal skin & form wax granuloma.
o Management:
▪ Syringing:
✓ Procedure:
• Water at body temperature →
• Postero-superior direction →
• At moderate pressure →
©shovandas 25
• Injected by a syringe into the external auditory canal.
✓ Complications:
• Vertigo: If water is not at body temperature.
• TM rupture: If water is injected at high pressure.
✓ Contraindication: Perforated TM.
▪ Instrumental manipulation:
✓ Cerumen hook, scoop or Jobson Horne probe: Wax is removed by passing probe behind it.
✓ Wax is held with forceps and removed.
▪ Softening of wax:
✓ If the wax is too hard & impacted, to be removed by syringing or instruments, it should be softened.
✓ Softeners:
• 5% sodium bicarbonate in equal parts of glycerine & water,
• Hydrogen peroxide,
• Liquid paraffin,
• Olive oil,
• Commercial drop containing ceruminolytic agents like para dichlorobenzene 2%.

06. Cholesteatoma. [2008,2016]


➢ Introduction:
o Normally middle ear cleft is lined by different type of epithelium:
▪ Ciliated columnar in the anterior and inferior part,
▪ Cuboidal in the middle part,
▪ Pavement-like in the attic.
o The middle ear is nowhere lined by squamous epithelium.
o The presence of cystic bag like structure lined by stratified squamous epithelium on a fibrous matrix in the middle
ear or mastoid is called cholesteatoma.
o Contains desquamated squamous epithelium.
o In other words, cholesteatoma is a “skin in the wrong place.”
➢ Etiology:
o Otitis media with effusion.
o Eustachian tube dysfunction.
o Cranio-facial abnormalities.
o Genetics and ethnicity: Highest among Caucasians, followed by Africans, rare in non-Indian Asians.
o Environment.
➢ Genesis of cholesteatoma:
o Presence of congenital cell rests.
o Wittmaack's theory: Retraction pocket.
o Ruedi's theory: Papillary in-growth.
o Habermann's theory: Epithelial migration.
o Sade's theory: Metaplasia or epidermidisation.
➢ Classification:
o Congenital cholesteatoma:
▪ White mass behind intact TM.
▪ Arises from congenital epidermal cell rests.
▪ Sites:
✓ Middle ear,
✓ Petrous apex,
✓ Cerebellopontine angle.
o Primary acquired cholesteatoma,
o Secondary acquired cholesteatoma.
➢ Progression:
o Progression towards healing: Auto-mastoidectomy.
o Progression towards complication: Osteitis → Bone erosion → Granulation tissue → Complications.
➢ Symptoms:
o Hearing loss:
▪ Ossicular erosion → Pars tensa retraction → Pars flaccida retraction.
▪ Cholesteatoma bridge reduses hearing loss.
o Ear discharge: Scanty foul smelling.
o Complications, if any.
➢ Diagnosis:
o Otoscopy.
o Hearing assessment: Tuning fork audiometry.
©shovandas 26
o Radiology:
▪ CT scan 1.5 mm cuts.
▪ X-ray skull lateral oblique view.
➢ Management:
o Aural suction & clearance.
o Medical management:
▪ Topical aminoglycosides +/- steroids.
▪ Fluoroquinolones.
o Surgical management:
▪ Mastoid exploration:
✓ Radical mastoidectomy,
✓ Modified radical mastoidectomy,
✓ CWU mastoidectomy.
▪ Tympanoplasty.
▪ Ossiculoplasty.

07. Malignant otitis externa. [2000, 2011,2013]


➢ Not a malignancy but mimicking malignancy.
➢ Also called necrotizing otitis externa.
➢ Etiology and pathogenesis:
o An inflammatory condition caused by pseudomonas infection.
o Usually seen in the immunocompromised patients like:
▪ Elderly diabetics,
▪ Patients on immunosuppressive drugs,
▪ HIV patients.
➢ Clinical features:
o Early manifestations resemble diffuse otitis externa.
o Excruciating pain.
o Appearance of granulations in the ear canal.
o Spreading of infection:
▪ Earliest to be involved is facial nerve causing facial
paralysis is common.
▪ Infection may spread to the skull base and jugular
foramen causing multiple cranial nerve palsies.
▪ Anteriorly: Infection spreads to temporomandibular
fossa.
▪ Posteriorly: Infection spreads to the mastoid.
▪ Medially: Infection spreads into the middle ear and
petrous bone.
➢ Diagnosis:
o Technetium 99m bone scan:
▪ For early diagnosis.
▪ Shows increased uptake due to osteoblastic activity.
▪ But test remains positive for a year or so and cannot be used to monitor the disease.
o Gallium-67:
▪ Useful in diagnosis and follow-up of the patient.
▪ Taken up by monocytes and reticuloendothelial cells, and is indicative of soft tissue infection.
o Biopsy:
▪ No mitotic activity.
▪ Only granulation tissue.
o CT scan may show bony destruction but is often not helpful.
➢ Management:
o Antibiotics against Pseudomonas:
▪ Gentamicin + ticarcillin i.v.
▪ Third-generation cephalosporins: Ceftriaxone 1–2 g/day i.v. or ceftazidime 1–2 g/day i.v. combined with an
aminoglycoside.
▪ Quinolones: Ciprofloxacin (oral), ofloxacin and levofloxacin.
o Ear toileting.
o Control of diabetes.
o Follow-up:
▪ Gallium-67 or Indium scan: Not commonly done.
▪ ESR: Easy and commonly used.
©shovandas 27
08. Circumscribed otitis externa. [2010]
➢ Also called furuncle or localized acute otitis externa.
➢ Pathogenesis: A furuncle is a staphylococcal infection of the hair follicle.
➢ Site: As the hair are confined only to cartilaginous part of the meatus, furuncle is seen only in this part of meatus.
➢ Appearance:
o Usually single.
o May be multiple.
➢ Clinical features:
o Severe pain and tenderness which are out of proportion to the size of the furuncle.
o Movements of the pinna are painful.
o Jaw movements, as in chewing, also cause pain in the ear.
o Furuncle of posterior meatal wall causes oedema over mastoid with obliteration of retroauricular groove.
o Periauricular lymph nodes (anterior, posterior and inferior) may also be enlarged and tender.
➢ Management:
o In early cases without abscess formation:
▪ Systemic antibiotics against staphylococcus e.g. Amoxi-clav.
▪ Analgesics.
▪ Local heat.
▪ Ear pack of 10% ichthammol glycerine provides splintage and reduces pain. Hygroscopic action of glycerine
reduces oedema, while ichthammol is mildly antiseptic.
o If there is abscess formation: Incision and drainage.
o Recurrent furunculosis:
▪ Diabetes should be excluded.
▪ Attention paid to the patient’s nasal vestibules which may harbour staphylococci and the infection trans-
ferred by patient’s fingers.
▪ Staphylococcal infections of skin as a possible source should also be excluded and suitably treated.

09. External auditory canal. [2001]


➢ Extends from the bottom of the concha to the tympanic membrane.
➢ Length: 24 mm.
➢ Shape:
o Not a straight tube.
o Outer part is directed upwards, backwards and medially.
o Inner part is directed downwards, forwards and medially.
o To see the tympanic membrane, the pinna has to be pulled upwards, backwards and laterally.
➢ Parts: The canal is divided into two parts: (i) cartilaginous and (ii) bony.
o Cartilaginous part:
▪ Forms outer one-third (8 mm) of the canal.
▪ Cartilage: Continuation of the cartilage which forms the framework of the pinna.
▪ Skin:
✓ Thick.
✓ Lining epithelium is same as skin.
▪ Glands: Ceruminous and pilosebaceous glands which secrete wax.
▪ Hair: Present and therefore furuncles are seen.
▪ Fissures of Santorini: Two deficiencies in this part of the cartilage and
through them the parotid or superficial mastoid infections can appear
in the canal or vice versa.
o Bony part:
▪ Forms inner two-thirds (16 mm) of the canal.
▪ Skin:
✓ Thin.
✓ Continuous over the tympanic membrane.
✓ Lining epithelium is stratified squamous epithelium without skin appendages.
▪ Hair: Absent.
▪ Glands: Absent.
▪ Isthmus:
✓ About 6 mm lateral to tympanic membrane, the narrowing in the bony meatus.
✓ Foreign bodies, lodged medial to the isthmus, get impacted, and are difficult to remove.
▪ Anterior recess:
✓ Recess present in the anteroinferior part of the deep meatus, beyond the isthmus.
✓ Acts as a cesspool for discharge and debris in cases of external and middle ear infections.

©shovandas 28
▪ Foramen of Huschke: Anteroinferior part of the bony canal may present this deficiency in children up to the
age of four or sometimes in adults, permitting infections to and from the parotid.

10. Lateral sinus thrombosis. [2018]


➢ It is an inflammation of inner wall of lateral venous sinus with formation of an intrasinus thrombus.
➢ Etiology: As a complication of:
o Acute coalescent mastoiditis,
o Masked mastoiditis,
o CSOM and cholesteatoma.
o Bacteriology:
▪ In acute infections: Haemolytic streptococcus, pneumococcus, staphylococcus.
▪ In chronic infection with cholesteatomas: proteus, Pseudomonas pyocyaneus, E. coli, Staphylococci.
➢ Pathogenesis:
o Complications spread into outer dural wall sigmoid sinus →
o Formation of perisinus abscess →
o Inflammation in inner wall of sinus with deposition of fibrin, platelets and blood cells →
o Endophlebitis and mural thrombus formation →
o Enlargement of mural thrombus →
o Complete occlusion of the sinus lumen →
o Invasion of organisms causing intrasinus abscess →
o Extension of thrombus:
▪ Proximally: To confluence of sinuses and to superior sagittal sinus or cavernous sinus.
▪ Distally: Into mastoid emissary vein, to jugular bulb or jugular vein.
➢ Clinical features:
o Hectic Picket-fence type of fever:
▪ Infected thrombus into the blood → septicaemia →
▪ Fever with chills and rigors (resembles malaria but lacks regularity).
o Headache.
o Progressive anaemia and emaciation.
o Griesinger’s sign:
▪ Thrombosis of mastoid emissary vein →
▪ Non tender pitting edema over posterior part of mastoid.
o Papilloedema:
▪ Due to thrombosed right sinus or clot in superior saggital sinus.
▪ Fundus: Blurring of disc margins, retinal haemorrhages, dilated veins.
o Tobey-Ayer test:
▪ Thrombosis in IJV of affected site → IJV of other side takes up the function.
▪ Compression of affected IJV → no change in cranial CSF pressure.
▪ Compression of normal IJV → rapid rise in cranial CSF pressure.
o Crowe-Beck test: Pressure on jugular vein of healthy side → engorgement of retinal veins and supraorbital veins.
o Tenderness along jugular vein: Due to thrombophlebitis along the jugular vein. Associated with enlargement and
infla-mmation of jugular chain of lymph nodes and torticollis.
➢ Diagnosis:
o Blood smear: To rule out malaria.
o Blood culture: To find causative organisms.
o CSF examination: To exclude meningitis and to measure CSF pressure.
o Radiology:
▪ X-ray mastoid: Clouding of air cells (acute mastoiditis) or destruction of bone (cholesteatoma).
▪ Contrast-enhanced CT scan & MRI:
✓ Empty triangle sign: Thrombus does not take up contrast and walls appear empty.
✓ Delta sign.
o Culture and sensitivity of ear swab.
➢ Complications:
o Septicaemia and pyaemic abscesses in lung, bone, joints or subcutaneous tissue.
o Meningitis and subdural abscess.
o Cerebellar abscess.
o Thrombosis of jugular bulb and jugular vein with involvement of IXth, Xth and XIth cranial nerves.
o Cavernous sinus thrombosis. There would be chemosis, proptosis, fixation of eyeball and papilloedema.
o Otitic hydrocephalus, when thrombus extends to sagittal sinus via confluens of sinuses.
➢ Management:
o I.v. antibacterial therapy.
o Mastoidectomy and exposure of sinus: Complete cortical or modified radical mastoidectomy.
©shovandas 29
o Ligation of internal jugular vein: Indicated when antibiotic and surgical treatment have failed.
o Anticoagulant therapy: Indicated when thrombosis is extending to cavernous sinus.
o Supportive treatment: Repeated blood transfusions to combat anaemia and improve patient’s resistance.

11. Marked mastoiditis. [2009]


➢ Introduction:
o Condition of slow destruction of mastoid air cells without the acute signs and symptoms of acute mastoiditis.
o Also called latent mastoiditis.
o Seen as a intratemporal complication of CSOM.
➢ Etiology:
o Due to inadequate antibiotic therapy in terms of dose, frequency and duration of administration.
o Most often due to use of oral penicillin in cases of acute otitis media when acute symptoms subside but smouldering
infection continues in the mastoid.
➢ Clinical features:
o No pain or mild pain behind the ear, no discharge, no fever and no mastoid swelling.
o Mostly seen in children.
o Mastoidectomy may show extensive destruction of the air cells with granulation tissue and dark gelatinous material
filling the mastoid.
o Erosion of the tegmen tympani and sinus plate with an extradural or perisinus abscess may be seen.
o Persistent hearing loss is seen.
o Tympanic membrane appears thick with loss of translucency.
o Slight tenderness may be elicited over the mastoid.
➢ Diagnosis:
o Tuning fork test: Conductive hearing loss.
o X-ray mastoid: Clouding of air cells with loss of cell outline.
➢ Complications: May progress for complications if undetected for long.
➢ Management:
o Cortical mastoidectomy with full doses of antibiotics.
o This is the treatment of choice.
o This may cause tympanic membrane to return to normal with improvement in hearing.

12. MacEwan's triangle. [2015]


➢ Mastoid antrum is marked externally on the surface of mastoid in temporal bone by MacEwen’s triangle.
➢ Also called suprameatal triangle.
➢ Boundary:
o Temporal line (a),
o Posterosuperior segment of bony external auditory canal (b),
o The line drawn as a tangent to the external canal (c).
➢ Anatomical importance:
o Mastoid antrum is a large, air-containing space in the upper part of mastoid.
o The lateral wall of antrum is formed by a plate of bone which is indicate by this triangle.
o On an average 1.5 cm thick in the adult.
o Lies deep to the cymba conchae.
➢ Surgical importance:
o Important landmark to locate the mastoid antrum in cortical mastoidectomy.
o Mastoid is drilled only in this particular area.

13. Carhart's notch. [1996, 2016]


➢ Introduction:
o The Carhart's notch is a depression in the bone-conduction audiogram of patients with clinical otosclerosis.
o Audiogram in air conduction shows a decrease in air conduction at all frequencies for these patients.
o But Carhart's notch is seen as a decrease in bone conduction.
o It is different at different frequencies but maximum at 2000 Hz (5 dB at 500 Hz, 10 dB at 1000 Hz, 15 dB at 2000 Hz
and 5 dB at 4000 Hz)
➢ Cause:
o After successful stapes surgery the Carhart's notch disappears when the conductive hearing improves. The fact shows
that Carhart's notch does not represent sensorineural reserve of a patient.
o It is hence considered to be due to stapes footplate fixation causing less vibration.
o The cause is also considered as the primary resonance frequency for ossicular chain bone conduction is 1700 Hz so
that maximum decrease is found in 2000 Hz.
➢ Homma’s study: Middle ear ossicle resonances for air & bone conduction are slightly different & show 2 modes of vibration.

©shovandas 30
o Mode 1:
▪ For air conduction.
▪ Peak at around 1200 Hz.
▪ Due to hinging movement of ossicles at the level of umbo of TM.
▪ Has no relation with Carhart's notch.
o Mode 2:
▪ For bone conduction.
▪ Peak at around 1700 Hz.
▪ Due to pivoting motion of malleus and incus complex.
▪ Decreased mobility of ossicles in this mode caused due to otosclero-
sis is considered to be the cause for Carhart’s notch due to loss of
additional vibration.
➢ Tondroff hypothesis: When skull is vibrated by bone conduction, sound is transferred to cochlea via three routes:
o By direct vibration of skull.
o By vibration of ossicular chain which is suspended within the skull.
o By transmission via external auditory canal.
▪ In otosclerosis, route 2 and 3 are affected showing Carhart's notch.
▪ But can be regained following successful stapes surgery.
▪ Hence bone conduction thresholds improve around 2 KHz frequency range.

14. Facial nerve anatomy. [2005].


➢ 7th cranial nerve.
➢ Roots: This nerve consists of two roots:
o A large medial motor root (facial nerve proper).
o A small lateral sensory root (nervous intermedius or nerve of
Wrisberg).
➢ Nucleus:
o Motor nucleus of VII nerve:
▪ Situated in the pontomedullary junction lateral to the supe-
rior end of olive of the medulla.
▪ Upper part of the nucleus innervating forehead muscles
receives fibres from both the cerebral hemispheres.
▪ Lower part of nucleus innervating lower face gets only the
crossed fibres from one hemisphere.
o Superior salivatory nucleus:
▪ Situated in the pontomedullary junction between motor
nucleus of facial nerve and NTS.
▪ Forms the secretomotor fibres innervating lacrimal, subma-
ndibular and sublingual glands and the smaller secretory
glands in the nasal mucosa and the palate.
o Nucleus tractus solitarius:
▪ Situated bellow the superior salivatory nucleus in medulla.
▪ Forms special visceral afferent fibers.
▪ Brings taste from the anterior 2/3rd of tongue via chorda
tympani.
➢ Course:
o Intracranial part:
▪ From pons to internal acoustic meatus.
▪ 15–17 mm.
▪ Motor fibres take origin from the nucleus of 7th nerve, hook round the nucleus of 6th nerve and are joined
by the nerve of Wrisberg.
▪ Facial nerve leaves the brainstem at pontomedullary junction.
▪ Travels through posterior cranial fossa.
▪ Enters the internal acoustic meatus.
o Intratemporal part:
▪ From internal acoustic meatus to stylomastoid foramen.
▪ Meatal segment:
✓ 8-10 mm.
✓ Within internal acoustic meatus.
▪ Labyrinthine segment:
✓ 4.0 mm.
✓ From fundus of meatus to geniculate ganglion.
©shovandas 31
✓ Takes a turn posteriorly forming a genu.
✓ The nerve in the labyrinthine segment has the narrowest diameter (0.61–0.68 mm) & the bony
canal in this segment is also the narrowest.
▪ Tympanic or horizontal segment:
✓ 11.0 mm.
✓ In relation with the structures of medial wall.
✓ From geniculate ganglion to just above the pyramidal eminence.
✓ Above the oval window and below the lateral semicircular canal.
✓ Takes a turn at junction of medial and posterior wall.
▪ Mastoid or vertical segment:
✓ 13.0 mm.
✓ In relation with the structures of posterior wall.
✓ From just behind the pyramid to stylomastoid foramen.
✓ Facial recess is present lateral to the facial nerve.
✓ Between tympanic and mastoid segments is the second genu of the nerve.

o Extracranial part: From stylomastoid foramen to the termination of its peripheral branches.
➢ Branches:
o Greater superficial petrosal nerve: Arises from geniculate ganglion and carries secretomotor fibres to lacrimal gland
and the glands of nasal mucosa and palate.
o Nerve to stapedius: Arises at the level of second genu and supplies the stapedius muscle.
o Chorda tympani: Arises from the middle of vertical segment, passes between the incus and neck of malleus, and
leaves the tympanic cavity through petrotympanic fissure.
o Communicating branch: Joins auricular branch of vagus and supplies the concha, retroauricular groove, posterior
meatus and the outer surface of tympanic membrane.
o Posterior auricular nerve: Supplies muscles of pinna, occipital belly of occipitofrontalis and communicates with
auricular branch of vagus.
o Muscular branches: To stylohyoid and posterior belly of digastric.
o Peripheral branches: After crossing the styloid process forms two divisions, an upper temporofacial and a lower
cervicofacial, which further divide into smaller branches. These are the temporal, zygomatic, buccal, mandibular and
cervical and together form pes anserinus (goosefoot). They supply all the muscles of facial expression.
➢ Blood supply:
o Anterior-inferior cerebellar artery: Supplies the nerve in cerebellopontine angle.
o Labyrinthine artery: Branch of anterior-inferior cerebellar artery, which supplies the nerve in internal auditory canal.
o Superficial petrosal artery: Branch of middle meningeal artery, supplies geniculate ganglion and the adjacent region.
o Stylomastoid artery: Branch of posterior auricular artery, supplies the mastoid and tympanic segment.
o All the arteries form an external plexus which lies in the epineurium and feeds a deeper intraneural internal plexus.

©shovandas 32
15. Facial nerve palsy of sudden onset. [2010]
➢ A lower motor neurone (LMN) palsy. Also called Bells palsy, idiopathic peripheral facial paralysis or paresis of acute onset.
➢ Aetiology:
o Viral infection:
▪ Most common aetiological factor.
▪ Pathogens: Herpes simplex (HSV-1 is most common), Herpes zoster, Epstein Barr virus.
▪ Other cranial nerves may also be involved which is thus considered a part of the polyneuropathy.
o Vascular ischaemia:
▪ Primary ischaemia: Due to cold or emotional stress.
▪ Secondary ischaemia: Result of primary ischaemia which causes increased capillary permeability leading to
exudation of fluid, oedema and compression of microcirculation of the nerve.
o Hereditary: The fallopian canal is narrow due to hereditary predisposition making the nerve susceptible to early
compression with slightest oedema. 10% of the cases of Bell palsy have a positive family history.
o Autoimmune disorder: T-lymphocyte changes have been observed.
o Risk factors:
▪ Diabetics due to angiopathy.
▪ Pregnant women due to retention of fluid.
➢ Clinical features:
o Sudden onset. o Epiphora: Tears flow down from the eye.
o Bells phenomenon: Patient is unable to close o Pain in ear may precede or accompany nerve paralysis.
eye. On attempting to close the eye, eyeball o Noise intolerance due to stapedial paralysis.
turns up and out. o Loss of taste due to involvement of chorda tympani.
o Dribbling of saliva from the angle of mouth. o Paralysis may be complete or incomplete.
o Asymmetrical face.
➢ Diagnosis:
o Diagnosis by exclusion: With the help of careful history, complete otological and head and neck examination, X-ray
studies, blood tests such as total count, peripheral smear, sedimentation rate, blood sugar and serology.
o Nerve excitability tests: To compare with the normal side to monitor nerve degeneration.
o Topodiagnosis: To establish the aetiology and site of surgical decompression of nerve, if that becomes necessary.
➢ Management:
o General:
▪ Reassurance.
▪ Relief of ear pain by analgesics.
▪ Care of the eye to protect against exposure keratitis.
▪ Physiotherapy.
o Medical:
▪ Steroids: To prevent incidence of synkinesis, crocodile tears and to shorten recovery time of facial paralysis.
©shovandas 33
▪ Acyclovir: For Herpes zoster infection.
▪ Other drugs: Vasodilators, vitamins, mast cell inhibitors and antihistaminics have not been found useful.
o Surgical:
▪ Vertical and tympanic segment nerve decompression.
▪ Total decompression including labyrinthine segment by postaural and middle fossa approach.

©shovandas 34
Group – d
01. Ototoxic drugs. [2018]
➢ Aminoglycosides: Delayed onset of complications. Causes hearing loss, tinnitus and/or giddiness.
o Vestibulotoxic:
▪ Drugs: Streptomycin, gentamicin and tobramycin.
▪ Action: Selectively destroy type I hair cells of the crista ampullaris but, large doses can also damage cochlea.
o Cochleotoxic:
▪ Drugs: Neomycin, kanamycin, amikacin, sisomycin and dihydrostreptomycin.
▪ Action: Selective destroy outer hair cells, starting at the basal coil and progressing onto the apex of cochlea.
➢ Diuretics: Sudden onset of complications. Causes reversible bilateral symmetrical hearing loss.
o Drugs: Furosemide, bumetanide and ethacrynic acid.
o Action: Forms oedema and cystic changes in the stria vascularis of the cochlear duct.
➢ Antimalarials: Causes reversible tinnitus and SNHL.
o Drugs: Quinine, chloroquine and hydroxychloroquine.
o Action: Causes vasoconstriction in the small vessels of the cochlea and stria vascularis.
➢ Cytotoxic drugs:
o Drugs: Nitrogen mustard, cisplatin and carboplatin.
o Action: They affect the outer hair cells of the cochlea.
➢ Analgesics: Causes reversible tinnitus and SNHL.
o Drugs: Salicylate, ibuprofen, indomethacin, phenylbutazone.
o Action: Interfere at enzymatic action of cochlea.
➢ Miscellaneous:
o Desferioxamine: Sudden or delayed onset of complications. Causes permanent high-frequency SNHL. Toxicity occurs
in the nerves. Children are affected more.
o Alcohol, tobacco and marijuana also cause damage to the inner ear.
o Isolated cases of deafness have been reported with erythromycin, ampicillin and chloramphenicol.
o Topical ear drops: Cause damage to the cochlea by absorption through oval and round windows.
▪ Chlorhexidine which was used in the preparation of ear canal before surgery, can cause deafness.
▪ Ototoxic potential is also present in ear drops containing polymyxin B, propylene glycol, antifungal agents.

02. Foreign body ear. [2012]


➢ Aetiology:
o Commonly seen in children.
o Carelessness during scratching ear with a matchstick or an overlooked cotton swab.
o Lack of hygiene.
➢ Foreign bodies:
o Non-living:
▪ Vegetable: Grain seeds like rice, wheat, maize etc. Tend to swell up with
time and get tightly impacted in the ear canal or may even suppurate.
▪ Non-vegetable: Piece of paper or sponge, slate pencil, piece of chalk or metallic ball bearings.
o Living:
▪ Flying or crawling insects: Mosquitoes, beetles, cockroach or an ant may enter the ear canal and cause
intense irritation and pain. No attempt should be made to catch them alive. It should be killed by instilling
oil (a household remedy), spirit or chloroform water.
▪ Maggots: Flies may be attracted to the foul-smelling ear discharge and lay eggs which hatch out into larvae
called maggots. There is severe pain with swelling round the ear & blood-stained watery discharge. Maggots
may be seen filling the ear canal. TM is ruptured in these cases.
➢ Methods of removing foreign body:
o Forceps removal:
▪ Indications: Soft and irregular foreign bodies like a piece of paper, swab or a piece of sponge can be removed
with fine crocodile forceps. Insects and maggots after killing are removed using forceps.
▪ Contraindications: Smooth and hard objects like steel ball bearing should not be grasped with forceps as
they tend to move inwards and may injure the TM.
o Syringing:
▪ Indications: Most of the seed grains and smooth objects can be removed with syringing.
▪ Contraindications: Can not be performed if TM is ruptured.
o Suction.
o Microscopic removal: In all impacted foreign bodies or in those where earlier attempts at extraction have been
made, it is preferable to use general anaesthetic and an operating microscope.

©shovandas 35
o Postaural approach: Used to remove foreign bodies impacted in deep meatus, medial to the isthmus or those which
have been pushed into the middle ear.
➢ Complications: Unskilled attempts at removal of foreign bodies may lacerate the meatal lining, damage the tympanic
membrane or the ear ossicles.

03. Otomycosis. [2008]


➢ It is a common case of inflammation of the external auditory canal due to fungal infection.
➢ Aetiology:
o Causative organisms: Aspergillus niger, A. fumigatus or Candida albicans.
o It is seen in hot and humid climate of tropical and subtropical countries.
o Secondary fungal growth is also seen in patients using topical antibiotics for treatment of otitis externa or middle ear
suppuration.
➢ Clinical features:
o intense itching. o Watery discharge with a musty odour.
o Discomfort or pain. o Ear blockage.
o The colour of the fungal mass can be white, brown or black.
o The fungal mass has wet newspaper appearance when mixed with discharge in case of Aspergillus niger infection.
o The fungal mass has cotton wool appearance in case of Candida infection.
➢ Diagnosis: Otoscopy:
o A. niger: Appears as black headed filamentous growth.
o A. fumigatus : Appears as pale blue or green coloured growth.
o Candida: Appears as white or creamy deposit.
o Meatal skin appears sodden, red and oedematous.
➢ Treatment:
o Ear toilet: To remove all discharge and epithelial debris.
o Antifungal treatment:
▪ Nystatin against Candida.
▪ Broad-spectrum antifungal agents like clotrimazole and povidone iodine.
▪ 2% salicylic acid in alcohol as it is a keratolytic which removes superficial layers of epidermis and fungal
mycelia growing into them.
▪ Antifungal treatment should be continued for a week even after apparent cure to avoid recurrences.
o Ear must be kept dry.
o Antibiotic/steroid preparation: Otomycosis is often associated with bacterial infection, so this is used to reduce
inflammation and oedema and thus permitting better penetration of antifungal agents.

04. Gradenigo syndrome. [2007]


➢ Also called Gradenigo-Lannois syndrome.
➢ Aetiopathogenesis:
o Bacterial infection in the middle ear cavity → spread to the highly vascular mastoid air cells → spread to the adjacent
petrous temporal bone, around which there are trigeminal ganglion and abducens nerve, which are separated from
the petrous bone only by dura mater → Inflammation in this region damages these nerves causing symptoms.
o Causative organisms: S. pneumoniae, H. influenzae, Pseudomonas and Staph. aureus.
o It has become very rare after the antibiotic era.
➢ Clinical features:
o Classical triad:
▪ Periorbital unilateral pain related to Vth nerve involvement.
▪ Diplopia due to VIth nerve palsy.
▪ Persistent otorrhea, associated with suppurative otitis media with petrositis.
o Other features:
▪ Fever. ▪ Dizziness.
▪ Deep unilateral facial pain. ▪ Nausea and vomiting.
▪ Headache. ▪ Confusion.
➢ Diagnosis: CT scan or MRI is done to see the extent of lesions in the petrous part.
➢ Management:
o High dose antibiotic treatment.
o Myringotomy to drain fluid.
o Surgeries like mastoidectomy or petrosectomy is done in complicated cases.

05. ABC test. [2016, 2003]


➢ Absolute bone conduction (ABC) test is a variant of Schwabach test.
➢ Principle: Bone conduction is a measure of cochlear function. In ABC test, patient’s bone conduction is compared with that
of the examiner presuming that the examiner has normal hearing.
©shovandas 36
➢ Procedure:
o The vibrating fork is placed over the mastoid process of the patient after occluding the external auditory canal by
pressing the tragus inwards to prevent ambient noise entering through AC route.
o As soon as the patient indicates that he is unable to hear the sound anymore, the fork is transferred to the mastoid
process of the examiner after occluding the external canal.
o This test is ideally performed using 256, 512 and 1024 Hz tuning forks. These frequencies are used because they fall
within the normal speech frequency range.
➢ Observation and inference:
o In cases of normal hearing the examiner must not be able to hear the fork.
o In cases of conductive deafness also, the examiner must not be able to hear the fork.
o In cases of SNHL the examiner will be able to hear the sound, then the test is interpreted as ABC reduced.

06. Weber test. [2006, 2004]


➢ The Weber test is a useful, quick, and simple screening test for the evaluation of hearing loss. The test can detect unilateral
conductive and sensorineural hearing loss.
➢ Principle: The inner ear is more sensitive to sound via air conduction than bone conduction. In the presence of a purely
unilateral conductive hearing loss, there is a relative improvement in the ability to hear a bone-conducted sound in the
affected ear due to masking effect and occlusion effect. In the presence of sensorineural hearing loss, the sound will be
perceived louder in the unaffected ear.
➢ Procedure:
o Place the vibrating tuning fork on the vertex (other common sites used are the midline of the forehead, bridge of
the nose, and chin), equidistant from both ears.
o These vibrations will be conducted through the skull and reach the cochlea.
o Ask the patient whether it is heard loudest in either one side or the midline.
➢ Observation and inference:
o In cases of normal hearing test does not show any lateralization of sound.
o In cases of unilateral conductive deafness test show lateralization of sound to the affected ear.
o In cases of unilateral SNHL test shows lateralization of sound to the unaffected ear.
o Lateralization of sound in weber test with a tuning fork of 512 Hz implies a conductive loss of 15–25 dB in ipsilateral
ear or a sensorineural loss in the contralateral ear.

07. Rinne test. [2000]


➢ In Rinne test air conduction of the ear is compared with its bone conduction.
➢ Principle: The inner ear is more sensitive to sound via air conduction than bone conduction (AC > BC).
➢ Procedure:
o A vibrating tuning fork is placed on the patient’s mastoid.
o When he stops hearing, it is brought beside the meatus.
o If he still hears, AC is more than BC.
o Alternatively, the patient is asked to compare the loudness of sound heard through air and bone conduction.
o Rinne test is called positive when AC is longer or louder than BC, otherwise test is negative.
o A prediction of air-bone gap can be made if tuning forks of 256, 512 and 1024 Hz are used.
➢ Observation and inference:
o In cases of normal hearing test is positive.
o In cases of conductive deafness test is negative.
o In cases of SNHL test is positive.
o Prediction of air-bone gap:
▪ A Rinne test equal or negative for 256Hz but positive for 512 Hz indicates air-bone gap of 20-30 dB.
▪ A Rinne test negative for 256 and 512Hz but positive for 1024 Hz indicates air-bone gap of 30-45 dB.
▪ A Rinne negative for all the three tuning forks of 256, 512 and 1024 Hz indicates air-bone gap of 45-60 dB.

08. Myringotomy. [2003, 2004, 2005, 2011]


➢ It is incision of TM.
➢ Aim:
o To drain suppurative or nonsuppurative effusion of the middle ear.
o To provide aeration in case of malfunctioning eustachian tube.
➢ Indications:
o Acute suppurative otitis media:
▪ Severe earache with bulging tympanic membrane.
▪ Incomplete resolution with opaque drum and persistent conductive deafness.
▪ Complications of acute otitis media, e.g. facial paralysis, labyrinthitis or meningitis with bulging TM.
o Serous otitis media.
o Aero-otitis media: To drain fluid and unlock the eustachian tube.
©shovandas 37
o Atelectatic ear: Grommet is often inserted for long-term aeration.
➢ Contraindications: Suspected intratympanic glomus tumour due to risk of profuse bleeding.
➢ Procedure:
o Ear canal is cleaned of wax and debris.
o General anaesthesia is given to adults with inflamed TM and in all children.
o Operation is performed under operating microscope using a sharp myringotome and a suction apparatus.
o Incision:
▪ In acute suppurative otitis media: Circumferential incision in the posteroinferior quadrant of TM, midway
between handle of malleus and tympanic annulus, avoiding injury to incudostapedial joint.
▪ In serous otitis media: Small radial incision in the posteroinferior or anteroinferior quadrant.
➢ Postoperative care:
o Dry mopping of ear discharge.
o No water in the ear canal for at least 1 week.
➢ Complications:
o Injury to incudostapedial joint or stapes.
o Injury to jugular bulb with profuse bleeding, if jugular bulb is high and floor of the middle ear dehiscent.
o Middle ear infection.

09. Myringoplasty. [2010]


➢ Closure of perforation of pars tensa of the tympanic membrane is called myringoplasty.
➢ Aim and indications:
o Restoring hearing loss and in some cases of tinnitus.
o Checking repeated infection from external auditory canal and eustachian tube.
o Checking aeroallergens reaching the exposed middle ear mucosa, leading to persistent ear discharge.
➢ Contraindications:
o Active discharge from the middle ear. o When the other ear is dead or not suitable for
o Nasal allergy. hearing aid rehabilitation.
o Otitis externa. o Children below 3 years.
o Ingrowth of squamous epithelium into the middle ear.
➢ Procedure:
o Position: Supine with face turned to one side; the ear to be operated is up.
o Anaesthesia: Local (more preferable) or general.
o Underlay technique:
▪ Graft is harvested from temporalis fascia or perichondrium from tragus →
▪ Incision is made along the edge of perforation →
▪ Ring of epithelium along with a strip of mucosal layer from the inner side
of perforation is removed →
▪ Middle ear is inspected to see the integrity and mobility of the ossicular
chain and to ensure no squamous epithelium growth →
▪ Graft is placed as such that its edges extend under the margins of perfora-
tion all round and a small part also extends over the posterior canal wall →
▪ Tympanomeatal flap is replaced.
o Overlay technique:
▪ Graft is harvested from temporalis fascia →
▪ Incision in meatus is made →
▪ Meatal skin is raised along with all epithelium from the outer surface of TM
remnant and preserved to be used later →
▪ Graft is placed on the outer surface of TM →
▪ Meatal skin removed earlier, is replaced covering the periphery of the graft

▪ Ear canal is packed with gelfoam and small anti biotic pack →
▪ Closure of endaural or postaural incision → Mastoid dressing.
➢ Complications:
o Overlay technique: o Underlay technique:
▪ Narrow middle ear. ▪ Blunting of the anterior sulcus.
▪ Adherence of graft to promontory. ▪ Epithelial pearl formation.
▪ Anterior perforation. ▪ Lateralization of graft.

10. Cortical mastoidectomy. [2007]


➢ Introduction:
o It is known as simple or complete mastoidectomy or Schwartz operation.
o It is complete exenteration of all accessible mastoid air cells and converting them into a single cavity.
©shovandas 38
o Posterior meatal wall and middle ear structures are left intact.
➢ Indications:
o Acute coalescent mastoiditis.
o Incompletely resolved acute otitis media with reservoir sign.
o Masked mastoiditis.
o As an initial step of other operations:
▪ Endolymphatic sac surgery.
▪ Decompression of facial nerve.
▪ Translabyrinthine or retrolabyrinthine procedures for acoustic neuroma.

➢ Procedure:
o Position: Supine with face turned to one side; the ear to be operated is up.
o Anaesthesia: General.
o Technique:
▪ A curved postaural incision about 1 cm behind but parallel to the retroauricular sulcus, starting at the
highest attachment of pinna to the mastoid tip is made →
▪ Periosteum is incised at line of 1st incision to expose lateral surface of mastoid and MacEwen’s triangle →
▪ Mastoid cortex is removed with burr, or gouge and hammer to expose mastoid antrum →
▪ All accessible mastoid air cells are removed leaving behind the bony plate of tegmen tympani above, sinus
plate behind and posterior meatal wall in front →
▪ Lateral wall of the mastoid tip, zygomatic cells in the root of zygoma, retrosinus cells between sinus plate
and cortex, behind the sinus are also removed →
▪ Cavity is irrigated with saline to remove bone dust and the wound is closed in 2 layers → Mastoid dressing.
➢ Complications:
o Injury to facial nerve. o Injury to dura of middle cranial fossa.
o Dislocation of incus. o Postoperative wound infection and/or wound
o Injury to horizontal semicircular canal. breakdown.

11. Otosclerosis. [2004]


➢ Otosclerosis, more aptly called otospongiosis, is a primary disease of the bony labyrinth where, one or more foci of irregularly
laid spongy bone replace part of normally dense enchondral layer of bony otic capsule.
➢ Epidemiology:
o Race: More in whites. More in Indian than Chinese.
o Sex: Female : male = 2 : 1.
o Age: Most common in 3rd decade.
➢ Aetiology:
o Location: Fissula ante fenestram, a cartilage rest in bony labyrinth, lying in front of the oval window.
o Inheritance: Autosomal dominant.
o The disease may be associated with osteogenesis imperfecta with history of multiple fractures. The triad of symp-
toms of osteogenesis imperfecta, otosclerosis and blue sclera is called van der Hoeve syndrome.
o May be initiated or made worse by pregnancy.
o Associated with measles virus infection.
➢ Types:
o Stapedial otosclerosis:
▪ Causes stapes fixation and conductive deafness.
©shovandas 39
▪ Lesion:
✓ Anterior focus: At fissula ante fenestram.
✓ Posterior focus: Behind the oval window.
✓ Circumferential: Around the margin of the stapes footplate.
✓ Biscuit type: In the footplate but annular ligament being free.
✓ Obliterative: Complete obliteration of the oval window niche.
o Cochlear otosclerosis:
▪ Causes SNHL.
▪ Lesion: Region of round window or other areas in the otic capsule.
o Histologic otosclerosis: Remains asymptomatic and causes neither conductive nor SNHL.
➢ Clinical features:
o Symptoms:
▪ Hearing loss. ▪ Vertigo.
▪ Tinnitus. ▪ Monotonous, well-modulated soft speech.
▪ Paracusis Willisii: Patient hears better in noisy than in quiet surroundings. This is because a normal person
will raise his voice in noisy surroundings.
o Signs:
▪ Rinne: -ve.
▪ ABC: Normal.
▪ Gelles: -ve.
▪ Weber: Lateralized to worst ear.
▪ Schwartze: Lengthen.
▪ Pure tone audiometry:
✓ Loss of air conduction, more for lower frequencies.
✓ No change in bone conduction.
✓ Carhart's notch is seen.
➢ Treatment: Stapedectomy/stapedotomy with a placement of prosthesis is the treatment of choice. Here the fixed otosclerotic
stapes is removed and a prosthesis inserted between the incus and oval window.

©shovandas 40
Second part

Nose

©shovandas 41
©shovandas 42
Group – a
01. Describe anatomy of nasal septum. Draw and label a diagram of it. [2019, 2012]
➢ Nasal septum forms the medial wall of nasal cavity or nostril. It is an osteo cartilaginous structure.
➢ Parts: Nasal septum consists of three parts:
o Columellar septum: It is formed of columella containing the medial crura of alar cartilages united together by fibrous
tissue and covered on either side by skin.
o Membranous septum: It consists of double layer of skin with no bony or cartilaginous support. It lies between the
columella and the caudal border of septal cartilage. Both columellar and membranous parts are freely movable from
side to side.
o Septum proper: It consists of osteocartilaginous framework, covered with nasal mucous membrane. It has two parts:
▪ Bony part:
✓ Perpendicular plate of ethmoid: Forms the posterosuperior part of the septum.
✓ Vomer: Forms the posteroinferior part of the septum.
✓ Other bones with minor contributions at periphery:
• Crest of nasal bones.
• Nasal spine of frontal bone.
• Rostrum of sphenoid.
• Crest of palatine bones and the crest maxilla.
• Anterior nasal spine of maxilla.
▪ Cartilaginous part: Entirely formed by septal cartilage, which is a quadrilateral cartilage wedged between
the ethmoid and vomer anteriorly. Septal cartilage not only forms a partition between the right and left
nasal cavities but also provides support to the tip and dorsum of cartilaginous part of nose.

➢ Blood supply:
o Arterial supply:
▪ Internal carotid system:
✓ Anterior ethmoidal artery: Branch of ophthalmic artery.
✓ Posterior ethmoidal artery: Branch of ophthalmic artery.
▪ External carotid artery:
✓ Sphenopalatine artery: Branch of maxillary artery, gives nasopalatine and posterior medial nasal
branches.
✓ Septal branch of greater palatine artery: Branch of maxillary artery.
✓ Septal branch of superior labial artery: Branch of facial artery.
o Venous drainage: The veins draining the nasal septum form plexus beneath the mucosa and in general accompany
the arteries. The veins drain into facial vein, pterygoid venous plexus and pharyngeal venous plexus.
➢ Lymphatic drainage:
o Anterior half: Submandibular lymph nodes.
o Posterior half: Retropharyngeal lymph nodes.
➢ Nerve supply:
o Olfactory nerve: Supply the upper part (one-third) just below the cribriform plate.
o Internal nasal branch of the anterior ethmoidal nerve, a branch from nasociliary: Supplies the anterosuperior part.
o Nasopalatine nerve, a branch of pterygopalatine ganglion: Supplies the posteroinferior part.
©shovandas 43
o Medial posterior superior nasal branches of pterygopalatine ganglion: Supply the posterosuperior part.
o Nasal branch of greater palatine nerve: Supplies the posterior part.
o Anterior superior alveolar nerve, a branch of maxillary nerve: Supplies the anteroinferior part.

02. Describe constituents of nasal septum and vascular supply with diagram. Surgical importance of little’s area. [2016](3+5+2)
➢ Constituents of nasal septum: Group A, question no. 01.
➢ Vascular supply: Group A, question no. 01.
➢ Surgical importance of little's area:
o Situated in the anterior inferior part of nasal septum, just above the vestibule.
o Four arteries; anterior ethmoidal, septal branch of superior labial, septal branch of sphenopalatine and the greater
palatine anastomose here to form a vascular plexus called ‘Kiesselbach's plexus’.
o This area is exposed to the drying effect of inspiratory current and to finger nail trauma, and is the most common
site for epistaxis in children and young adults.
o In surgical management of epistaxis, ligation of sphenopalatine artery and anterior ethmoidal artery is done.

03. Describe lateral wall of nose. Mention its surgical importance. [2015, 2006, 2005, 2004](7+3)
➢ Lateral wall of nose: The lateral wall is an osteo cartilaginous structure.
o Components:
▪ Bony components:
✓ Nasal.
✓ Frontal process of maxilla.
✓ Lacrimal.
✓ Conchae and labyrinth of ethmoid
✓ Inferior nasal concha.
✓ Perpendicular plate of palatine.
✓ Medial pterygoid plate of sphenoid.
▪ Cartilaginous components:
✓ lateral nasal cartilage (upper nasal cartilage).
©shovandas 44
✓ Major alar cartilage (lower nasal cartilage).
✓ Three to four tiny cartilages of the alae (minor alar cartilages).
o Parts: The lateral wall is divided into the following three areas:
▪ Anterior part: Presents a small depressed area, the vestibule. It is lined by the skin containing vibrissae
(short, stiff curved hair).
▪ Middle part: Known as atrium of middle meatus. It is limited above by a faint ridge of mucous membrane,
the agger nasi. The curved mucocutaneous junction between the atrium and vestibule is known as limen
nasi.
▪ Posterior part: Presents three scroll-like projections, the conchae or turbinates. The spaces separating the
conchae are called meatuses. The conchae and meatuses form the main features of the lateral wall.

➢ Conchae:
o Also called turbinates.
o These are the curved bony projections directed downwards and medially. Below and lateral to each concha is a
corresponding meatus.
o From above downwards the conchae are superior, middle, and inferior nasal conchae. Sometimes a 4th concha, the
concha suprema is also present.
▪ Superior nasal conchae: Smallest. Made by projections from the medial surface of the ethmoidal labyrinth.
▪ Middle nasal conchae: Made by projections from the medial surface of the ethmoidal labyrinth.
▪ Inferior nasal concha: Largest and is an independent bone.
➢ Meatuses:
o Meatuses are the passages (recesses) beneath the overhanging conchae.
o They are visualized once conchae are removed.
▪ Inferior meatus: Largest and lies underneath the inferior nasal concha.
▪ Middle meatus: Lies underneath the middle concha. It presents following features:
✓ Ethmoidal bulla (bulla ethmoidalis): A round elevation produced by the underlying middle
ethmoidal sinuses.
✓ Hiatus semilunaris: A deep semicircular sulcus below the bulla ethmoidalis.
✓ Infundibulum: A short passage at the anterior end of middle meatus.
▪ Superior meatus: Smallest and lies below the superior concha. A triangular depression, above and behind
the superior concha is known as the sphenoethmoidal recess.
➢ Openings:
o Sphenoethmoidal recess: Opening of sphenoidal air sinus.
o Superior meatus: Opening of the posterior ethmoidal air sinus.
o Middle meatus:
▪ On bulla: Opening of the middle ethmoidal air sinus.
▪ In hiatus semilunaris:
✓ Anterior part: Opening of the frontal air sinus.
✓ Middle part: Opening of the anterior ethmoidal air sinus.
✓ Posterior part: Opening of maxillary air sinus.
o Inferior meatus: Opening of the nasolacrimal duct (in the anterior part of meatus).
➢ Surgical importance:

©shovandas 45
04. Describe anatomy of maxillary antrum and function of nose. [2014](7+3)
➢ Maxillary antrum (antrum of Highmore):
o One of the anterior group paranasal air sinuses. It is the largest paranasal air sinus and is present in body of maxilla.
o Measurements:
▪ Volume: 15 mL.
▪ Vertical diameter: 3.5 cm.
▪ Transverse diameter: 2.5 cm.
▪ Anteroposterior diameter: 3.25 cm.
o Shape: It is pyramidal in shape with the base directed medially towards the lateral wall of the nose and its apex
laterally towards the zygomatic bone.
o Relations:
▪ Roof: It is formed by the floor of the orbit. The infraorbital nerve and artery traverse the roof in a bony
canal.
▪ Floor: Very small, it is formed by the alveolar process of maxilla and lies about 1.25 cm below the floor of
the nasal cavity. The level of the floor corresponds to the level of the ala of nose. Normally the roots of the
first and second molar teeth project into the floor producing elevations but sometimes roots of the first and
second premolars, third molar and rarely even that of canine may project into the floor. Sometimes roots
of teeth are separated from the sinus only by a thin layer of mucous lining.

▪ Base: It is formed by the lateral wall of the nose. It possesses the opening or ostium of the sinus in its upper
part, i.e. close to the roof, a disadvantageous position for natural drainage. In the disarticulated skull, the
base of maxillary sinus (medial surface of the body of maxilla) presents a large opening, the maxillary hiatus,
which is reduced in size by the following bones:
✓ From above: Uncinate process of ethmoid.
✓ From in front: Descending process of lacrimal.
✓ From below: Ethmoidal process of inferior nasal concha.
✓ From behind: Perpendicular plate of palatine.
▪ Apex: It extends into the zygomatic process of maxilla.

©shovandas 46
▪ Anterior wall: It is formed by the anterior surface of the body of maxilla and is related to infraorbital plexus
of nerves. Within this wall runs the anterior superior alveolar nerve in a curved bony canal called canalis
sinuosus.
▪ Posterior wall: It is formed by the infratemporal surface of the maxilla, separating the sinus from the
infratemporal and pterygopalatine fossae. It is pierced by the posterior superior alveolar nerves and vessels.
o Opening: Maxillary sinuses open in the hiatus semilunaris of middle meatus near the roof of the sinus.
o Arterial supply: It is by the anterior, middle and posterior superior alveolar arteries from maxillary and infraorbital
arteries.
o Lymphatic Drainage: The sinus drains into submandibular lymph nodes.
o Nerve Supply: Maxillary sinuses are supplied by the anterior, middle and posterior superior alveolar nerves from the
maxillary and infraorbital nerves.
➢ Function of nose:
o Respiration: Nose is the natural pathway for breathing.
o Air-conditioning of inspired air: Nose is called ‘air-conditioner’ for lungs.
▪ Filtration and purification: By nasal vibrissae and mucus.
▪ Temperature control: By highly vascular mucous membrane through radiator mechanism.
▪ Humidification: This function goes on simultaneously with the temperature control of inspired air.
o Protection of lower airway:
▪ Mucociliary mechanism: The inspired bacteria, viruses and dust particles are entrapped on the viscous
mucous blanket and then carried to the nasopharynx by conveyor belt mechanism to be swallowed.
▪ Enzymes: Nasal secretions contain an enzyme called muramidase (lysozyme) which kills bacteria and virus.
▪ Immunoglobulins: IgA, IgE and interferon in nasal secretions provide immunity against URTIs.
▪ Sneezing: It is a protective reflex. Foreign particles which irritate nasal mucosa are expelled by sneezing.
o Vocal resonance: Nose forms a resonating chamber for certain consonants (e.g. M/N/NG) in speech.
o Nasal reflex functions:
▪ Smell of a palatable food cause reflex secretion of saliva and gastric juice.
▪ Nasal function is closely related to pulmonary function through nasobronchial and nasopulmonary reflexes.
o Olfaction: Sense of smell is important for pleasure and enjoying the taste of food. When nose is blocked, food tastes
bland and unpalatable.

05. Describe the anatomy of nasal septum and function of nose. [2003]
➢ Anatomy of nasal septum: Group A, question no. 01.
➢ Function of nose: Group A, question no. 04.

©shovandas 47
©shovandas 48
Group – b
01. How will you investigate a case of 14 year old with recurrent moderate to severe epistasis? Write differential diagnosis &
management. [2020, 2017, 2011, 2004, 1996](3+7)
➢ Differential diagnosis:
o Local causes:
▪ Nose:
✓ Trauma: Finger nail trauma (most common cause in children and young adults), injuries of nose,
intranasal surgery, fractures of middle 3rd of face and base of skull, hard-blowing of nose, violent
sneeze.
✓ Infections:
• Acute: Viral rhinitis, nasal diphtheria, acute sinusitis.
• Chronic: All crust-forming diseases, e.g. atrophic rhinitis, rhinitis sicca, TB, syphilis septal
perforation, granulomatous lesion of the nose, e.g. rhinosporidiosis.
✓ Foreign bodies:
• Non-living: Any neglected foreign body, rhinolith.
• Living: Maggots, leeches.
✓ Neoplasms of nose and paranasal sinuses:
• Benign: Haemangioma, papilloma.
• Malignant: Carcinoma or sarcoma (angiofibroma is the most common cause of recurrent
epistaxis in young male).
✓ Atmospheric changes: High altitudes, sudden decompression (Caisson disease).
✓ Deviated nasal septum.
▪ Nasopharynx:
✓ Adenoiditis.
✓ Juvenile angiofibroma.
✓ Malignant tumours.
o General causes:
▪ Cardiovascular system: HTN, arteriosclerosis, mitral stenosis, pregnancy (hypertension and hormonal).
▪ Disorders of blood and blood vessels: Aplastic anaemia, leukaemia, thrombocytopenic, vascular purpura,
haemophilia, Christmas disease, scurvy, vitamin K deficiency and hereditary haemorrhagic telangectasia.
▪ Liver disease: Hepatic cirrhosis (deficiency of factor II, VII, IX and X).
▪ Kidney disease: Chronic nephritis.
▪ Drugs: Excessive use of salicylates and other analgesics, anticoagulant therapy.
▪ Mediastinal compression: Tumours of mediastinum (raised venous pressure in the nose).
▪ Acute general infection: Influenza, measles, chickenpox, whooping cough, rheumatic fever, infectious
mononucleosis, typhoid, pneumonia, malaria and dengue fever.
▪ Vicarious menstruation: Epistaxis occurring at the time of menstruation.
o Idiopathic: All over most common cause, where the cause of epistaxis is not clear.
➢ Management:
o Trotter’s/Hippocratic method:
▪ Indication: Done when the bleeding is from Little's area.
▪ Procedure: The patient is made to sit, leaning a little forward and pinching the nose for 5 minutes.
o Cauterization:
▪ Indication: Done in anterior epistaxis when bleeding point has been located.
▪ Procedure: The area is topically anaesthetized and the bleeding point cauterized with a bead of silver nitrate
or coagulated with electrocautery.

o Anterior nasal packing:


▪ Indication: Done in anterior epistaxis when bleeding is profuse and/or site of bleeding is difficult to localize.

©shovandas 49
▪ Procedure:
✓ A ribbon gauze is soaked with liquid paraffin →
✓ Few centimetres of gauze are folded upon itself and inserted along the floor →
✓ Then the whole nasal cavity is packed tightly by layering the gauze from floor to the roof and from
before backwards.
▪ Packing can also be done in vertical layers from back to the front. One or both cavities may need to be
packed. Pack can be removed after 24 h, if bleeding has stopped.
o Posterior nasal packing:
▪ Indication: It is required for patients bleeding posteriorly into the throat.
▪ Procedure:
✓ Postnasal pack is prepared by tying three silk ties to a piece of gauze rolled into shape of a cone →
✓ A rubber catheter is passed through the nose and its end brought out from the mouth →
✓ Ends of the silk threads are tied to it and catheter withdrawn from nose →
✓ Pack, which follows the silk thread, is now guided into the nasopharynx with the index finger →
✓ Anterior nasal cavity is now packed and silk threads tied over a dental roll →
✓ The third silk thread is cut short and allowed to hang in the oropharynx for easy removal.
o Endoscopic cauterization:
▪ Indication: Done for localized bleeding. Can not be done when profuse bleeding does not permit localization
of the bleeding point.
▪ Procedure:
✓ Topical or general anaesthesia is given →
✓ Bleeding point is localized with a rigid endoscope →
✓ Cauterized with a malleable unipolar suction cautery or a bipolar cautery.
o Elevation of mucoperichondrial flap and submucous resection (SMR) operation:
▪ Indication: Done in case of persistent or recurrent bleeds from the septum.
▪ Procedure: Elevation of mucoperichondrial flap and repositioning it back causes fibrosis and constrict blood
vessels. SMR operation can be done to achieve the same result or remove any septal spur which is some-
times the cause of epistaxis.
o Ligation of vessels:
▪ External carotid: When bleeding is from the external carotid system and the conservative measures have
failed, ligation of external carotid artery above the origin of superior thyroid artery should be done.
▪ Maxillary artery: Ligation of this artery is done in uncontrollable posterior epistaxis. Approach is via
Caldwell–Luc operation. Posterior wall of maxillary sinus is removed and the maxillary artery or its branches
are blocked by applying clips.
▪ Ethmoidal arteries: In anterosuperior bleeding above the middle turbinate, not controlled by packing,
anterior and posterior ethmoidal arteries, which supply this area, can be ligated.

02. A 17 years old male patient presents with history of recurrent profuse epistaxis for last one year. Write down the probable
causes, investigation & management. [2010](3+2+5)
➢ Probable causes: Group B, question no. 01.
➢ Investigation:
o Mode of onset: Spontaneous or finger nail trauma.
o Duration and frequency of bleeding.
o Amount of blood loss.
o Side of nose from where bleeding is occurring.
o Whether bleeding is of anterior or posterior type.
o Any known bleeding tendency in the patient or family.
o History of known medical ailment: Hypertension, leukaemia, mitral valve disease, cirrhosis and nephritis.
o History of drug intake: Analgesics, anticoagulants etc.
o Endoscopy: Rigid endoscopy to localize bleeding in posterior type epistaxis.
➢ Management: Group B, question no. 01.

03. Write down the aetiology, clinical features & management of acute maxillary sinusitis. [2019, 2003, 1999](3+4+3)
➢ Aetiology:
o Most commonly, it is viral rhinitis which spreads to involve the sinus mucosa. This is followed by bacterial invasion.
o Diving and swimming in contaminated water.
o Dental infections:
▪ Periapical dental abscess may burst into the sinus.
▪ The root of a tooth, during extraction, may be pushed into the sinus.
▪ In case of oroantral fistula, following tooth extraction, bacteria from oral cavity enter the maxillary sinus.
o Trauma to the sinus: Compound fractures, penetrating injuries or gunshot wounds may be followed by sinusitis.

©shovandas 50
➢ Clinical features: Clinical features depend on severity of inflammatory process and efficiency of ostium to drain the exudates.
Closed ostium sinusitis is of greater severity and leads more often to complications.
o Constitutional symptoms: Fever, general malaise and body ache due to toxaemia.
o Headache: Usually confined to forehead and may thus be confused with frontal sinusitis.
o Pain: Typically, it is situated over the upper jaw, but may be referred to the gums or teeth. Pain is aggravated by
stooping, coughing or chewing. Occasionally, pain is referred to the ipsilateral supraorbital region and thus may
simulate frontal sinus infection.
o Tenderness: Pressure or tapping over the anterior wall of antrum produces pain.
o Redness and oedema of cheek: Commonly seen in children. The lower eyelid may become puffy.
o Nasal discharge: Anterior rhinoscopy/nasal endoscopy shows pus or mucopus in the middle meatus. Mucosa of the
middle meatus and turbinate may appear red and swollen.
o Postnasal discharge: Pus may be seen on the upper soft palate on posterior rhinoscopy or nasal endoscopy.
➢ Treatment:
o Medical:
▪ Antimicrobial drugs:
✓ Ampicillin and amoxicillin are mainly used.
✓ Erythromycin or doxycycline or cotrimoxazole to penicillin sensitive cases.
✓ β-lactamase-producing strains of H. influenzae and M. catarrhalis may necessitate the use of
amoxicillin/ clavulanic acid or cefuroxime axetil.
✓ Sparfloxacin is also effective, and has the advantage of single daily dose.
▪ Nasal decongestant drops: 1% ephedrine or 0.1% xylo- or oxymetazoline are used as nasal drops or sprays
to decongest sinus ostium and encourage drainage.
▪ Steam inhalation: Steam alone or with menthol or Tr. Benzoin Co. provide symptomatic relief and increases
sinus drainage. Inhalation should be given 15-20 min after nasal decongestion for better penetration.
▪ Analgesics: Paracetamol or other analgesic for relief of pain and headache.
▪ Hot fomentation: Local heat to affected sinus is often soothing and helps in the resolution of inflammation.
o Surgical: Antral lavage: It is done only when medical treatment has failed and that too only under cover of antibiotics.

04. Young adult presented with left sided nasal polyp. Write down differential diagnosis and management of the antrochoanal
polyp. [2012](3+7)
➢ Differential diagnosis:
o Blob of mucus: A blob of mucus often looks like a polypus but it would disappear on blowing the nose.
o Hypertrophied middle turbinate: It is differentiated by its pink appearance and hard feel of bone on probe testing.
o Angiofibroma: It has history of profuse recurrent epistaxis. It is firm in consistency and easily bleeds on probing.
o Mucocele: Contains mucous and desquamated epithelium.
o Haemangioma: Rare benign vascular lesion in the nasal cavity and paranasal sinuses. Most of them arise from the
anterior nasal septum and the nasal turbinates.
o Malignant tumours of the nasopharynx: These may be differentiated by their fleshy pink appearance, friable nature
and their tendency to bleed.
▪ Lymphoma.
▪ Rhabdomyosarcoma.
▪ Lymphoepithelioma.
▪ Esthesioneuroblastoma.
▪ Chordoma.
➢ Management: The treatment of antrochoanal polyp is always surgical.
o Simple polypectomy:
▪ Previously preferred methods for surgically treating antrochoanal polyps.
▪ Simple polypectomy carries a high recurrence rate.
▪ The antral part of the polyp should be removed to avoid post-operative recurrence.
o Caldwell-Luc procedure:
▪ Previously preferred methods for surgically treating antrochoanal polyps.
▪ The Caldwell-Luc procedure offers good exposure for complete removal of the antral part of the polyp.
▪ Complications:
✓ Cheek anaesthesia.
✓ Cheek swelling.
✓ Injury of the infraorbital nerve.
✓ It carries the risk of damaging the growing teeth and the growth centres of the maxilla in children.
o Functional endoscopic sinus surgery (FESS):
▪ FESS has recently been shown to be the safest and most effective method for treating antrochoanal polyps.
▪ Procedure:
✓ Resection of the nasal part of the polyp and the cystic antral part.
✓ Attachment to the maxillary wall via the middle meatus.
©shovandas 51
✓ The lower part of the uncinate process is removed.
✓ The maxillary ostium is widened.

05. Write a note on deviated nasal septum and its effect on the ear. [2010]
➢ Aetiology:
o Trauma: A lateral blow on the nose or trauma during difficult labour when nose is pressed.
o Developmental error: Unequal growth between the palate and the base of skull , adenoid hypertrophy, cleft lip and
palate and dental abnormalities can give rise to DNS.
o Racial factors: Caucasians are affected more than black Americans.
o Hereditary factors: Several members of the same family may have deviated nasal septum.
➢ Types:
o Anterior dislocation: Septal cartilage may be dislocated into one of the nasal chamber. This is better appreciated by
looking at the base of nose when patient’s head is tilted backwards.
o C-shaped deformity: Septum is deviated in a simple curve to one side. Nasal chamber on the concave side of the
nasal septum will be wider and may show compensatory hypertrophy of turbinates.
o S-shaped deformity: Septum is deviated either in vertical or anteroposterior plane. Such a deformity may cause
bilateral nasal obstruction.
o Spurs: A spur is a shelf-like projection often found at the junction of bone and cartilage. A spur may press on the
lateral wall and gives rise to headache. It may also predispose to repeated epistaxis from the vessels stretched on its
convex surface.
o Thickening: It may be due to organized haematoma or overriding of dislocated septal fragments.
➢ Clinical features:
o Nasal obstruction: Site of obstruction:
▪ Vestibular: Caudal septal dislocation, synechiae or stenosis.
▪ At the nasal valve: Synechiae, usually postrhinoplasty.
▪ Attic: Along the upper part of nasal septum due to high septal deviation.
▪ Turbinal: Hypertrophic turbinates or concha bullosa.
▪ Choanal: Choanal atresia or a choanal polyp.
o Headache: Deviated septum, especially a spur, may press on the lateral wall of nose giving rise to pressure headache.
o Sinusitis: Deviated septum may obstruct sinus ostia resulting in poor ventilation of the sinuses. Therefore, it forms
an important cause to predispose or perpetuate sinus infections.
o Epistaxis: Mucosa over the deviated part of septum is exposed to the drying effects of air currents leading to
formation of crusts, which when removed causes bleeding. Bleeding may also occur from vessels over a septal spur.
o Anosmia: Failure of the inspired air to reach the olfactory region may result in total or partial loss of sense of smell.
o External deformity: Septal deformities may be associated with deviation of the cartilaginous or both the bony and
cartilaginous dorsum of nose, deformities of the nasal tip or columella.
o Middle ear infection: DNS also predisposes to middle ear infection.
➢ Treatment:
o Minor degrees of septal deviation with no symptoms are commonly seen in patients and require no treatment.
o Surgery is only indicated, when deviated septum produces mechanical nasal obstruction or the symptoms.
▪ Submucosal resection (SMR) operation.
▪ Septoplasty.

©shovandas 52
Group – c
01. Antrochoanal polyp. [2008, 1993]
➢ Also known as Killian's polyp.
➢ This polyp arises from the mucosa of maxillary antrum near its accessory ostium, comes out of it and grows in the choana and
nasal cavity. Thus it has three parts:
o Antral part: Thin stalk.
o Choanal part: Round and globular.
o Nasal part: Flat from side to side.
➢ Aetiology: Exact cause is unknown. Nasal allergy with bacterial maxillary sinus infection near the ostium is incriminated.
➢ Clinical features:
o Symptoms:
▪ Most commonly seen in children.
▪ Single unilateral polyp with nasal obstruction.
▪ Obstruction may be bilateral when polyp grows into nasopharynx and starts obstructing opposite choana.
▪ Thick and dull voice due to hyponasality.
▪ Nasal discharge, mostly mucoid.
o Signs:
▪ When large, a smooth greyish mass covered with nasal discharge may be seen.
▪ Soft and can be moved up and down with a probe.
▪ A large polyp may protrude from the nostril and show a pink congested look on its exposed part.
➢ Diagnosis by posterior rhinoscopy:
o As the antrochoanal polyp grows posteriorly, it may be missed on anterior rhinoscopy.
o Posterior rhinoscopy reveal a globular mass filling the choana or the nasopharynx.
o A large polyp may hang down behind the soft palate and present in the oropharynx.
➢ Differential diagnosis and management: Group B, question no. 04.

02. Antral puncture. [2012, 2007]


➢ Also known as proof puncture or antral lavage.
➢ Indications: Chronic and subacute maxillary sinusitis.
o Diagnostic purpose: To collect specimen of antral content
for cytological examination to exclude early malignancy.
o Treatment purpose:
▪ For washing out the pus.
▪ To collect the specimen of the antral contents for culture and sensitivity.
➢ Contraindications: Acute maxillary sinusitis for fear of osteomyelitis.
➢ Procedure:
o Anaesthesia: General anaesthesia for children and local anaesthesia for adults are given.
o Position: The patient is kept in tonsillectomy position in case of general anaesthesia or in sitting position in case of
local anaesthesia.
o Puncture: The lateral wall of inferior meatus is punctured with a ‘crack’ with Lichtwitz trocar and cannula at a point
1.5-2. cm from anterior end of inferior turbinate and near the attachment of concha with lateral wall.
o Now remove the trocar and advance the cannula till it reaches the opposite antral wall and then withdraw a little.
o Syringing: The antrum can now be irrigated with normal saline at 37°C with a 20 mL or Higginson’s syringe. Syringing
is continued till return is clear.
o Pack: After the puncture is over, cannula is removed and a pack kept in the inferior meatus to control bleeding.
➢ Post operative care:
o Pack is removed after about an hour.
o Antibiotics should be given for 5-6 days in cases of suppuration.
o Nasal decongestant drops should be used to improve patency of the ostium.
o Analgesics may be required for headache or postoperative pain.
➢ Complications:
o Swelling of cheek: Due to faulty technique.
o Orbital injury and cellulites: If trocar and cannula pierces the roof of antrum.
o Puncture of the posterior antral wall: Causes swelling in posterior part of cheek.
o Bleeding: Due to injury to nasal mucosa.
o Air embolism: Rare but may prove fatal.

03. CSF rhinorrhoea. [2007, 2014, 2019]


➢ Definition: Leakage of CSF into the nose is called CSF rhinorrhoea.
➢ Aetiology:
©shovandas 53
o Trauma: Most common cause (95%). It can be accidental or surgical.
o Inflammations: Mucoceles of sinuses, sinunasal polyposis, fungal infection of sinuses and osteomyelitis.
o Neoplasms: Both benign or malignant.
o Congenital lesions: Meningocele, meningoencephaloceles and gliomas.
o Idiopathic: Where cause is unknown and patient has spontaneous leak.
o Risk factors: CSF pressure rises on coughing, sneezing, nose blowing, straining on stools or lifting heavy weight
activities whih can cause CSF leak.
➢ Sites of leakage: CSF from anterior cranial fossa reaches the nose via:
o Cribriform plate of ethmoid sinus (most common site).
o Frontal sinus.
o Sphenoid sinus.
o Eustachian tube during temporal bone fracture.

➢ Diagnosis:
o Clinical examination:
▪ Sniff test: Can not sniff back.
▪ Handkerchief test: Do not stiffen.
▪ Halo/target/double ring test: Discharge, collected on a piece of filter paper, shows central red spot (blood)
and peripheral lighter halo.
▪ Reservoir sign: Make the patient lie down and let him sit suddenly & flex his neck → gush of fluid from nose.
o Biochemical examination:
▪ Presence of ß2 transferrin.
▪ Presence of ß trace protein.
▪ Glucose testing by oxidase peroxidase or biochemical estimation are no longer used.
o Radiological examination: To localize the site.
▪ High-resolution CT scan of base of the skull (best investigation).
▪ MRI.
▪ Intrathecal fluorescein study.
▪ CT cisternogram.
➢ Treatment:
o Conservative treatment:
▪ Bed rest, elevating the head of the bed and avoidance of nose blowing, sneezing and straining.
▪ Prophylactic antibiotics to prevent meningitis.
▪ Acetazolamide to decrease CSF formation.
▪ Above measures can be combined with lumbar drain if indicated.
o Surgical treatment: If there is no improvement with conservative therapy in 2 weeks. Surgical repair is done by:
▪ Neurosurgical intracranial approach.
▪ Extradural approaches:
✓ External ethmoidectomy for cribriform plate and ethmoid area.
✓ Trans-septal sphenoidal approach for sphenoid.
✓ Osteoplastic flap approach for frontal sinus leak.
▪ Transnasal endoscopic approach.

04. Ostiomeatal complex. [2017,2013,2011]


➢ Definition: The area on the lateral nasal wall (middle meatus) that receives drainage from the anterior and medial ethmoid
cells, frontal sinus and maxillary sinus.
©shovandas 54
➢ Boundaries:
o Medially: Middle turbinate.
o Laterally: Lamina papyracea.
o Superiorly and posteriorly: Basal lamella.
o Inferiorly and anteriorly: It is open.
➢ Components:
o Uncinate process: Hook like bony extension of medial wall.
o Hiatus semilunaris: Crescent passage between uncinate
process and ethmoid bulla through which middle meatus
communicates with ethmoidal infundibulum.
o Frontal recess: Drainage channel of frontal sinus.
o Bulla ethmoidalis: Most constant, largest anterior ethmoid
air cell projecting inferomedially over hiatus semilunaris.
o Ethmoidal infundibulum: Funnel shaped passage through
which anterior ethmoid cells & maxillary sinus drains into middle meatus.
o Maxillary sinus ostium: Drainage channel of maxillary sinus.
➢ Clinical significance:
o It is an anatomically constricted area that is prone to blockage, especially in the presence of structural anomalies,
mucosal swelling or tumours.
o An impairment in the ventilation of sinus due to such reasons lead to chronic rhinosinusitis.
➢ Anatomic variations:
o Deviated Nasal Septum.
o Concha bullosa: Enlarged, pneumatised middle turbinate.
o Intralamellar cell: Air cell within vertical portion of middle turbinate.
o Paradoxical middle concha: Convexity of turbinate directed towards lateral nasal wall.
o Haller cells: Infraorbital ethmoid air cells.
o Agger nasi cells: Extension of anterior ethmoid air cells into lacrimal bone.
o Uncinate process bulla.
o Deviation of uncinate process.
➢ Silent sinus syndrome: Maxillary sinus atelectasis due to chronic occlusion of maxillary sinus ostia, resulting in inward bowing
of all 4 walls of the sinus including the orbital floor and increased orbital volume, leading to enophthalmos and hypoglobus.

05. FESS. [2018, 2005]


➢ Indications:
o Different types of sinusitis with their complications.
o Antrochoanal polyp.
o Mucocele of frontoethmoid or sphenoid sinus.
o Control of epistaxis by endoscopic cautery.
o Removal of foreign body or benign tumours from the nose or sinus.
o Endoscopic septoplasty.
o Repair of CSF leak.
➢ Contraindications:
o Disease inaccessible by endoscopic procedure, e.g. lateral frontal sinus disease and stenosis of internal opening of
frontal sinus.
o Osteomyelitis.
o Threatened intracranial or intraorbital complication.
➢ Procedure:
o Anaesthesia: General anaesthesia.
o Position: Supine position with head resting on a ring or head rest.
o Techniques:
▪ Anterior to posterior (Stammberger’s technique): Surgery proceeds from uncinate process backward to
sphenoid sinus. Advantage of this technique is to tailor the extent of surgery to the extent of disease.
▪ Posterior to anterior (Wigand’s technique): Surgery starts at sphenoid sinus and proceeds anteriorly along
the base of skull and medial orbital wall. Mostly done in extensive polyposis or in revisional sinus surgery.
o Steps:
▪ Uncinectomy →
▪ Identification and enlargement of maxillary ostium →
▪ Bullectomy →
▪ Penetration of basal lamella and removal of posterior ethmoid cells →
▪ Clearance of frontal recess and frontal sinusotomy →
▪ Sphenoidotomy →
▪ Nasal packs.
©shovandas 55
➢ Post operative care:
o Antibiotics, antihistaminics and analgesics use.
o Nasal irrigations.
o Steroid nasal sprays.
o Endoscopic toilet.
➢ Complications:
▪ Major:
▪ Brain abscess. ▪ Massive haemorrhage requiring blood transfusion.
▪ Anosmia. ▪ Intracranial haemorrhage and direct brain trauma.
▪ Diplopia. ▪ Loss of vision/blindness.
▪ CSF leak. ▪ Injury to internal carotid artery in sphenoid sinus.
▪ Meningitis. ▪ Injury to nasolacrimal duct and epiphora.
▪ Death. ▪ Orbital haemorrhage.
o Minor:
▪ Periorbital ecchymosis. ▪ Stenosis of maxillary or frontal sinus opening.
▪ Periorbital emphysema. ▪ Exacerbation of asthma.
▪ Postoperative epistaxis. ▪ Hyposmia.
▪ Postop. infection: rhinitis, sinusitis. ▪ Dental pain.
▪ Adhesions.

06. Function of nose. [2002]


➢ Group A, question no. 04.

07. Rhinosporidiosis. [2004]


➢ Causative organism: Rhinosporidium seeberi.
o It has long been considered to be a fungus but difficult to classify. It has not been cultured so far.
o Some consider it to be a protozoa or a fish parasite belonging to DRIP clade.
o There are three stages in their life cycle: trophic stage, development of sporangia and production of endospores.
➢ Mode of transmission: The disease is acquired through contaminated water of ponds also frequented by animals.
➢ Clinical features:
o Site: The disease mostly affects nose and nasopharynx; other sites such as lip, palate, conjunctiva, epiglottis, larynx,
trachea, bronchi, skin, vulva and vagina may also be affected.
o Symptoms:
▪ In early stages, the patient may complain of nasal discharge which is often blood tinged and nasal stuffiness.
▪ Sometimes, frank epistaxis is the only presenting complaint.
o On examination:
▪ A leafy, polypoidal mass, pink to purple in colour, attached to nasal septum or lateral wall of nose.
▪ Sometimes it extends into the nasopharynx and may hang behind the soft palate.
▪ The mass is very vascular and bleeds easily on touch.
▪ Its surface is studded with white dots representing the sporangia of fungus.
➢ Diagnosis: Biopsy shows several sporangia, oval or round in shape and filled with spores which may be seen bursting through
its chitinous wall. It has not been possible to culture the organism or transfer the disease to experimental animals.
➢ Treatment:
o Surgical management: Complete excision of the mass with diathermy knife and cauterization of its base. Recurrence
may occur after surgical excision.
o Medical management: Not many drugs are effective against the disease. Dapsone has been tried with some success.

©shovandas 56
Group – d
01. Aetiology and management of ethmoidal polyp. [2020]
➢ Aetiology: Very complex and not well understood. Various diseases associated with the formation of nasal polypi are:
o Chronic rhinosinusitis: Polypi are seen in chronic rhinosinusitis of both allergic and non-allergic origin. Non-allergic
rhinitis with eosinophilia syndrome (NARES) is a form of chronic rhinitis associated with polypi.
o Asthma: 7% of the patients with asthma of atopic or non-atopic origin show nasal polypi.
o Aspirin intolerance: 36% of the patients with aspirin intolerance may show polypi. Samter’s triad consists of nasal
polypi, asthma and aspirin intolerance.
o Cystic fibrosis: 20% of patients with cystic fibrosis form polypi. It is due to abnormal mucus.
o Allergic fungal sinusitis: Almost all cases of fungal sinusitis form nasal polypi.
o Kartagener syndrome: This is a disease of ciliary dysfunction, which consists of bronchiectasis sinusitis, situs inversus
and ciliary dyskinesis.
o Young syndrome: This is a disease of ciliary dysfunction, which consists of sinopulmonary disease and azoospermia.
o Churg-Strauss syndrome: Consists of asthma, fever, eosinophilia, vasculitis and granuloma.
o Nasal mastocytosis: It is a form of chronic rhinitis in which nasal mucosa is infiltrated with mast cells but few
eosinophils. Skin tests for allergy and IgE levels are normal.
➢ Management:
o Conservative:
▪ Antihistaminic drugs.
▪ Leukotriene inhibitors.
▪ Mast cell stabilizers.
▪ Steroid nasal sprays.
o Surgical: Done when conservative treatment fails or if recurrence occurs.
▪ Polypectomy: 1-2 or multiple pedunculated polyps are removed with snare or by special forceps.
▪ Intranasal ethmoidectomy: In case of multiple sessile polyps, uncapping of the ethmoidal air cells by intra-
nasal route is done.
▪ Extra-nasal ethmoidectomy: In case of recurrent polyps approach is taken through the medial wall of the
orbit by an external incision, medial to medial canthus.
▪ Transantral ethmoidectomy: When pathology is also seen in the maxillary antrum, the antrum is opened
by Caldwell-Luc approach and the ethmoid air cell approached through the medial wall of the antrum.
▪ FESS: These days, the above procedures are superseded by FESS. Now ethmoidal polypi are removed by
functional endoscopic sinus surgery (FESS). Wigand's technique is used in cases of extensive polyposis.

02. Fungal sinusitis. [2020]


➢ Fungal ball:
o Causative organism: Aspergillus niger.
o Pathology: Chronic, immunocompetent, non-atopic.
o Clinical features:
▪ Maxillary sinus is the most commonly involved followed by sphenoid, ethmoid and the frontal in that order.
▪ Secretion: Cheesy secretion in seen.
▪ Bone erosion: No evidence of bone erosion.
▪ Fungal invasion: No evidence of fungal invasion.
o Diagnosis: CT scan shows heterogeneous appearance due to entrapment of metals and double density sign.
o Management: Removal of fungal ball and drainage by FESS. No antifungal needed.
➢ Allergic fungus sinusitis:
o Causative organisms: Aspergillus niger.
o Pathology: Chronic, immunocompetent, atopic or allergic.
o Clinical features:
▪ Secretion: Peanut butter like mucinous secretion having eosinophil, Charcot Leyden crystal, fungal hyphae.
▪ Bone erosion: There may be sinus expansion and bone erosion due to pressure.
▪ Fungal invasion: No evidence of fungal invasion.
o Diagnosis: CT scan shows heterogeneous appearance due to entrapment of metals and double density sign.
o Management: Surgical clearance of the sinuses with drainage and ventilation by FESS, combined with pre and post-
operative systemic steroids.
➢ Chronic invasive sinusitis:
o Causative organism: Mucor mycosis.
o Pathology: Chronic, immunocompromised.
o Clinical features:
▪ Chronic rhinosinusitis.
▪ Bone erosion: Occurs.
©shovandas 57
▪ Fungal invasion: Fungal invasion of mucosa, submucosa and granulomatous reaction with multinucleated
giant cells are seen. Intracranial or infraorbital invasion may also be seen.
o Diagnosis: CT scan shows thickened mucosa with opacification of sinus and bone erosion.
o Management: Surgical removal of the involved mucosa, bone and soft tissues followed by antifungal therapy with
i.v. amphotericin B, followed by itraconazole therapy.
➢ Fulminant fungal sinusitis:
o Causative organism: Aspergillus, Mucor.
o Pathology: Acute, immunocompromised (diabetic, on steroids).
o Clinical features:
▪ Mucor → rhinocerebral invasion → thrombosis → ischemia → necrosis → black eschar formed.
▪ Aspergillus → tissue invasion → sinusitis and sepsis → necrosis → no black eschar.
o Management: Surgical debridement of necrotic area with antifungal therapy.

03. Atrophic rhinitis (Ozaena). [2018, 2009, 2004, 1996]


➢ A chronic inflammation of nose characterized by atrophy of nasal mucosa and turbinate bones along with foul-smelling crusts.
➢ Primary atrophic rhinitis:
o Aetiology:
▪ Hereditary factors: Disease in more than one member in the same family.
▪ Endocrinal disturbance: More after puberty, less after menopause, more in female than male.
▪ Racial factors: More in whites than native Africans.
▪ Nutritional deficiency: Deficiency of vitamin A, D or iron or some other dietary factors.
▪ Infective: Klebsiella ozaenae, diphtheroids, Proteus vulgaris, E. coli, staphylococci and streptococci are con-
sidered to be secondary invaders responsible for foul smell rather than the primary causative organisms.
▪ Autoimmune process: Viral infection or other unspecified agents may trigger antigenicity of nasal mucosa.
o Pathology:
▪ Ciliated columnar epithelium is replaced by stratified squamous type.
▪ Atrophy of seromucinous glands, venous blood sinusoids and nerve elements.
▪ Obliterative endarteritis in the mucosa, periosteum and bone.
▪ Bone resorption → widening of nasal chambers.
▪ Paranasal sinuses are small due to their arrested development.
o Clinical features:
▪ Foul smelling nasal discharge (merciful anosmia).
▪ Nasal obstruction due to excessive crusting.
▪ Epistaxis during removal of crust.
▪ Atrophic laryngitis and atrophic pharyngitis may also be seen.
▪ Hearing impairment may be noticed because of obstruction to eustachian tube and middle ear effusion.
o Management:
▪ Medical:
✓ Nasal irrigation and removal of crusts with warm normal saline or an alkaline solution.
✓ 25% glucose in glycerine to inhibit the growth of proteolytic organisms.
✓ Spraying or painting the nose with appropriate antibiotics.
✓ Oestradiol spray to increase vascularity of nasal mucosa and regeneration of seromucinous glands.
✓ Placental extract injected submucosally in the nose may provide some relief.
✓ Systemic streptomycin 1 g/day for 10 days.
✓ Potassium iodide given by the mouth promotes and liquefies nasal secretion.
▪ Surgical:
✓ Young’s operation: Both the nostrils are closed completely just within the nasal vestibule by raising
flaps. They are opened after 6 months or later. In these cases, mucosa may revert to normal and
crusting reduced. Modified Young’s operation may also be done.
✓ Narrowing the nasal cavities:
• Submucosal injection of teflon paste.
• Insertion of fat, cartilage, bone or teflon strips under the mucoperiosteum of the floor and
lateral wall of nose and the mucoperichondrium of the septum.
• Section and medial displacement of lateral wall of nose.
➢ Secondary atrophic rhinitis:
o Infections like syphilis, lupus, leprosy and rhinoscleroma may cause destruction of nose leading to atrophic changes.
o It can also result from long-standing purulent sinusitis, radiotherapy or excessive surgical removal of turbinates.

04. Rhinolith. [2015]


➢ Rhinolith is a rare disorder of nasal cavity. They are calcareous concretions that are formed by the deposition of salts on an
intranasal foreign body.
➢ Aetiology: Foreign body in nasal cavity.
©shovandas 58
o Endogenous: Blood clots, ectopic teeth, bone fragments, dry secretions, mucosal necrosis.
o Exogenous: Fruit seeds, grains, plant material, beads, small stone fragments, cotton wool, plastic parts, insects etc.
➢ Pathogenesis:
o Incidental or accidental access of foreign body in the nasal cavity →
o Foreign body acts as the nucleus (thus becoming a focal point) for encrustation →
o Local inflammatory reaction, leading to deposition of:
▪ Inorganic substances: Carbonate and calcium phosphate, magnesium, iron and aluminium.
▪ Organic substances: Glutamic acid and glycine →
o Slow and progressive increases in size.
➢ Clinical features:
o Symptoms:
▪ Commonly seen in adults.
▪ Most common presentation is unilateral nasal obstruction and foul-smelling bloodstained discharge.
▪ Frank epistaxis and neuralgic pain may result from ulceration of the surrounding mucosa.
o Examination:
▪ A grey-brown or greenish-black mass with irregular surface and stony hard feel is seen in the nasal cavity
between the septum and turbinates.
▪ The mass is often brittle and a portion of it may break off while manipulating.
▪ Sometimes it is surrounded by granulations.
➢ Treatment:
o Surgically removed under general anaesthesia.
o Smaller ones are removed through anterior nares.
o Large ones need to be broken into pieces before removal.
o Some particularly hard and irregular ones require lateral rhinotomy.

05. Complications of SMR operation. [2014]


➢ SMR operation is submucous resection of nasal septum.
➢ Complications:
o Bleeding: It may require repacking, if severe.
o Septal haematoma: Evacuate the haematoma and give intranasal packing on both side of septum for equal pressure.
o Septal abscess: This can follow infection of septal haematoma.
o Perforation: When tears occur on opposing side of mucous membrane.
o Depression of bridge: Usually occurs in supratip area due to too much removal of cartilage along the dorsal border.
o Retraction of columella: Often seen when caudal strip of cartilage is not preserved.
o Persistence of deviation: It usually occurs due to inadequate surgery and may require revision operation.
o Flapping of nasal septum: Rarely seen, when too much of septal framework has been removed. Septum, which now
consists of two mucoperichondrial flaps, moves to the right or left with respiration.
o Toxic shock syndrome: It is rare after septal surgery. It can follow staphylococcal (sometimes streptococcal) infection
and is characterized by nausea, vomiting, purulent secretions, hypotension and rash. It should be diagnosed early. It
is treated by removal of packing, hydrating the patient, maintaining blood pressure, administering proper antibiotics.

06. Difference between septoplasty and SMR. [2011]

Features Septoplasty SMR


Type of procedure Conservative. Radical or destructive.
Age Any age. Adults (>17 years).
Incision Freer’s incision. Killian's incision.
Mucoperichondrial flap One side. Both side.
Risk of tearing the flap Low. High.
Elevation of flap Easier. Difficult.
Cartilage Preserved. Removed.
Caudal dislocation Corrected. Not corrected.
Risk of septal perforation Low. High.
Risk of haemorrhage Low. High.
Cosmetic deformity & complications Uncommon. Common.
Rhinoplasty Can be combined with rhinoplasty by Cannot be combined.
extending the incision.
Revision surgery Easier. Difficult.

©shovandas 59
07. Septoplasty. [2008]
➢ Septoplasty is a conservative approach to septal surgery, which has almost replaced the SMR operation.
➢ Indications:
o Deviated nasal septum and its complications.
o As a part of septorhinoplasty for cosmetic reasons.
o Recurrent epistaxis usually from the spur.
o As part of an another surgery: Septorhinoplasty, endoscopic dacryocystorhinostomy, trans-septal trans-sphenoidal
approach.
➢ Contraindications:
o Acute nasal or sinus infection.
o Untreated diabetes.
o Hypertension.
o Bleeding diathesis.
➢ Procedure:
o Anaesthesia: Local or general.
o Position: Reclining position with head-end of the table raised.
o Technique:
▪ Infiltrate the septum with 1% lignocaine with adrenaline 1:100,000 →
▪ In cases of deviated septum, Killian’s incision and in case of caudal dislocation, Freer’s incision is made →
▪ Raise mucoperichondrial / mucoperiosteal flap on one side only →
▪ Separate septal cartilage from the vomer and ethmoid plate and raise mucoperiosteal flap on the opposite
side of septum →
▪ Remove maxillary crest to realign the septal cartilage →
▪ Correct the bony septum by removing the deformed parts by:
✓ Scoring on the concave side.
✓ Cross-hatching or morselizing.
✓ Shaving.
✓ Wedge excision →
▪ Further manipulations like realignment of nasal spine, separation of septal cartilage from upper lateral
cartilages, implantation of cartilage strip in the columella or the dorsum of nose may be required →
▪ Trans-septal sutures are placed to coapt mucoperichondrial flaps →
▪ Nasal pack.
➢ Postoperative care:
o Overnight observation.
o Avoid strenuous exercise.
o Pack, if kept is removed the next day and patient be instructed not to blow the nose or sneeze hard.
o Saline spray or steam inhalation are encouraged after pack removal.
o Xylo- or oxymetazoline drops if nose becomes stuffy.
o Nasal splints, if used, are removed on fourth to eighth day and gentle suction of nose is done.
o Patient should avoid trauma to nose, wipe the nose gently and in no case push the nose from one side to another.
➢ Complications: Same as SMR operation. Group D, question no. 05.

08. Maxillary sinus. [2012, 2000]


➢ Group A, question no. 04.

09. Caldwell-Luc surgery. [2013, 2009]


➢ Caldwell-Luc operation is a process of opening the maxillary antrum through canine fossa by sublabial approach and dealing
with the pathology inside the antrum. The operation is also known as anterior antrostomy.
➢ Indications:
o Chronic maxillary sinusitis with irreversible changes in the sinus mucosa.
o Removal of foreign bodies or root of a tooth.
o Dental cyst.
o Oroantral fistula.
o Suspected neoplasm in the antrum and its biopsy.
o Recurrent antrochoanal polyp.
o Fracture of maxilla or blow-out fractures of the orbit.
o As an approach to ethmoids (Horgan’s transantral ethmoidectomy).
o Approach to pterygopalatine fossa for ligation of maxillary artery.
o Vidian neurectomy.
o Contraindicated in patient below 17 years of age.
➢ Procedure:
o Anaesthesia: Normally general, sometimes local anaesthesia is also given.
©shovandas 60
o Position: Reclining position with head-end of the table raised.
o Technique:
▪ Incision: A horizontal incision with its ends upward is made below the gingivolabial sulcus, from lateral inci-
sor to the second molar. It cuts through mucous membrane and periosteum.
▪ Elevation of flap: The mucoperiosteal flap is raised from the canine fossa to the infraorbital nerve avoiding
injury to the nerve.
▪ Opening the antrum: Using cutting burr or gouge and hammer, a hole is made in the antrum. Opening is
enlarged using Kerrison’s punch.
▪ Dealing with pathology: Once maxillary antrum has been opened, pathology is removed.
▪ Making nasoantral window: A curved haemostat is pushed into the antrum from the inferior meatus and
then this opening is enlarged with Kerrison’s and side-biting forceps to make a window <1.5 cm in diameter.
▪ Packing the antrum: Ribbon gauze, impregnated with liquid paraffin or Furacin™ ointment can be packed
in the antrum and its end brought out from the nasoantral window into the nose. Intrasinus packing is done
if there is severe bleeding. Pack is also kept in the nose.
▪ Closure of wound: Sublabial incision is closed with one or two catgut sutures.
➢ Postoperative care:
o Ice packs over the cheek in the first 24 h prevent oedema, haematoma and discomfort to the patient.
o Packing in the sinus and nose can be removed in 24-48 h.
o Antibiotics are given for 5-7 days.
o Patient should avoid blowing his nose for 2 weeks to avoid surgical emphysema.
o Maxillary sinus is irrigated through the antrostomy with saline. Patient can do it himself with Higginson’s syringe.
➢ Complications:
o Postoperative bleeding controlled by nasal pack.
o Anaesthesia of the cheek and teeth.
o Injury to nasolacrimal duct.
o Sublabial fistula.
o Osteomyelitis of maxilla (rare).

10. Indications of Caldwell-Luc operation. [2013]


➢ Group D, question no. 09.

11. Inferior turbinate. [2001]


➢ There are two inferior turbinate or inferior nasal conchae. Each inferior nasal concha projects downwards from the lateral
wall of the nasal cavity.
➢ Anatomy:
o Medial surface: Convex.
o Lateral surface: Deeply concave and forms the medial wall of the inferior meatus of the nose.
o Superior border: Thin and irregular. Its anterior
▪ Anterior part articulates: Articulates with conchal crest of maxilla and posterior.
▪ Posterior part: Articulates with conchal crest of the palatine bone. The middle presents
▪ Middle part: Presents three processes from before backwards: lacrimal, maxillary, and ethmoidal
✓ Lacrimal process.
✓ Maxillary process.
✓ Ethmoidal process.
o Inferior border: Thick and gently curved.
o Anterior and Posterior Ends: Both ends are pointed; the latter is more marrow.
➢ Clinical importance: The lacrimal process of inferior nasal concha articulates with the margins of nasolacrimal groove of
maxilla to form nasolacrimal canal., guarded at its terminal end by a mucosal valve called Hasner’s valve. The surgical fracture
of inferior nasal concha is sometimes needed to manage congenital lacrimal defects.

12. Uncinate process of nose. [2006]


➢ Uncinate process is a hook-like structure of ethmoid bone in the lateral wall of nose, running in from anterosuperior to
posteroinferior direction.
➢ Anatomy:
o Hiatus semilunaris: Its posterosuperior border is sharp and runs parallel to anterior border of bulla ethmoidalis; the
gap between the two is called hiatus semilunaris (inferior). It is a two-dimensional space of 1–2 mm width. The
anteroinferior border of uncinate process is attached to the lateral wall.
o Posteroinferior end of uncinate process is attached to inferior turbinate dividing the membranous part of lower
middle meatus into anterior and posterior fontanelle. The fontanel area is devoid of bone and consists of membrane
only and leads into maxillary sinus when perforated.
o Upper attachment of uncinate process: This shows great variation and may be inserted into lateral nasal wall, upw-
ards into base of skull or medially into middle turbinate. This also accounts for variations in drainage of frontal sinus.
©shovandas 61
o Infundibulum: The space limited medially by the uncinate process and frontal process of maxilla and sometimes
lacrimal bone, and laterally by the lamina papyracea is called infundibulum. Natural ostium of the maxillary sinus is
situated in the lower part of infundibulum. Accessory ostium or ostia of maxillary sinus are sometimes seen in the
anterior or posterior fontanel.

©shovandas 62
Third part

Throat

©shovandas 63
©shovandas 64
Contents

Chapters Page no.


Chapter 01 : pharynx 67

Chapter 02 : larynx and trachea 79

Chapter 03 : oral cavity and salivary glands 95

Chapter 04 : thyroid 99

Chapter 05 : oesophagus 103

Chapter 06 : recent advances 111

Chapter 07 : clinical methods and neck masses 115

Chapter 08 : operative surgery 119

©shovandas 65
©shovandas 66
Chapter 01

Pharynx

©shovandas 67
©shovandas 68
Group – a
01. Describe the aetiology and clinical features of acute retropharyngeal abscess. How will you treat it? [2005](3+4+3)
➢ Aetiology:
o It is commonly seen in children below 3 years, but can occur in adults also.
o In children: It is the result of suppuration of retropharyngeal lymph nodes or lymph nodes of Rouviere secondary to
infection in the adenoids, nasopharynx, posterior nasal sinuses or nasal cavity.
o In adults: It may result from penetrating injury of posterior pharyngeal wall or cervical oesophagus.
o Rare: Pus from acute mastoiditis tracks along under surface of petrous bone can present as retropharyngeal abscess.
o Risk factors:
▪ Poor oral hygiene. ▪ Immunocompromised state.
▪ Diabetes. ▪ Low socioeconomic status.
➢ Clinical features:
o Symptoms:
▪ Dysphagia. ▪ Stridor.
▪ Odynophagia. ▪ Croupy cough.
▪ Neck stiffness. ▪ Difficulty in breathing.
▪ Torticollis. ▪ ‘Hot potato voice', muffled voice.
▪ Truisms. ▪ Cervical lymphadenopathy.
▪ Chest pain on mediastinal extension.
▪ Refusal to extend neck due to pain or discomfort.
▪ Inability to tolerate oral secretions in late stage.
▪ Often associated with upper respiratory tract
infection.
o Signs:
▪ Bulge in posterior pharyngeal wall.
▪ Pharyngeal erythema.
▪ Usually unilateral.
▪ X-ray: Radiograph of soft tissue, lateral view of
neck shows widening of prevertebral shadow
and possibly even the presence of gas.
➢ Management:
o Incision and drainage:
▪ This is usually done without anaesthesia as there is risk of rupture of abscess during intubation.
▪ Child is kept supine with head low, and mouth is opened with a gag.
▪ A vertical incision is given in the most fluctuant area of the abscess.
▪ Suction should always be available to prevent aspiration of pus.
o Antibiotics: Suitable systemic antibiotics like ampicillin-sulbactam, clindamycin are given.
o Tracheostomy: A large abscess may cause mechanical obstruction to the airway or lead to laryngeal oedema. Trach-
eostomy becomes mandatory in these cases.
o Airway maintenance is necessary during treatment of abscess.

02. A child has come to OPD with a patch in throat. What are the causes of patch in throat? How will you differentiate between
a patch of acute follicular tonsillitis and with faucial diphtheria? [1996, 2000, 2009](6+4)
➢ Causes of patch in throat: Group B, question no. 03.
➢ Difference between acute follicular tonsillitis and faucial diphtheria:

Features Acute follicular tonsillitis Faucial diphtheria


Age Generally seen in > 5 years of age. Mostly seen in 2 to 5 years of age.
Onset Acute onset. Slower onset.
General symptoms More. Less.
Odynophagia More. Less.
Temperature High. Low.
Tachycardia Proportionate with temperature. Disproportionate.
Tonsil Enlarged and congested. Normal.
Membrane ✓ Bilateral. ✓ Unilateral.
✓ Whitish yellow. ✓ Grey.
✓ Thick. ✓ Thin.
✓ Limited to tonsil. ✓ Extended beyond tonsil.

©shovandas 69
✓ Easily removable. ✓ Bleeding occurs during removal.
Urine Normal. Albumin is present.
Culture Group A beta haemolytic Streptococci. Corynebacterium diphtheriae.
Lymph node involved Jugulodigastric (woods). Generalized (bull neck).

03. A young man comes to you with quinsy. Write down clinical features, complications, diagnosis & treatment. [1994, 1996,
1998, 2004, 2012] (3+2+2+3)
➢ Also called peritonsillar abscess. It is a collection of pus in the peritonsillar space which lies between the capsule of tonsil and
the superior constrictor muscle.
➢ Clinical features:
o Mostly affects adults and rarely the children.
o Usually unilateral though occasionally bilateral abscesses are recorded.
o General symptoms:
▪ They are due to septicaemia and resemble any acute infection.
▪ Fever (upto 104°F). ▪ Body aches.
▪ Chills. ▪ Headache.
▪ Rigors. ▪ Nausea.
▪ General malaise. ▪ Constipation.
o Local symptoms:
▪ Pain: Severe pain in throat, usually unilateral. Pain in ear due to IXth nerve involvement.
▪ Odynophagia: It is so marked that the patient cannot even swallow his own saliva which dribbles from the
angle of his mouth. Patient is usually dehydrated.
▪ Muffled and thick speech: Often called ‘hot potato voice'.
▪ Foul breath: Due to sepsis in the oral cavity and poor hygiene.
o Signs:
▪ The tonsil, pillars and soft palate on the involved side are
congested and swollen.
▪ Swollen and oedematous uvula, pushed to the opposite side.
▪ Bulging of the soft palate and anterior pillar above the tonsil.
▪ Mucous covering the tonsillar region.
▪ Cervical lymphadenopathy.
▪ Jugulodigastric lymph node involvement.
▪ Torticollis, patient keeps the neck tilted to the side of abscess.
➢ Complications:
o Parapharyngeal abscess: A peritonsillar abscess is a potential parapharyngeal abscess.
o Oedema of larynx: Tracheostomy may be required.
o Septicaemia: Other complications like endocarditis, nephritis, brain abscess may occur.
o Pneumonitis or lung abscess: Due to aspiration of pus, if spontaneous rupture of abscess has taken place.
o Jugular vein thrombosis.
o Spontaneous haemorrhage from carotid artery or jugular vein.
➢ Diagnosis:
o Clinical investigations: Signs and symptoms.
o Laboratory Investigations:
▪ Complete blood count (CBC) and electrolytes.
▪ Heterophil antibody test to rule out suspicion of infectious mononucleosis.
▪ Pus culture sensitivity from needle aspirate of the abscess.
▪ C-reactive protein blood culture might be required in patients presenting with features of sepsis.
o Radiological Investigations:
▪ X-ray: Of the soft tissue of the neck.
▪ Contrast-enhanced CT: Required in a very young patient where clinical diagnosis is not feasible.
▪ Intraoral USG: Simple, non-invasive imaging modality to differentiate peritonsillitis from a peritonsillar
abscess. Another important use is in the exact localization of site for abscess drainage.
➢ Management:
o Conservative treatment:
▪ Hospitalization.
▪ Intravenous fluids to combat dehydration.
▪ Antibiotics: Suitable antibiotics in large i.v. doses to cover both aerobic and anaerobic organisms.
▪ Analgesics: Paracetamol is given for relief of pain and to lower the temperature. Sometimes, stronger
analgesics like pethidine may be required. Aspirin is avoided because of the danger of bleeding.
▪ Oral hygiene should be maintained by hydrogen peroxide or saline mouth washes.

©shovandas 70
o Surgical treatment:
▪ Incision and drainage: Required if a frank abscess has formed. A peritonsillar abscess is opened at the point
of maximum bulge above the upper pole of tonsil or just lateral to the point of junction of anterior pillar with
a line drawn through the base of uvula. With the help of a guarded knife, a small stab incision is made and
then a sinus forceps inserted to open the abscess. Putting the sinus forceps the following day may also be
necessary to drain any reaccumulation.
▪ Interval tonsillectomy: Tonsils are removed 4–6 weeks following an attack of quinsy.
▪ Abscess or hot tonsillectomy: Some people prefer to do ‘hot’ tonsillectomy instead of incision and drainage.
Hot tonsillectomy has the risk of rupture of the abscess during anaesthesia and excessive bleeding at the
time of operation.

Group – b
01. Describe the clinical features, investigation and management of acute retropharyngeal abscess. [2017](4+3+3)
➢ Clinical features: Group A, question no. 01.
➢ Diagnosis:
o Clinical investigations: Signs and symptoms.
o Laboratory investigations: Complete blood count and blood cultures are necessary. White blood cell counts are
greater than 12,000 in most cases.
o Radiological investigations:
▪ X-ray:
✓ Radiograph of soft tissue, lateral view of neck shows widening of prevertebral shadow and possibly
even the presence of gas.
✓ Patients who are presenting with chest pain, should have a chest x-ray for mediastinal involvement.
▪ CT scan: CT scan of the neck with intravenous contrast is the best imaging study.
▪ Ultrasound: In children, ultrasound is preferred as it does not involve radiation and is portable. It can help
to the determine the size and location of the abscess also.
➢ Management: Group A, question no. 01.

02. Write down clinical features, treatment, complications of acute tonsillitis. [2013, 2016](4+3+3)
➢ Clinical features:
o Local Symptoms: The symptoms vary with severity of infection.
▪ Sore throat.
▪ Odynophagia: The child may refuse to eat anything due to local pain.
▪ Fever: It may vary from 38 to 40°C and may be associated with chills and rigors. Sometimes, a child presents
with an unexplained fever and it is only on examination that an acute tonsillitis is discovered.
▪ Earache: It is either referred pain from tonsil or result of acute otitis media which may occur as complication.
o General symptoms:
▪ Usually more marked than simple pharyngitis.
▪ Headache. ▪ General body aches.
▪ Malaise. ▪ Constipation.
▪ Abdominal pain due to mesenteric lymphadenitis.
o Signs:
▪ Often the breath is foetid and tongue is coasted.
▪ There is hyperaemia of pillars, soft palate and uvula.
▪ Tonsils:
✓ Acute follicular tonsillitis: Red and swollen with yellowish spots of purulent material presenting at
the opening of crypts.
✓ Acute membranous tonsillitis: Whitish membrane on the medial surface of tonsil which can be
easily wiped away with a swab.
✓ Acute parenchymatous tonsillitis: The tonsils may be enlarged and congested so much so that they
almost meet in the midline along with some oedema of the uvula and soft palate.
▪ The jugulodigastric lymph nodes are enlarged and tender.
➢ Management:
o Patient is put to bed and encouraged to take plenty of fluids.
o Analgesics: Aspirin or paracetamol are given according to the age of the patient to relieve local pain and fever.
o Antimicrobial therapy: Most of the infections are due to Streptococcus and penicillin is the drug of choice. Patients
allergic to penicillin can be treated with erythromycin. Antibiotics should be continued for 7–10 days.
➢ Complications:
o Chronic tonsillitis with recurrent acute attacks: This is due to incomplete resolution of acute infection. Chronic
infection may persist in lymphoid follicles of the tonsil in the form of microabscesses.
©shovandas 71
o Peritonsillar abscess.
o Parapharyngeal abscess.
o Cervical abscess: Due to suppuration of jugulodigastric lymph nodes.
o Acute otitis media: Recurrent attacks of acute otitis media may coincide with recurrent tonsillitis.
o Rheumatic fever: Often seen in association with tonsillitis due to Group A beta-haemolytic Streptococci.
o Acute glomerulonephritis: Rare these days.
o Subacute bacterial endocarditis: Acute tonsillitis in a patient with valvular heart disease may be complicated by
endocarditis. It is usually due to Streptococcus viridans infection.

03. Outline the clinical features, investigations and management of patches over tonsil in short. [2015, 2018]
➢ General investigations:
o History.
o Physical examination.
o Total and differential counts: For agranulocytosis, leukaemia, infectious mononucleosis.
o Blood smear: For atypical cells.
o Throat swab and culture: For pyogenic bacteria, Vincent angina, diphtheria, candidal infection.
o Bone marrow aspiration or needle biopsy.
o Other tests: Paul-Bunnell test.
➢ Agranulocytosis:
o Clinical features: Ulcerative necrotic lesions not only on tonsils but elsewhere in oropharynx. Patient is severely ill.
o Investigations:
▪ Acute fulminant form: TLC is decreased to <2000/cu mm or even as low as 50/cu mm and polymorph
neutrophils may be reduced to 5% or less.
▪ Chronic or recurrent form: TLC is reduced to 2000/cu mm with less marked granulocytopenia.
o Treatment: Close monitoring with serial blood counts, withdrawal of the offending agent e.g. medication.
➢ Leukaemia:
o Clinical features: In children, 75% of leukaemias are acute lymphoblastic and 25% acute myelogenous or chronic,
while in adults 20% of acute leukaemias are lymphocytic and 80% nonlymphocytic.
o Investigations: Peripheral blood shows TLC >1 lack/cu mm. It may be normal or less than normal. Anaemia is always
present and may be progressive. Blasts cells are seen on examination of the bone marrow.
o Treatment: For slow-growing leukaemias, treatment may include monitoring. For aggressive leukaemias, treatment
includes chemotherapy followed by radiation and stem-cell transplant.
➢ Vincent angina:
o Clinical features: Low fever, discomfort in throat, membrane over one tonsil, which can be easily removed revealing
an irregular ulcer. Swollen bleeding dirty gingiva causing necrotising gingivitis.
o Investigations: Throat swab will show fusiform bacilli and spirochaetes.
o Treatment: Irrigation and debridement of necrotic areas, oral hygiene along with antibiotics and analgesics.
➢ Infectious mononucleosis:
o Clinical features: Bilaterally enlarged congested tonsils, covered with membrane; marked local discomfort, posterior
triangle lymph node enlargement along with splenomegaly. Antibiotic therapy is failed.
o Investigations: Blood smear shows more than 50% lymphocytes, of which about 10% are atypical. TLC may be normal
in the 1st week but rises in 2nd week. Paul-Bunnell test will shows high titre of heterophil antibody.
o Treatment: Symptomatic treatment with analgesics, anti-pyretics along with rest and fluid intake.
➢ Traumatic ulcer:
o Clinical features: Membrane over tonsil without any other general symptoms.
o Investigations: History of any injury to oropharynx, which heals by formation of a membrane within 24 hours.
o Treatment: Symptomatic treatment.
➢ Aphthous ulcers:
o Clinical features: They may involve any part of oral cavity (tonsil and pillars), sometimes solitary. It may be small or
quite large and very painful.
o Investigations: Most important diagnostic feature is history of recurrent, self healing ulcer at fairly regular intervals.
o Treatment: Normally self-healed within 2 weeks. Anti-inflammatory drugs, steroids are given.
➢ Malignancy: Squamous cell carcinomas are most common. Lymphomas may also be seen.
o Clinical features: Persistent sore throat, difficulty in swallowing, pain in the ear or lump in the neck are the presenting
symptoms. Later on, bleeding from the mouth, fetor oris and trismus may occur.
o Investigations: Palpation of tonsillar area should never be omitted to find the extent of tumour. Biopsy is
essential for histological typing.
o Treatment: Radiotherapy, surgery or combined therapy is done.
➢ Moniliasis:
o Clinical features: Candidial infection in the throat presenting white membrane with dysphasia and odynophagia.
o Investigations: Throat swab culture shows fungal growth.
o Treatment: Topical antifungal medication is given.
©shovandas 72
➢ Pyogenic infection:
o Clinical features: Membranous tonsillitis due to pyogenic organisms. It presents with exudative membrane over the
medial surface of the tonsils, along with the features of acute tonsillitis.
o Investigations: Throat swab culture shows bacterial growth.
o Treatment: Antibiotic medication is given along with analgesics and antipyretics.
➢ Diphtheria:
o Clinical features: Slower in onset with less local discomfort. The membrane extends beyond the tonsils, on to the
soft palate and is dirty grey in colour. It is adherent and its removal leaves a bleeding surface.
o Investigations: Urine may show albumin. Smear and culture of throat swab will reveal Corynebacterium diphtheriae.
o Treatment: Antibiotics used are benzyl penicillin 600 mg 6 hourly for 7 days. Erythromycin is used in penicillin-
sensitive individuals (500 mg 6 hourly orally).

Group – c
01. Retropharyngeal abscess. [2018, 2002]
➢ Acute retropharyngeal abscess: Group A, question no. 01.; Group B, question no. 01.
➢ Chronic retropharyngeal abscess: Also called prevertebral abscess.
o Aetiology: It is tubercular in nature and is the result of:
▪ Caries of cervical spine: Presents centrally behind the prevertebral fascia. Diagnosed by X-ray.
▪ Tuberculous infection: Infection of retropharyngeal lymph nodes secondary to tuberculosis of deep cervical
nodes. It is limited to one side of midline as in true retropharyngeal abscess behind buccopharyngeal fascia.
o Clinical features:
▪ Discomfort in throat.
▪ Dysphagia, not marked.
▪ Fluctuant swelling in posterior pharyngeal wall centrally or on one side of midline.
▪ Tuberculous neck nodes.
o Treatment:
▪ Incision and drainage: Vertical incision along anterior border of sternomastoid (for low abscess) or along
its posterior border (for high abscess) is done.
▪ Full course antitubercular therapy.

02. Acute retropharyngeal abscess. [2013, 2016]


➢ Group A, question no. 01.; Group B, question no. 01.

03. Peritonsillar abscess. [2008, 2010, 2014, 2015]


➢ Group A, question no. 03.
➢ Aetiopathogenesis:
o Peritonsillar abscess usually follows acute tonsillitis though it may arise de novo without previous history of sore
throats. First, one of the tonsillar crypts, usually the crypta magna, gets infected and sealed off. It forms an intra-
tonsillar abscess which then bursts through the tonsillar capsule to set up peritonsillitis and then an abscess.
o Culture of pus reveals pure growth of Streptococcus pyogenes, S. aureus or anaerobic organisms. More often the
growth is mixed.

04. Chronic retropharyngeal abscess. [2004]


➢ Group C, question no.01.

05. Treatment of peritonsillar abscess. [1996]


➢ Group A, question no. 03.

06. Adenoid facies. [2009]


➢ Adenoid facies, also known as long face syndrome, refers to long, open-mouthed face of children with adenoid hypertrophy.
➢ Aetiology:
o Physiological enlargement in children with generalized lymphoid hyperplasia.
o Recurrent rhinitis, sinusitis or chronic tonsillitis.
o Allergy of the upper respiratory tract.
➢ Clinical features:
o Characteristic facial features:
▪ Chronic nasal obstruction and mouth breathing lead to characteristic facial feature called adenoid facies.
▪ Elongated face. ▪ Open mouth.
▪ Dull expression. ▪ Prominent and crowded upper teeth.
▪ Hitched up upper lip.
©shovandas 73
▪ Pinched in appearance of nose due to disuse atrophy of alaenasi.
▪ Highly arched hard palate as the moulding action of the tongue on palate is lost.
o Nasal symptoms:
▪ Nasal obstruction. ▪ Sinusitis.
▪ Nasal discharge. ▪ Epistaxis.
o Aural symptoms:
▪ Eustachian tube obstruction. ▪ Chronic suppurative otitis media.
▪ Recurrent of acute otitis media ▪ Serous otitis media.
o Pulmonary hypertension, loss of concentration can also occur.
➢ Diagnosis:
o Flexible nasopharyngoscopy.
o Soft tissue lateral radiograph of nasopharynx to reveal the size of adenoids and its extent.
➢ Treatment:
o Breathing exercises, decongestant nasal drops and antihistaminics for any co-existent nasal allergy.
o Adenoidectomy, if symptoms are marked.

07. Waldeyer's ring. [2016]


➢ The aggregations of lymphoid tissue underneath the epithelial lining of pharyngeal wall called tonsils, surround the comme-
ncement of air and food passages. These aggregations together constitute an interrupted circle called Waldeyer’s ring, which
forms the special feature of the interior of the pharynx.
➢ Function: Provide local and systemic immunity to children. Gets atrophied in later stage.
➢ Contents:
o Adenoids:
▪ Also called as Luschka's tonsil or nasopharyngeal tonsil.
▪ Located in nasopharynx.
o Tubal tonsils:
▪ Also called as Gerlach's tonsil.
▪ Located behind eustachian tube in fossa of Rosenmuller.
o Palatine tonsils:
▪ Also known as faucial tonsils.
▪ Located in oropharynx.
o Lingual tonsils: Located in base of the tongue.
o Lateral pharyngeal bands.
o Nodules: Located in posterior pharyngeal wall.

08. Diagnosis of diphtheria. [2017]


➢ Clinical investigations:
o Onset is slower.
o Membrane over tonsil:
▪ Unilateral. ▪ Thin.
▪ Grey. ▪ Tenacious.
▪ Extended beyond tonsil to the soft palate and posterior pharyngeal wall.
▪ Bleeding occurs during removal.
o Cervical lymph nodes, particularly the jugulodigastric, become enlarged and tender, sometimes presenting a ‘bull-
neck’ appearance.
o Patient is ill and toxaemic but fever seldom rises above 38°C.
➢ Laboratory investigations:
o Isolation of C. diphtheriae from a Gram stain or Albert stain of throat swab culture.
o In vivo toxin demonstration by genie pig inoculation and in vitro toxin demonstration by Elek's gel precipitation test.
o Toxin detection can also be done by PCR, ELISA or ICT.
o Albumin may be present in urine.

09. Obstructive sleep apnoea. [2013]


➢ Apnoea means no breathing. When there is cessation of airflow during sleep, it is called OSA.
➢ Aetiology:
o Upper airway collapse.
o Obstructive conditions of nose, nasopharynx, oral cavity and oropharynx, base of tongue or larynx.
➢ Consequences:
o Congestive heart failure/cor pulmonale. o Atrial and ventricular arrhythmias and left heart failure.
o Polycythaemia and hypertension. o Attacks of angina.

©shovandas 74
o Snoring spouse syndrome. o Decreased libido.
o Loss of memory. o Traffic accidents.
➢ Management:
o Non-surgical:
▪ Weight reduction.
▪ Avoidance of alcohols, sedatives and smoking.
▪ Positional therapy.
▪ Intraoral devices: Mandibular advancement device, tongue retention device.
▪ Positive airway pressure (PAP): Continuous PAP (CPAP), bilevel PAP (BiPAP), autotitrating PAP (APAP).
o Surgical:
▪ Tonsil & adenoid surgery: In children.
▪ Nasal surgery: Septoplasty, turbinate reduction, polypectomy.
▪ Palate surgery: Uvulopalatoplasty (UPP), uvulopalatopharyngoplasty (UPPP).
▪ Advancement pharyngoplasty.
▪ Tongue base surgery: Lingual tonsillectomy, laser midline glossectomy.
▪ Tongue base radiofrequency reduction.
▪ Mandibular osteotomy with genioglossus advancement.
▪ Hyoid myotomy and suspension: Hyoid bone suspended to lower border of mandible or upper border of
thyroid cartilage.
▪ Maxillomandibular osteotomy and advancement.
▪ Tracheostomy: Gold standard.

Group – d
01. Juvenile Nasopharyngeal Angiofibroma. [2010]
➢ Introduction:
o Juvenile Nasopharyngeal Angiofibroma is also called as Nasopharyngeal Fibroma.
o It is a rare tumour, though it is most common benign tumour of nasopharynx.
o Locally invasive in nature.
o Arises from sphenopalatine foramen.
o Blood supply by maxillary artery.
➢ Aetiology: It is predominantly seen in adolescent males in 2nd decade of life, it is thought to be testosterone dependent.

Spread Radkowski's staging


01. Medial spread Stage I
(a) Nasopharynx and nose Ia
(b) Sinuses Ib
02. Lateral spread Stage II
(a) Sphenopalatine fossa IIa
(b) Completely filled sphenopalatine fossa
IIb
and orbit (proptosis)
(c) Infratemporal fossa (swelling cheek) IIc
03. Intracranial spread Stage III
(a) Limited IIIa
(b) Extensive IIIb

➢ Clinical features:
o Profuse recurrent and continuous epistaxis.
o Progressive nasal obstruction and denasal speech.
o Conductive hearing loss and otitis media with effusion.
o Mass in nasopharynx.
o Broadening of nasal bridge, proptosis, swelling of cheek, infratemporal fossa or involvement of IInd , IIIrd , IVth and VIth
cranial nerves.
➢ Investigations:
o Contrast enhanced CT: Shows anterior bowing of the posterior wall of maxillary sinus, called antral sign or Holman-
Miller sign.
o MRI: Can also be done.
o Cardiac angiography: To show the extent.

©shovandas 75
➢ Management:
o Surgical excision: Approaches taken are:
▪ Endoscopic.
▪ Transnasal.
▪ Transmaxillary: Lateral rhinotomy, mid facial degloving.
▪ Infratemporal.
o Radiotherapy: Only for stage IIIb, when surgery is not possible.

02. Indication of tonsillectomy. [2012]


➢ Absolute:
o Recurrent infections of throat:
▪ Seven or more episodes in 1 year.
▪ Five episodes per year for 2 years.
▪ Three episodes per year for 3 years.
▪ Two weeks or more of lost school or work in 1 year.
o Peritonsillar abscess: In children, tonsillectomy is done 4–6 weeks after abscess has been treated. In adults, second
attack of peritonsillar abscess forms the absolute indication.
o Tonsillitis: It causes febrile seizures.
o Hypertrophy of tonsils: It causes:
▪ Airway obstruction (sleep apnoea).
▪ Difficulty in deglutition.
▪ Interference with speech.
o Suspicion of malignancy: A unilaterally enlarged tonsil may be a lymphoma in children and an epidermoid carcinoma
in adults. An excisional biopsy is done.
➢ Relative:
o Diphtheria carriers, who do not respond to antibiotics.
o Streptococcal carriers, who may be the source of infection to others.
o Chronic tonsillitis with bad taste or halitosis which is unresponsive to medical treatment.
o Recurrent streptococcal tonsillitis in a patient with valvular heart disease.
➢ As a part of another operation:
o Palatopharyngoplasty: Done for sleep apnoea syndrome.
o Glossopharyngeal neurectomy: Tonsil is removed first and then IX nerve is severed in the bed of tonsil.
o Removal of styloid process.

03. Reactionary haemorrhage after tonsillectomy. [2004]


➢ One of the most common immediate complication of tonsillectomy is reactionary hemorrhage.
➢ Presentation:
o Occurs within a period of 24 hours after operation.
o Presence of a clot prevents the clipping action of the superior constrictor muscle on the vessels which pass through
it (compare postpartum uterine bleeding).
➢ Management:
o It can be controlled by simple measures such as removal of the clot, application of pressure or vasoconstrictor.
o If above measures fail, ligation or electrocoagulation of the bleeding vessels can be done under general anaesthesia.

04. Killian's dehiscence. [2008]


➢ Anatomy:
o Inferior constrictor muscle has two parts:
▪ Thyropharyngeus made up of oblique fibres and
▪ Cricopharyngeus made up of transverse fibres.
o The potential gap posteriorly between the thyropharyngeus and cricopharyngeus is called pharyngeal dimple or
Killian’s dehiscence.
➢ Clinical importance:
o Gateway of tears: It is also called ‘gateway of tears' as perforation can occur at this site during oesophagoscopy.
o Zenker's diverticulum:
▪ The mucosa and submucosa of the pharynx may bulge through this weak area to form a pharyngeal pouch
or diverticulum.
▪ Formation: The formation is attributed to the neuro-muscular incoordination in this region, which may be
because the two parts of the inferior constrictor have different nerve supply. Propulsive thyropharyngeus
is supplied by pharyngeal plexus and sphincteric cricopharyngeus is supplied by the recurrent laryngeal
nerve. If cricopharyngeus fails to relax when thyropharyngeus contracts, the bolus of food is pushed back-
wards and tends to produce a diverticulum.

©shovandas 76
05. Keratosis pharyngis. [2003, 2006]
➢ Keratosis pharyngis is a benign pathological condition where keratin grows on the surface of the pharynx.
➢ Clinical features:
o Aetiology is unknown.
o Presence of whitish-yellow dots on the pharyngeal wall, tonsils or lingual tonsils.
o Firmly adherent and cannot be wiped off.
o Surrounding has no inflammation or any other constitutional signs.
o Mild dysphagia may occur sometimes.
➢ Treatment: Generally heals spontaneously.

06. Vincent angina. [2005]


➢ It is a common, acute, non-contagious bacterial infection of throat.
➢ Also called trench mouth.
➢ Aetiology: The causative agents are actually normal flora of the mouth, but they are potential pathogen in presence of
underlying malnutrition or viral infection.
➢ Clinical features:
o More often the disease affects young adults and middle-aged persons.
o Low fever.
o Inflamed pharyngeal mucosa.
o Discomfort in throat.
o Membrane over one tonsil.
o Membrane can be easily removed revealing an irregular ulcer.
o Swollen, bleeding, dirty gingiva causes necrotising gingivitis.
➢ Investigations: Throat swab will show fusiform bacilli and spirochaetes Borrelia vincentii.
➢ Treatment:
o Irrigation and debridement of necrotic areas.
o Oral hygiene:
▪ Frequent mouth washes with sodium bicarbonate solution.
▪ Attention to dental hygiene.
o Antibiotics:
▪ Penicillin or erythromycin.
▪ Metronidazole.
o Analgesics.

©shovandas 77
©shovandas 78
Chapter 02

Larynx
And
trachea

©shovandas 79
©shovandas 80
Group – a
01. Draw a neat labelled diagram of larynx seen on indirect laryngoscopy. Describe briefly levels and groups of lymph node in
the neck. Describe pre-epiglottic space & its significance. [2010](2+3+3+2)
➢ Structures seen on indirect laryngoscopy: Indirect laryngoscopy permits examination of structure of the oropharynx, larynx
and laryngopharynx.
o Larynx: Epiglottis, aryepiglottic fold, arytenoids, cuneiform and corniculate cartilages, ventricular bands, ventricles,
true cords, anterior commissure, posterior commissure, subglottis and rings of trachea.
o Laryngopharynx: Both pyriform fossae, postcricoid region, posterior wall of laryngopharynx.
o Oropharynx: Base of tongue, lingual tonsils, valleculae,
medial and lateral glossoepiglottic folds.
➢ Levels and groups of lymph node in the neck:
o Level I:
▪ IA:
✓ Submental LNs.
✓ Location: In the submental triangle, i.e.
between right & left anterior bellies of
digastric muscles and the hyoid bone.
✓ Drainage: Lower lip, tip of tongue.
▪ IB:
✓ Submandibular LNs.
✓ Location: Between anterior and posterior bellies of digastric muscle and the lower border of the
body of mandible.
✓ Drainage: Buccal mucosa, parotid, ear lobules, teeth, lateral borders of tongue.
o Level II:
▪ Upper jugular LNs.
▪ Location: Along the upper third of jugular vein , i.e. between the skull base above and the level of lower
border of hyoid bone (or bifurcation of carotid artery) below.
▪ Drainage: Tonsil, posterior 1/3rd of tongue, pharynx, larynx.
o Level III:
▪ Middle jugular LNs.
▪ Location: Along the middle 3rd jugular vein, from the level of hyoid
bone above , to the level of lower border of cricoid cartilage (or
where omohyoid muscle crosses the jugular vein) below.
▪ Drainage: Posterior 1/3rd of tongue, pharynx, larynx.
o Level IV:
▪ Lower jugular LNs.
▪ Location: Along the lower third of the jugular vein, from lower
border of cricoid cartilage to the clavicle. Virchow’s node is in-
cluded into this level.
▪ Drainage: Posterior 1/3rd of tongue, pharynx, larynx.
o Level V:
▪ Posterior cervical group of LNs or LNs of posterior triangle.
▪ Location: In the posterior triangle, i.e. between posterior border of sternocleidomastoid (anteriorly),
anterior border of trapezius (posteriorly) and the clavicle below.
▪ Sub-groups: They include lymph nodes of spinal accessory chain, transverse cervical nodes and supraclav-
icular nodes. Level V nodes are further subdivided into upper, middle and lower, corresponding to planes
that define levels II, III and IV.
▪ Drainage: Scalp, oesophagus, lower part of pharynx.
o Level VI:
▪ Anterior compartment LNs or central LNs.
▪ Location: Between the medial borders of sternocleidomastoid muscles (or carotid sheaths) on each side,
hyoid bone above & suprasternal notch below. They include prelaryngeal , pretracheal and paratracheal
nodes.
▪ Drainage: Thyroid, larynx, pharynx.
o Level VII:
▪ LNs of upper mediastinum.
▪ Location: Below the suprasternal notch.
▪ Drainage: Nasopharynx, eustachian tube, soft palate.

©shovandas 81
➢ Pre-epiglottic space:
o Also called pre-epiglottic space (PES) of Boyer.
o It is a triangular fat containing space between
epiglottis and the hyoid bone.
o Boundaries:
▪ Superior: Hyoepiglottic ligament.
▪ Anterior: Thyrohyoid membrane, upper
part of thyroid cartilage.
▪ Inferior: Thyroepiglottic ligament.
▪ Posterior: Infra-hyoid epiglottis, quadrangular membrane.
▪ Anterior: Continuous with paraglottic space.
o Contents: Fat, lymphatics and areolar tissue.
➢ Significance of PES:
o This area is rich in lymphatics which drain from the
supraglottic larynx to cervical nodes.
o The PES is at high risk of involvement of oropharyn-
geal and supraglottic laryngeal carcinomas.
o Involvement of PES indicates TNM staging T3.
o PES involvement necessitates an aggressive surgical
treatments with higher morbidity such as supraglot-
tic or total laryngectomy, in contrast to more conser-
vative treatment such as local resection or radiation
therapy.

02. Describe trachea and its function. [2002](5+5)


➢ Trachea:
o The trachea is a noncollapsible membranocartilaginous tube forming the beginning of lower respiratory passage.
o Extent: The trachea is the continuation of the larynx. It extends from lower border of cricoid cartilage at the lower
border of the C6 vertebra to the lower border of T4 vertebra in supine position, where it terminates by dividing in-
to right and left principal bronchi.
o Dimensions:
▪ Length: The trachea is a 4–6 inch (10–11 cm) long tube.
▪ Cervical part: The upper half of trachea, located in the neck.
▪ Thoracic part: The lower half of trachea, located in the superior mediastinum of the thoracic cavity.
▪ Diameter:
✓ Adults: The external diameter of trachea is 2 cm in males and 1.5 cm in females.
✓ Newborn: It is about 3 mm in newborns and remains so up to the third year of life; thereafter the
lumen increases by 1 mm each year up to 12 years, after which it remains fairly constant.
o Structure: The trachea is composed of about 16–20 C-shaped rings of hyaline cartilages lying one above the others.
The cartilages are deficient posteriorly where the gap is filled by connective tissue and an involuntary muscle called
trachealis. The absence of cartilages on the posterior aspect allows expansion of oesophagus during deglutition. In
cross section the trachea appears D-shaped or horseshoe-shaped.
o Arterial supply: Branches of the inferior thyroid arteries.
o Venous drainage: Left brachiocephalic vein.
o Lymphatic drainage: Pre and para tracheal LNs.
o Nerve supply:
▪ Parasympathetic fibres:
✓ Derived from vagus through the recurrent laryngeal nerve.
✓ They are secretomotor and sensory to the mucus membrane
and motor to the trachealis muscle.
▪ Sympathetic fibres:
✓ Derived from the middle cervical sympathetic ganglion.
✓ They are vasomotor in nature.
➢ Function:
o Respiration: The primary function of the trachea is to transport air between la-
rynx and the lungs.
o Protection: The mucus in trachea helps capture microorganisms such as viruses
and harmful bacteria before they enter the lungs. These trapped organisms are
expelled upwards and can either be excreted from the body as phlegm or swall-
owed in the oesophagus.
o Thermoregulation:
▪ On cold days, the trachea helps warm & provide humidity to the air before it reaches the lungs.
©shovandas 82
▪ On hot days, the trachea helps cool the air through evaporation.
o Coughing: If a foreign object, liquid, or irritant (like smoke) enters the trachea, the muscles can contract violently,
causing coughing to expel the substance.

Group – b
01. A 65 years old male presents with hoarseness of voice for last 3 months. How will you investigate the case to arrive at a
diagnosis. [2013, 2011]
➢ Hoarseness of voice: Hoarseness is defined as roughness of voice resulting from variations of periodicity and/or intensity of
consecutive sound waves.
➢ Investigations:
o History:
▪ Mode of onset and duration of illness, patient’s occupation, habits and associated complaints are importa-
nt and would often help to elucidate the cause.
▪ Any hoarseness persisting for more than 3 weeks deserves examination of larynx.
▪ Malignancy should be excluded in patients above 40 years.
o Indirect laryngoscopy: Many of the local laryngeal causes can be diagnosed.
o Examination: Neck, chest, cardiovascular and neurological system examination would help to find cause for laryng-
eal paralysis.
o Laboratory investigations and radiological examination: Should be done as per dictates of the cause suspected on
clinical examination.
o laryngoscopy and microlaryngoscopy: Help in detailed examination, biopsy of the lesions & assessment of the mo-
bility of cricoarytenoid joints.
➢ Differential diagnosis:
o Inflammation:
▪ Acute: Acute laryngitis usually following cold, influenza, exanthematous fever, laryngo-tracheo-bronchitis,
diphtheria.
▪ Chronic:
✓ Specific: Tuberculosis, syphilis, scleroma, fungal infections.
✓ Non-specific: Chronic laryngitis, atrophic laryngitis.
o Tumour:
▪ Benign: Papilloma (solitary and multiple), haemangioma, chondroma, fibroma, leukoplakia.
▪ Malignant: Carcinoma.
▪ Tumour like masses: Vocal nodule/ polyp, angiofibroma, amyloid tumour, contact ulcer, cyst, laryngocele.
o Trauma: Submucosal haemorrhage, laryngeal trauma (blunt and sharp), foreign bodies, intubation.
o Paralysis: Paralysis of recurrent, superior laryngeal or both nerves.
o Fixation of cords: Arthritis or fixation of cricoarytenoid joints.
o Congenital: Laryngeal web, cyst, laryngocele.
o Miscellaneous: Dysphonia plica ventricularis, myxoedema, gout.
o Functional: Hysterical aphonia.

02. What is stridor? A 3 years old child presented with stridor for 2 hours. What are the common causes and outline the inve-
stigation of such cases. [2008, 2006](1+4+5)
➢ Stridor: Noisy respiration produced by turbulent airflow through the narrowed air passages, heard during inspiration, expi-
ration or both.
➢ Common causes/Differential diagnosis:
o Nose: Choanal atresia in newborn.
o Tongue: Macroglossia due to cretinism, haemangioma, lymphangioma, dermoid at base of tongue, lingual thyroid.
o Mandible: Micrognathia, Pierre-Robin syndrome. In these cases, stridor is due to falling back of tongue.
o Pharynx: Congenital dermoid, adenotonsillar hypertrophy, retropharyngeal abscess, tumours
o Larynx:
▪ Congenital: Laryngeal web, laryngomalacia, cysts, vocal cord paralysis, subglottic stenosis.
▪ Inflammatory: Epiglottitis, laryngotracheitis, diphtheria, tuberculosis.
▪ Neoplastic: Haemangioma and juvenile multiple papillomas, carcinoma in adults.
▪ Traumatic: Injuries of larynx, foreign bodies, oedema following endoscopy, or prolonged intubation.
▪ Neurogenic: Laryngeal paralysis due to acquired lesions.
▪ Miscellaneous: Tetanus, tetany, laryngismus stridulus.
o Trachea and bronchi:
▪ Congenital: Atresia, stenosis, tracheomalacia.
▪ Inflammatory: Tracheobronchitis.
▪ Neoplastic: Tumours of trachea.
©shovandas 83
▪ Traumatic: Foreign body, stenosis trachea (e.g. following prolonged intubation or tracheostomy).
o Lesions outside respiratory tract:
▪ Congenital: Vascular ring (cause stridor and dysphagia), oesophageal atresia, tracheo-oesophageal fistula,
congenital goitre, cystic hygroma.
▪ Inflammatory: Retropharyngeal and retro-oesophageal abscess.
▪ Traumatic: Foreign body oesophagus (secondary tracheal compression).
▪ Tumours: Masses in neck.
➢ Investigations:
o History:
▪ Time of onset: To find whether cause is congenital or acquired.
▪ Mode of onset:
✓ Sudden onset: Foreign body, oedema.
✓ Gradual and progressive onset: Laryngomalacia, subglottic haemangioma, juvenile papillomas.
▪ Duration:
✓ Short: Foreign body, oedema, infections.
✓ Long: Laryngomalacia, laryngeal stenosis, subglottic haemangioma, anomalies of tongue and jaw.
▪ Relation to feeding: Aspiration in laryngeal paralysis, oesophageal atresia, laryngeal cleft, vascular ring, fo-
reign body oesophagus.
▪ Cyanotic spells: Indicate need for airway maintenance.
▪ Aspiration or ingestion of a foreign body.
▪ Laryngeal trauma: Blunt injuries to larynx, intubation, endoscopy.
o Physical examination:
▪ Examination for recession in suprasternal notch, sternum, intercostal spaces and epigastrium during inspi-
ratory efforts due to associated respiratory distress.
▪ Note whether stridor is inspiratory, expiratory or biphasic which indicates probable site of obstruction.
▪ Sound:
✓ Snoring or snorting sound: Nasal or nasopharyngeal cause.
✓ Gurgling sound and muffled voice: Pharyngeal cause.
✓ Hoarse voice: Laryngeal cause at vocal cords. Normal in laryngomalacia and subglottic stenosis.
✓ Expiratory wheeze: Bronchial obstruction.
▪ Associated fever indicates infective condition.
▪ Stridor of laryngomalacia , micrognathia , macroglossia and innominate artery compression disappears in
prone position.
▪ Sequential auscultation with unaided ear and with stethoscope over the nose, open mouth , neck and the
chest helps to localize the probable site of origin of stridor.
▪ Examination of nose, tongue, jaw, pharynx and larynx can exclude local pathology in these areas. In adults,
indirect laryngoscopy can be done, while infants and children require flexible fibreoptic laryngoscopy.
o Radiology:
▪ X-ray: Soft tissue lateral and PA view of neck; X-ray chest in PA and lateral view to identify foreign body.
▪ CT-scan: For mediastinal mass e.g., double aortic arch or an anomalous left pulmonary artery forming a sli-
ng around the trachea.
▪ Angiography: For vascular anomalies.
▪ Oesophagogram with contrast: For tracheo-bronchial fistula or aberrant vessels or oesophageal atresia.
▪ Microlaryngoscopy and bronchoscopy are also done.

03. How will you investigate a case of 55 year old male smoker , suffering from gradually increasing hoarseness of voice for 4
months followed by stridor for last week? Write differential diagnosis. [2020](5+5)
➢ Investigation and differential diagnosis:
o Hoarseness of voice: Group-B, question no. 01.
o Stridor: Group-B, question no. 02.

04. A 50 years old patient has hoarseness of voice for over a month. Investigate & outline the management of benign or mali-
gnant growth in vocal card. [2004]
➢ Investigation of hoarseness of voice: Group-B, question no. 01.
➢ Management:
o Benign growth:
▪ Vocal nodules:
✓ Conservative: Early cases can be treated by educating the patient in proper use of voice. By this
many nodules in children disappear completely.
✓ Surgery: For large nodules or nodules of long standing in adults excision under operating microsc-
ope either with cold instruments or laser is done avoiding trauma to underlying vocal ligament.
▪ Vocal polyp: Surgical excision under operating microscope followed by speech therapy.
©shovandas 84
▪ Reinke's oedema: Longitudinal incision in the cord and removal of gelatinous fluid is done. Reeducation in
voice production and cessation of smoking are essential to prevent recurrence.
▪ Contact ulcer or granuloma:
✓ Antireflux therapy.
✓ Speech therapy: To stop throat clearing and correct the pitch of voice.
✓ Inhaled steroids or intralesional injection: To correct inflammation and size of granuloma.
✓ Surgery: Micro-laryngeal surgery may be needed to remove granuloma.
▪ Leukoplakia or keratosis: Stripping of vocal cords and subjecting the tissues to histology for any malignant
change is done. Chronic laryngeal irritants as the aetiology should be eliminated.
▪ Amyloid tumour: Surgical removal is done.
o Malignant growth:
▪ Radiotherapy: Done in early cases.
▪ Surgery:
✓ Conservation surgery:
• Cordectomy via laryngofissure: Excision of vocal cord after splitting the larynx.
• Partial frontolateral laryngectomy: Excision of vocal cord and anterior commissure region.
• Partial horizontal laryngectomy: Excision of supraglottis , i.e. epiglottis, aryepiglottic folds,
false cords and ventricle, a sort of transverse section of larynx above the vocal cords.
✓ Total laryngectomy: The entire larynx including the hyoid bone, pre-epiglottic space, strap muscles
and rings of trachea are removed. Pharyngeal wall is repaired and lower tracheal stump sutured to
the skin for breathing.
▪ Combined therapy: Surgery with pre and post operative radiotherapy is used.
▪ Endoscopic resection with CO2 laser: Carcinoma of the mobile membranous vocal cord can be excised by
CO2 laser under microscope.
▪ Organ preservation: The voice is preserved. Induction chemotherapy followed by radiotherapy or concurr-
ent chemoradiation showed better control than combined therapy in many cases.

Group – c
01. Vocal cord polyp. [2019]
➢ Vocal cord polyps are one of the most common benign laryngeal lesions.
➢ Epidemiology:
o Males are affected more than females.
o Age group: 30-50 years.
➢ Aetiopathogenesis:
o Vocal abuse:
▪ Sudden shouting → Haemorrhage in the vocal cord →
▪ Accumulation of blood or haematoma formation → Subsequent submucosal oedema.
o Other contributing factors:
▪ Allergy. ▪ Anti-coagulant use.
▪ Smoking. ▪ Aspiration of aggressive chemical substances.
▪ Reflux. ▪ Intense respiratory activities.
➢ Types:
o Haemorrhagic: Abrupt onset, due to extreme vocal effort.
o Non-haemorrhagic: Pseudocyst, due to outpouching of inflamed vocal cord.
➢ Clinical features:
o Signs:
▪ Mostly unilateral.
▪ Site: Midpoint of membranous part i.e. in the junction of anterior 1/3rd and posterior 2/3rd or anterior 1/3rd
and middle 1/3rd along the free edge, as this is the area of maximum vibration of the cord and thus subject
to maximum trauma.
▪ Soft, smooth, broad based and often pedunculated.
▪ It may flop up and down the glottis during respiration or phonation.
o Symptoms:
▪ Hoarseness: Most common symptom.
▪ Large polyp may cause dyspnoea, stridor or intermittent choking.
▪ Double voice: Some patients complain of diplophonia due to different vibratory frequencies of two cords.
➢ Management:
o Conservative treatment for small non-haemorrhagic polyps.
o Microlaryngeal surgery: Surgical excision under operating microscope is done for large sized polyps and small haem-
orrhagic polyps.
©shovandas 85
o Followed by post-operative speech therapy.
o Office-based pulsed-dye laser surgery can also be done.

02. Vocal nodule. [2014, 2009, 2003]


➢ Vocal nodules are one of the most common benign laryngeal lesions.
➢ Also called singer's node, screamer's node or teacher's node.
➢ Epidemiology: Teachers, actors, vendors or pop singers are mostly affected. Also seen in school going children who are too
assertive and talkative.
➢ Aetiopathogenesis:
o Result of vocal trauma when person speaks in unnatural low tones for prolonged periods or at high intensities.
o Trauma to the vocal cord →
o Oedema and haemorrhage in the submucosal space →
o Hyalinization and fibrosis →
o Hyperplasia of the overlying epithelium →
o Nodule is formed.
➢ Clinical features:
o Signs:
▪ Bilateral and sessile.
▪ Site: Midpoint of membranous part i.e. in the junction of anterior 1/3rd and posterior 2/3rd or anterior 1/3rd
and middle 1/3rd along the free edge, as this is the area of maximum vibration of the cord and thus subject
to maximum trauma.
▪ Size: Varies from that of pin-head to half a pea.
▪ Appearance:
✓ Early stage: Soft, reddish and oedematous.
✓ Late stage: Greyish or white in colour.
o Symptoms:
▪ Hoarseness: Most common symptom.
▪ Vocal fatigue and pain in the neck on prolonged phonation.
➢ Management:
o Conservative: Early cases can be treated by educating the patient in proper use of voice. By this many nodules in
children disappear completely.
o Surgery: For large nodules or nodules of long standing in adults excision under operating microscope either with cold
instruments or laser is done avoiding trauma to underlying vocal ligament.

03. Safety muscles of larynx. [2019, 2016]


➢ The posterior cricoarytenoid muscles are called the safety muscles of larynx.
➢ Anatomy:
o A pair of triangular muscles.
o Origin: Posterior surface of cricoid lamina lateral to median ridge.
o Insertion: Back of muscular process of the arytenoid cartilage.
o Nerve supply: Recurrent laryngeal nerve, branch of vagus.

➢ Function:
o The only intrinsic muscles of the larynx, which abduct the vocal cords to allow entry of air through rima glottidis in
the respiratory tract below it.
o When posterior cricoarytenoids contract, muscular processes of both the arytenoid cartilages rotate medially. As a
result, the vocal processes rotate laterally (abducting vocal cords) providing wide diamond-shaped opening of glottis.
o Action opposes the lateral cricoarytenoid muscles.

©shovandas 86
➢ Clinical significance or why called safety muscles: If posterior cricoarytenoids are paralyzed, the adductor muscles (of vocal
cords) take the upper hand and the person might die due to lack of air.

04. Laryngomalacia. [2010]


➢ Most common congenital anomaly of larynx.
➢ Also called congenital laryngeal stridor.
➢ Clinical features:
o Stridor:
▪ Excessive flaccidity of supraglottic larynx which is sucked in during inspiration produces stridor.
▪ More in supine position.
▪ Increased on crying.
▪ Subsides on placing the child in prone position.
o Sometimes cyanosis is also seen.
o The condition manifests at birth or soon after.
➢ Diagnosis:
o Flexible fibreoptic laryngoscopy:
▪ Inlet is large and floppy.
▪ Excessive tissue in supra-glottis.
▪ Large, curled up, omega shaped epiglottis.
▪ Short and floppy aryepiglottic fold.
▪ Large and prominent arytenoid.
▪ Cannot be diagnosed in a paralyzed patient.
o Airway fluoroscopy, direct laryngoscopy or bronchoscopy can also be done.
➢ Management:
o Reassurance.
o Usually disappears by 2 years of age.
o Conservative treatment: Done in most of the cases.
o Tracheostomy: May be required for some cases of severe respiratory obstruction.
o Supraglottoplasty: Required in cases of severe laryngomalacia.

05. Indications of tracheostomy. [2012]


➢ Tracheostomy: Tracheostomy is making an opening in the anterior wall of trachea and converting it into a stoma on the skin
surface.
➢ Indications:
o Respiratory obstruction:
▪ Infections:
✓ Acute laryngo-tracheo-bronchitis, acute epiglottitis, diphtheria.
✓ Ludwig’s angina, peritonsillar, retropharyngeal or parapharyngeal abscess, tongue abscess.
▪ Trauma:
✓ External injury of larynx and trachea.
✓ Trauma due to endoscopies, especially in infants and children.
✓ Fractures of mandible or maxillofacial injuries.
▪ Neoplasms: Benign and malignant neoplasms of larynx, pharynx, upper trachea, tongue and thyroid.
▪ Foreign body larynx.
▪ Oedema larynx: Due to steam, irritant fumes or gases, allergy (angioneurotic or drug sensitivity), radiation.
▪ Bilateral abductor paralysis.
▪ Congenital anomalies:
✓ Laryngeal web, cysts, tracheo-oesophageal fistula.
✓ Bilateral choanal atresia.
o Retained secretions:
▪ Inability to cough:
✓ Coma of any cause, e.g. head injuries, cerebrovascular accidents, narcotic overdose.
✓ Paralysis of respiratory muscles, e.g. spinal injuries, polio, Guillain-Barre syndrome, myasthenia
gravis.
✓ Spasm of respiratory muscles, tetanus, eclampsia, strychnine poisoning.
▪ Painful cough: Chest injuries, multiple rib fractures, pneumonia.
▪ Aspiration of pharyngeal secretions: Bulbar polio, polyneuritis, bilateral laryngeal paralysis.
o Respiratory insufficiency: Chronic lung conditions, viz. emphysema, chronic bronchitis, bronchiectasis, atelectasis.

06. Functions of tracheostomy. [2015]


➢ Tracheostomy: Tracheostomy is making an opening in the anterior wall of trachea and converting it into a stoma on the skin
surface.
©shovandas 87
➢ Functions:
o Alternative pathway for breathing: This circumvents any obstruction in upper airway from lips to the tracheostome.
o Improves alveolar ventilation: In cases of respiratory insufficiency, alveolar ventilation is improved by:
▪ Decreasing the dead space by 30–50% (normal dead space is 150 mL).
▪ Reducing the resistance to airflow.
o Protects the airways: By using cuffed tube, tracheobronchial tree is protected against aspiration of:
▪ Pharyngeal secretions: In case of bulbar paralysis or coma.
▪ Blood: In haemorrhage from pharynx, larynx or maxillofacial injuries. With tracheostomy, pharynx & larynx
can also be packed to control bleeding.
o Permits removal of tracheobronchial secretions: The tracheobronchial airway can be kept clean of secretions by
repeated suction through the tracheostomy, thus avoiding need for repeated bronchoscopy or intubation which is
not only traumatic but also requires expertise. This is done when:
▪ Patient is unable to cough as in coma, head injuries, respiratory paralysis.
▪ Cough is painful, as in chest injuries or upper abdominal operations.
o Intermittent positive pressure respiration (IPPR): If IPPR is required beyond 72 hour, tracheostomy is superior to
intubation.
o To administer anaesthesia: Where endotracheal intubation is difficult or impossible as in laryngopharyngeal
growths or truisms.

07. Types of tracheostomy. [2004]


➢ Tracheostomy: Tracheostomy is making an opening in the anterior wall of trachea and converting it into a stoma on the skin
surface.
➢ Types on the basis of clinical approach:
o Emergency tracheostomy: Employed when airway obstruction is complete or almost complete and there is an urgent
need to establish the airway. Intubation or laryngotomy are either not possible or feasible in such cases.
o Elective tracheostomy:
▪ Also called tranquil, orderly or routine tracheostomy.
▪ Planned, unhurried procedure.
▪ Endotracheal tube can be put and local or general anaesthesia can be given.
▪ Two types:
✓ Therapeutic: To relieve respiratory obstruction, remove tracheobronchial secretions or give assis-
ted ventilation.
✓ Prophylactic: To guard against anticipated respiratory obstruction or aspiration of blood or
pharyngeal secretions such as in extensive surgery of tongue, floor of mouth, mandibular resection
or laryngofissure.
o Permanent tracheostomy: Used in cases of bilateral abductor paralysis or laryngeal stenosis.
o Percutaneous dilatation tracheostomy: Commonly performed in critically sick patients.
o Mini tracheostomy (cricothyroidotomy): Used in the removal of airway secretions while maintaining glottic function.
➢ Types on the basis of anatomical approach:
o High tracheostomy:
▪ Done above the level of thyroid isthmus (II, III and IV tracheal rings).
▪ Complications: It violates the first ring of trachea. Tracheostomy at this site can cause perichondritis of the
cricoid cartilage and subglottic stenosis and is always avoided.
▪ Indication: Only indication is carcinoma of larynx.
o Mid tracheostomy:
▪ Mostly preferred.
▪ Done through the II or III ring and would entail division of the thyroid isthmus or its retraction upwards or
downwards to expose this part of trachea.
o Low tracheostomy:
▪ Done below the level of isthmus.
▪ Complications: Trachea is deep at this level and close to several large vessels. Also there are difficulties with
tracheostomy tube which impinges on suprasternal notch.

08. Steps of tracheostomy. [2003, 2004]


➢ Tracheostomy: Tracheostomy is making an opening in the anterior wall of trachea and converting it into a stoma on the skin
surface.
➢ Steps:
o Position:
▪ Rose's position.
▪ Same position as adenoidectomy and tonsillectomy.
▪ Extension of cervical and altanto-occipital joint to prevent aspiration.
o Skin incision:
©shovandas 88
▪ Horizontal:
✓ A horizontal or transverse 5 cm long skin crease incision 2.5 cm above suprasternal notch.
✓ Done in elective tracheotomy.
✓ Gives better cosmetic scar.
▪ Vertical:
✓ Vertical incision is made in midline, extending from cricoid cartilage to just above sternal notch.
✓ This is the most favoured incision and can be used in emergency and elective procedures.
✓ It gives rapid access with minimum of bleeding and tissue dissection.
o Approach towards trachea:
▪ After incision, tissues are dissected in the midline.
▪ Dilated veins are either displaced or ligated.
▪ Strap muscles are separated in the midline and retracted laterally.
▪ Thyroid isthmus is displaced upwards or divided between the clamps, and suture ligated.
o A few drops of 4% lignocaine are injected into the trachea to suppress cough when trachea is incised.
o Tracheal incision:
▪ Trachea is fixed with a hook and opening is made in tracheal rings T2, T3 or T4.
▪ The first tracheal ring is never divided as perichondritis of cricoid cartilage with stenosis can result.
▪ Before incision, air bubbles are aspirated with syringe to confirm location.
▪ A horizontal or vertical incision is made over trachea.
▪ This is then converted into a circular opening.
o Insertion of tube: Tracheostomy tube of appropriate size is inserted and secured by tapes.
o Skin incision should not be sutured or packed tightly as it may lead to development of subcutaneous emphysema.
o Gauze dressing is placed between the skin and flange of the tube around the stoma.

09. Intrinsic muscles of larynx. [2006]


➢ According to actions, intrinsic muscles of the larynx are arranged into three groups.
➢ Muscles that Open or Close the Laryngeal Inlet:
o Oblique arytenoids: Closes the inlet of larynx.
o Aryepiglotticus: Closes the inlet of larynx.
o Thyroepiglotticus: opens the inlet of larynx.
➢ Muscles that Abduct or Adduct the Vocal Cords:
o Posterior cricoarytenoids: abduct the vocal cords.
o Lateral cricoarytenoids: adduct the vocal cords.
o Transverse arytenoid: adduct the vocal cords.

©shovandas 89
➢ Muscles that Increase or Decrease the Tension of vocal cords:
o Cricothyroid: tenses the vocal cords.
o Vocalis: tenses the vocal cords.
o Thyroarytenoid: relaxes the vocal cords.
➢ All the intrinsic muscles of the larynx are paired except transverse arytenoid, which is unpaired.
➢ Posterior cricoarytenoids are called safety muscles of larynx, because if posterior cricoarytenoids are paralyzed, the adductor
muscles (of vocal cords) take the upper hand and the person might die due to lack of air.
➢ Figure: Group-C, question no. 03.

10. Functions of larynx. [2001]


➢ Protection of lower airways: Phylogenetically, this is the earliest function to develop.
o Sphincteric closure of laryngeal opening: When food is swallowed, its entry into air passage is prevented by closure
of three successive sphincters consisting of:
▪ laryngeal inlet i.e. aryepiglottic folds, tubercle of epiglottis and arytenoids, ,
▪ false cords and ▪ true cords,
which close the glottis. Thus, no foreign matter meant to be swallowed or accidentally vomited can enter the larynx.
o Cessation of respiration: Respiration temporarily ceases through a reflex generated by afferent fibres of IXth nerve,
when food comes in contact with posterior pharyngeal wall or the base of tongue.
o Cough reflex: Important mechanism to dislodge and expel a foreign particle when it comes into contact with respira-
tory mucosa. Larynx is aptly called watch-dog of lungs as it immediately “barks” at the entry of any foreign intruder.
➢ Phonation:
o Vocal cords are kept adducted →
o Infraglottic air pressure is generated by exhaled air from lungs by contraction of thoracic and abdominal muscles →
o The air force open the cords and is released as small puffs which vibrate the vocal cords and produce sound →
o Amplified by mouth, pharynx, nose and chest and is converted into speech by the modulatory action of lips, tongue,
palate, pharynx and teeth.
➢ Respiration: Larynx regulates flow of air into the lungs. Vocal cords abduct during inspiration and adduct during expiration.
➢ Fixation of the chest: When larynx is closed, chest wall gets fixed and various thoracic and abdominal muscles can then act
best. This function is important in digging, pulling and climbing. Coughing, vomiting, defaecation, micturition and childbirth
also require a fixed thoracic cage against a closed glottis.

Group – d
01. Complications of tracheostomy. [2019, 2017, 2011]
➢ Tracheostomy: Tracheostomy is making an opening in the anterior wall of trachea and converting it into a stoma on the skin
surface.
➢ Complications:
o Immediate: At the time of operation.
▪ Haemorrhage.
▪ Apnoea: This follows opening of trachea in a patient who had prolonged respiratory obstruction. This is due
to sudden washing out of CO2 which was acting as a respiratory stimulus. Treatment is to administer 5% CO2
in O2 or assisted ventilation.
▪ Pneumothorax due to injury to apical pleura.
▪ Injury to recurrent laryngeal nerves.
▪ Aspiration of blood.
▪ Injury to oesophagus: This can occur with tip of knife while incising the trachea and may result in tracheo-
oesophageal fistula.
o Intermediate: During first few hours or days.
▪ Bleeding: Reactionary or secondary.
▪ Displacement ▪ Subcutaneous emphysema.
of tube. ▪ Tracheitis and tracheobronchitis with crusting in trachea.
▪ Blocking of the ▪ Atelectasis and lung abscess.
tube. ▪ Local wound infection and granulations.
o Late: With prolonged use of tube for weeks and months.
▪ Haemorrhage: Due to erosion of major vessel.
▪ Laryngeal stenosis: Due to perichondritis of cricoid cartilage.
▪ Tracheal stenosis: Due to tracheal ulceration and infection.
▪ Tracheo-oesophageal fistula: Due to prolonged use of cuffed tube or erosion of trachea by the tip of
tracheostomy tube.
▪ Problems of decannulation: Seen commonly in infants and children.
▪ Persistent tracheocutaneous fistula.
©shovandas 90
▪ Tracheostomy scar: Keloid or unsightly scar.
▪ Corrosion of tracheostomy tube and aspiration of its fragments into the tracheobronchial tree.

02. Nerve supply of larynx. [2018]


➢ Motor: All the muscles which move the vocal cord (abductors, adductors or tensors) are supplied by the recurrent laryngeal
nerve except the cricothyroid muscle. The latter receives its innervation from external laryngeal nerve, a branch of superior
laryngeal nerve.
➢ Sensory: Above the vocal cords, larynx is supplied by internal laryngeal nerve, a branch of superior laryngeal, and below the
vocal cords by recurrent laryngeal nerve.
➢ Recurrent laryngeal nerve:
o Right recurrent laryngeal nerve: Arises from the vagus at the level of subclavian artery, hooks around it and then
ascends between the trachea and oesophagus.
o Left recurrent laryngeal nerve: Arises from the vagus in the mediastinum at the level of arch of aorta, loops around
it and then ascends into the neck in the tracheo-oesophageal groove. Thus, left recurrent laryngeal nerve has a much
longer course which makes it more prone to paralysis compared to the right one.
➢ Superior laryngeal nerve: Arises from inferior ganglion of the vagus, descends behind internal carotid artery and, at the level
of greater cornua of hyoid bone, divides into external and internal branches. The external branch supplies cricothyroid muscle
while the internal branch pierces the thyrohyoid membrane and supplies sensory innervation to the larynx and hypopharynx.

03. Branchial cyst. [2017]


➢ One of the most common anomalies of branchial arches.
➢ Epidemiology:
o It is common in the second decade of life but can occur at any age.
o Equal frequency in both sexes.
➢ Pathogenesis: Cystic collection due to persistent fusion of 2nd and 5th branchial arch.
➢ Clinical features:
o Present in anterior triangle.
o Soft, smooth, ovoid, fluctuant, non-tender.
o Transillumination negetive.
o Seen at junction between upper 1/3rd and lower 2/3rd of sternocleidomastoid.
o 2/3rd is in triangle and 1/3rd is behind the sternocleidomastoid muscle.
o Sometimes associated with sinus or fistula.
➢ Diagnosis:
o MRI neck.
o Contrast imaging (old).
➢ Complications: Infection may lead to abscess formation.
➢ Tretment: Excision surgery is done under general anaesthetia.
➢ Branchial fistula:
o Formed due to communication between two epithelial surfaces and non-fusion of 2nd and 5th branchial arch.
o External opening: Junction between upper 2/3rd and lower 1/3rd of sternocleidomastoid.
©shovandas 91
o Internal opening: Near the tonsillar fossa.

04. Acute epiglottitis. [2014]


➢ It is serious acute inflammatory condition confined to supraglottic structures, i.e. epiglottis, aryepiglottic folds and arytenoids.
➢ Aetiology:
o Affects children of 2–7 years of age but can also affect adults.
o Group A beta-haemolytic streptococcus is most common organism responsible for this condition.
o H. influenzae B is also a common organism responsible for this condition in children.
➢ Clinical features:
o Onset of symptoms is abrupt with rapid progression.
o There is marked oedema of these structures which may obstruct the airway.
o Sore throat and dysphagia are the common presenting symptoms in adults.
o Dyspnoea and stridor are the common symptoms in children. They are rapidly progressive and may prove fatal unless
relieved.
o Fever may go up to 40°C. It is due to septicaemia. Patient’s condition may rapidly deteriorate.
➢ Examination:
o Tongue depression: Depressing the tongue with a tongue depressor may show red and swollen epiglottis.
o Flexible fibreoptic laryngoscopy: Oedema and congestion of supraglottic structure is seen. This examination is
avoided for fear of precipitating complete obstruction. It is better done in operation theatre where facilities for
intubation are available.
o X-ray: Lateral soft tissue X-ray of neck may show swollen epiglottis (thumb sign).
➢ Treatment:
o Hospitalization: Essential because of the danger of respiratory obstruction.
o Antibiotics: Ampicillin or third generation cephalosporin are effective against H. influenzae and are given by
parenteral route (i.m. or i.v.) without waiting for results of throat swab and blood culture.
o Steroids: Hydrocortisone or dexamethasone is given in appropriate doses i.m. or i.v. They relieve oedema and may
obviate need for tracheostomy.
o Adequate hydration: Patient may require parenteral fluids.
o Humidification and oxygen: Patient may require mist tent or a croupette.
o Intubation or tracheostomy: It may be required for respiratory obstruction.

05. Laryngeal web. [1998]


➢ A common congenital anomaly of larynx.
➢ Pathogenesis: It is due to incomplete recanalization of larynx at the level of glottis. Mostly, the web is seen between the vocal
cords, limited to anterior commissure and has a concave posterior margin.
➢ Ckinical features:
o Airway obstruction.
o Weak cry or aphonia dating from birth.
o Inspiratoty stridor.
➢ Management:
o Depends on the thickness of the web.
o Thin webs: Can be cut with a knife or CO2 laser.
o Thick webs: Require excision via laryngofissure and placement of a silicon
keel to keep the edges apart and subse-quent dilatations.

06. Laryngicoele. [2008]


➢ It is an airfilled benign cystic swelling due to dilatation of saccule.
➢ Types:
o Internal: Confined within the larynx and presents as
distension of false cord and aryepiglottic fold.
o External: Distended saccule herniates through thyroid
membrane and presents in neck.
o Combined or mixed: Both internal and external compo-
nents are seen.
➢ Pathogenesis: A laryngocele is supposed to arise from raised
transglottic air pressure as in trumpet players, glass-blowers or
weight lifters.
➢ Clinical features:
o Hoarseness of voice.
o Cough.
o If large, obstruction to the airway.

©shovandas 92
o An external laryngocele presents as a reducible swelling in the neck which increases in size on coughing or performing
Valsalva.
o A laryngocele in an adult may be associated with carcinoma which causes obstruction of saccule.
➢ Diagnosis:
o Valsalva maneuver: Swelling increses.
o Bryce sign: On compressing the swelling sudden gush of air into the larynx produces hissing sound.
o Indirect laryngoscopy.
o Radiology:
▪ X-ray: Soft tissue AP and lateral views of neck.
▪ CT scan: To find the extent of lesion.
➢ Treatment:
o Surgical excision through an external neck incision.
o Marsupialization of an internal laryngocele can be done by laryngoscopy but there are chances of recurrence.

07. Laryngeal papilloma. [2007]


➢ Constitutes about 80% of the total occurrence of neoplasms of the larynx.
➢ Can be deivided into two types.
➢ Juvenile papillomatosis:
o Also called respiratory papillomatosis.
o Most common benign neoplasm of the larynx in children.
o Aetiology:
▪ Viral in origin and is caused by human papilloma DNA virus type 6 and 11.
▪ Type 11 is more virulent.
▪ Affected children got the disease at birth from their mothers who had vaginal HPV disease.
o Pathology: Papillomas mostly affect supraglottic and glottic regions of larynx but can also involve subglottis, trachea
and bronchi. Children who had tracheostomy for respiratory distress due to laryngeal papillomas have higher inci-
dence of tracheal and stomal involvement due to seeding. DNA virus particles have been found in the cells of base-
ment membrane of respiratory mucosa and may account for widespread involvement and recurrence.
o Clinical features:
▪ Patient, often a child, between the age of 3 and 5 years.
▪ Hoarseness of voice or aphonia.
▪ Respiratory difficulty.
▪ Stridor.
▪ Recurrence is common.
▪ Rarely undergoes malignant changes.
o Diagnosis: Flexible fibreoptic laryngoscopy is done and later confirmed by direct laryngoscopy and biopsy.
o Management:
▪ Surgical management:
✓ Microlaryngoscopy and CO2 laser excision: Avoiding injury to vocal ligament.
✓ Microdebrider: Tumour can be removed under microscope with cup forceps or a debrider similar
to the one used in endoscopic nasal surgery.
✓ Aim of therapy: To maintain a good airway, preserve voice and avoid recurrence.
▪ Medical management:
✓ Interferon alpha-2a: Has several side effects including fever, chills, myalgia, arthralgia, headache,
loss of weight and suppression of bone marrow.
✓ 13-cis-retinoic acid: This too has several side effects.
✓ Bevacizumab.
✓ Cidofovir.
➢ Adult onset papilloma:
o Usually single, smaller in size, less aggressive and does not recur after surgical removal.
o Common in males (2:1) in the age group of 30-50 years.
o Usually arises from the anterior half of vocal cord or anterior commissure.
o Treatment is the same as for juvenile type.

08. Indirect laryngoscopy. [2006]


➢ Technique:
o Position: Patient is seated opposite the examiner. He should sit erect with the head and chest leaning slightly towards
the examiner.
o Procedure: He is asked to protrude his tongue which is wrapped in gauze and held by the examiner between the
thumb and middle finger. Index finger is used to keep the upper lip or moustache out of the way. Gauze piece is used
to get a firm grip of the tongue and to protect it against injury by the lower incisors. Laryngeal mirror (size 4–6) which

©shovandas 93
has been warmed and tested on the back of hand is introduced into the mouth and held firmly against the uvula and
soft palate. Light is focussed on the laryngeal mirror and patient is asked to breathe quietly.
o Observations: To see movements of the cords, patient is asked to take deep inspiration (abduction of cords), say
“Aa” (adduction of cords) and “Eee” (for adduction and tension). Movements of both the cords are compared.
➢ Structures seen: Group-A, question no. 01.
➢ Diagnosis:
o Appearance of lesion:
▪ Lesions of suprahyoid epiglottis are usually exophytic while those of infrahyoid epiglottis are ulcerative.
▪ Lesions of vocal cord may appear as raised nodule, ulcer or thickening.
▪ Lesions of anterior commissure may appear as granulation tissue.
▪ Lesions of subglottic region appear as a raised submucosal nodule, mostly involving the anterior half.
o Vocal cord mobility: Impairment or fixation of vocal cord indicates deeper infiltration into thyroarytenoid muscle,
cricoarytenoid joint or invasion of recurrent laryngeal nerve and is an important sign.
o Extent of disease: Spread of disease to vallecula, base of tongue and pyriform fossa should be noticed.

09. Principal of laryngeal crepitus test. [2020]


➢ Normally, larynx moves with deglutition. It can also be moved from side to side producing a characteristic grating sound
(laryngeal crepitus). Fixity of larynx indicates inflammation of infiltration of growth into the surrounding structures. Loss of
laryngeal crepitus is due to postcricoid carcinoma.

©shovandas 94
Chapter 03

Oral cavity
And
Salivary glands

©shovandas 95
©shovandas 96
Group – a
No questions.

Group – b
No questions.

Group – c
No questions.

Group – d
01. Lingual thyroid. [2009]
➢ Lingual thyroid is a rare clinical entity of the oropharyngeal region showing the presence of thyroid tissue in the tongue.
➢ Representation:
o May be the only thyroid tissue or
o Be present in addition to normal thyroid or
o An ectopic thyroid.
➢ Epidemiology:
o Seen in 1:3000 to 1:4000 patients of thyroid disease.
o Female predominance of occurrence is seen seven times higher than male counterpart.
➢ Pathogenesis:
o The thyroid gland normally descends along the midline with growth of the neck
o But sometimes all or a part of the gland fails to migrate along the path from ventral floor of the pharynx to normal
location in front of trachea over thyroid cartilage.
o Then it sequestrates within the tongue substance giving rise to an embryological defect with location at the base of
the tongue, in a zone posterior to the circumvallate papillae.
➢ Clinical features:
o Dysphagia, o Rarely bleeding,
o Dysphonia, o Upper respiratory tract obstruction.
➢ Differential diagnosis:
o Lymphoma,
o Squamous cell carcinoma,
o Minor salivary gland tumour,
o Lingual tonsil,
o Rarely thyroglossal cyst.
➢ Diagnosis:
o Indirect laryngoscopy: Seen as a mass at the base of tongue.
o Technetium (Tc-99) scintigraphy: To identify lingual thyroid with other ectopic thyroids, if any.
➢ Treatment: Surgical removal should be done, if lingual thyroid causes symptoms of airway obstruction or dysphagia by the
supra-hyoid transpharyngeal approach.
➢ Complication: It requires lifelong replacement of thyroid hormones, if it was the only thyroid tissue.

02. Fossa of Rosenmullar. [2004]


➢ The fossa of Rosenmüller is a bilateral projection of the nasopharynx just below the skull base.
➢ Also called the lateral pharyngeal recess or pharyngeal recess.
➢ Location:
o Located behind the torus tubarius, a prominence caused by the medial cartilaginous end of the Eustachian tube.
o The torus is larger on the superior and posterior lips of the Eustachian tube, effectively hiding the fossa.
o The fossa is covered by nasopharyngeal mucosa.
o Extends through a defect between the fibers of the superior constrictor muscle and the base of the skull.
➢ Boundaries:
o Anterior: Eustachian tube and levator palatini muscle.
o Posterior: Posterior wall of the nasopharynx and retropharyngeal space.
o Lateral: Parapharyngeal space and tensor veli palatini muscle.
o Inferior: Upper edge of the superior constrictor muscle.
o Superior: The skull base with its various openings and prominences, including the foramen spinosum medially, the
carotid canal, and the foramina spinosum and ovale. At the apex, only a thin layer of fibroconnective tissue separa-
tes the mucosa from the cervical internal carotid artery.
➢ Clinical significance: It is the most common site of origin of nasopharyngeal carcinoma (NPC).

©shovandas 97
©shovandas 98
Chapter 04

Thyroid

©shovandas 99
©shovandas 100
Group – a
No questions.

Group – b
No questions.

Group – c
No questions.

Group – d
01. Management of solitary thyroid nodule. [2019]
➢ Solitary thyroid nodule: A solitary thyroid nodule can be:
o Colloid nodule (also called adenomatous nodule).
o Adenoma:
▪ Follicular adenoma,
▪ Hurthle cell adenoma.
o Thyroid cyst (cystic change in colloid nodule or follicular adenoma).
o Regenerative nodule.
o Dominant nodule (in multinodular goitre).
o Autonomous (or toxic) nodule.
o Carcinoma.

©shovandas 101
➢ Management:
o Suppressive therapy: Enlargement of thyroid and formation of nodule are due to trophic effect of TSH on thyroid
cells. Suppressing TSH with exogenous T4 is assumed to reduce nodule formation. Suppressive therapy has the risk
of cardiac arrhythmias and decrease in bone density in such patients who are usually euthyroid. In general suppre-
ssive therapy is not preferred.
o Surgery:

©shovandas 102
Chapter 05

Oesophagus

©shovandas 103
©shovandas 104
Group – a
No questions.

Group – b
01. A 60 years old male has presented with progressively increasing dysphagia. How will you investigate?
➢ Dysphagia is difficulty in swallowing.
➢ History: A detailed history is of paramount importance.
o Ascertain, if dysphagia is of:
▪ Sudden onset: Foreign body or any impaction of food on pre-existing stricture or malignancy, or neuro-
logical disorders.
▪ Progressive: Malignancy.
▪ Intermittent: Spasms or spasmodic episodes over an organic lesion.
▪ More to liquids: Paralytic lesions.
▪ More to solids and progressing even to liquids: Malignancy or stricture.
▪ Intolerance to acid food or fruit juices: Ulcerative lesions.
o Note associated symptoms:
▪ Regurgitation and heart burn (hiatus hernia).
▪ Regurgitation of undigested food while lying down, with cough at night (hypopharyngeal diverticulum).
▪ Aspiration into lungs (laryngeal paralysis).
▪ Aspiration into the nose (palatal paralysis).
➢ Clinical examination: Examination of oral cavity, oropharynx, larynx & hypopharynx can exclude most of the pre oesopha-
geal causes of dysphagia. Examination of neck, chest, nervous system, including cranial nerves should also be undertaken.
➢ Blood examination: Haemogram in the diagnosis and treatment of Plummer-Vinson syndrome and to know the nutritional
st-atus of the patient.
➢ Radiography:
o X-ray chest: To exclude cardiovascular, pulmonary and mediastinal diseases.
o Lateral view neck: To exclude cervical osteophytes and any postcricoid or retropharyngeal space soft tissue lesion.
o Barium swallow: It is useful in the diagnosis of malignancy, cardiac achalasia, strictures, diverticula, hiatus hernia
or oesophageal spasms. Combined with fluoroscopic control or cineradiography, it can help in the diagnosis of any
motility disorder of oesophageal wall or sphincters.
➢ Manometric and pH studies: A pressure transducer along with a pH electrode and an open-tipped catheter is introduced
into the oesophagus to measure the pressures in oesophagus and at its sphincters. Acid reflux into oesophagus is measured
by pH electrode. It also measures the effectiveness of oesophagus to clear acid load after acid solution is put in oesophagus.
These studies help in motility disorders , gastro-oesophageal reflux & to find whether oesophageal spasms are spontaneous
or acid induced.
➢ Oesophagoscopy: It gives direct examination of oesophageal the mucosa and permits biopsy specimens. Flexible fibreoptic
or rigid scopes can be used.
➢ Other investigations:
o Bronchoscopy: For bronchial carcinoma.
o Cardiac catheterization: For vascular anomalies.
o Thyroid scan: For malignant thyroid.

02. Mechanism of deglutition. [2009]


➢ Deglutition: Passage of bolus of food (solid or liquid) from the oral cavity to stomach via the pharynx and oesophagus.
➢ Oral phase: Under voluntary control and further divided into two phases:
o Oral preparatory phase:
▪ Breaking down of food in the oral cavity by mastication.
▪ Chewing and mixing of food with saliva to form the bolus.
▪ Done by the action of masseter and teeth.
▪ There is role of tongue, lips and cheeks also.
o Oral phase proper:
▪ Bolus is moved towards the back of the tongue.
▪ Elevation of the tongue against the palate.
▪ The bolus is propelled into the pharynx.
▪ There is role of mandible and hyoid bone also.
➢ Pharyngeal phase:
o It is initiated when the bolus of food comes into contact with pharyngeal mucosa.
o A series of reflex actions take place carrying the food past oro- and laryngopharynx into the oesophagus.

©shovandas 105
o Closure of nasopharynx:
▪ The nasopharynx is completely cut off from the oropharynx.
▪ Mechanism: Soft palate contracts against the Passavant’s ridge on the posterior pharyngeal wall.
o Closure of oropharyngeal isthmus:
▪ The entry of food back into oral cavity is prevented.
▪ Mechanism:
✓ Contraction of tongue against the palate.
✓ Sphincteric action of palatoglossal muscles.
o Closure of larynx:
▪ Aspiration into the larynx is prevented.
▪ Mechanism:
✓ Temporary cessation of respiration.
✓ Closure of laryngeal inlet by contraction of aryepiglottic folds.
✓ Closure of false and true cords.
✓ Rising of larynx under the base of tongue.
▪ Role of epiglottis:
✓ The bolus impinges on the epiglottis and gets diverted to the lateral food channel.
✓ Deflects backwards when food passes into the pyriform fossae.
o Contraction of pharyngeal muscles and relaxation of cricopharyngeus: Food passes from pharynx into oesophagus
by synchronized action of these muscles.
➢ Oesophageal phase:
o After food enters the oesophagus, the cricopharyngeal sphincter closes.
o Peristaltic movements of oesophagus take the bolus down the stomach.
o Gastro-oesophageal sphincter at the lower end of oesophagus relaxes well before peristaltic wave reaches, permits
fluids to pass.
o After the bolus enters into the stomach the sphincter closes.
o Regurgitation of food back from stomach into oesophagus is prevented by:
▪ Tone of gastro-oesophageal sphincter.
▪ Negative intrathoracic pressure.
▪ Pinch-cock effect of diaphragm.
▪ Mucosal folds.
▪ Oesophagogastric angle.
▪ Slightly positive intra-abdominal pressure.

Group – c
01. Plummer-Vinson syndrome. [2020, 2015, 2011]
➢ Plummer-Vinson syndrome, also known as Patterson-Brown-Kelly syndrome is an extremely rare disease of esophagus.
➢ Epidemiology:
o Predominantly it affects white females.
o Age: Usually more than 40 years.
➢ Aetiology:
o Iron deficiency anaemia: Most important.
o Malnutrition.
o Genetic predisposition.
o Autoimmune processes: As there is associated diseases like celiac disease, thyroid disease or rheumatoid arthritis.
➢ Pathogenesis: Web formation in the postcricoid region of oesophagus due to subepithelial fibrosis in this region.
➢ Clinical features:
o Classical triad:
▪ Dysphagia: Usually painless and intermittent or progressive over years , limited to solids and sometimes
associated with weight loss.
▪ Iron-deficiency anaemia: Weakness, pallor, fatigue and tachycardia.
▪ Oesophageal webs.
o Other features:
▪ Glossitis, ▪ Angular stomatitis,
▪ Achlorhydria, ▪ Atrophy of the mucous membrane of the alimentary tract.
▪ Koilonychia, ▪ Enlargement of the spleen and thyroid may also be observed.
➢ Diagnosis:
o Barium swallow: Shows a web in the postcricoid region.
o Oesophagoscopy: The web appears smooth, thin and gray with eccentric or central lumen.
©shovandas 106
➢ Complications:
o About 10% of the cases with this syndrome will develop postcricoid carcinoma.
o It also predisposes to the development of carcinoma in the tongue, buccal mucosa, pharynx, oesophagus and the
stomach i.e. upper GI tract squamous cell cancers.
➢ Treatment:
o To correct anaemia by oral/parenteral iron. Serum levels of iron are more important than haemoglobin level.
o Associated B12 and B6 deficiency should also be corrected.
o Dilatation of the webbed area by oesophageal bougies.
o The patients should be followed closely. Surveillance upper gastrointestinal endoscopy is recommended every year.

02. Oesophagial stricture. [2012, 2005]


➢ An oesophageal stricture is a narrowing of the oesophagus that impedes progress of a bolus as it transits to the stomach.
➢ Aetiology: The strictures usually occur when muscular coat of the oesophagus is damaged. The common causes are:
o Benign strictures:
▪ Burns due to corrosive substances or hot fluids.
▪ Trauma due to:
✓ Impacted foreign bodies.
✓ Instrumentation.
✓ External injuries.
▪ Ulcerations due to:
✓ Reflux oesophagitis.
✓ Diphtheria.
✓ Typhoid.
▪ Postsurgical:
✓ Antireflux surgery.
✓ Myotomy.
✓ Anastomotic.
✓ Osophageal atresia repair.
✓ Tracheo-esophageal fistula repair.
▪ Webs due to PVS. ▪ Schatzki's ring.
▪ Crohn's disease. ▪ Epidermolysis bullosa dystrophica.
▪ Congenital strictures. ▪ Extrinsic pressure.
o Malignant strictures:
▪ Adenocarcinoma of oesophagus.
▪ Squamous cell carcinoma of oesophagus.
➢ Clinical features:
o Dysphagia, first to solids and then to liquids, is the common complaint.
o When obstruction is complete, regurgitation and cough may occur.
o Patient is malnourished.
➢ Diagnosis:
o Barium swallow.
o Oesophagoscopy: To exclude malignancy.
➢ Treatment:
o Prograde dilatation with bougies: It should be done under direct vision through oesophagoscope. Dilatations may
be required frequently.
o Gastrostomy: It helps to feed the patients and give rest to the inflamed area above the strictures. After a few days,
when inflammation subsides, lumen may become visible and prograde dilatation can be restored. Patient can be gi-
ven a thread to swallow, which is recovered from the stomach, and prograde or retrograde bouginage can be done.
o Surgery: Excision of strictured segment and reconstruction of food passage using stomach, colon or jejunum.

Group – d
01. Cardiospasm. [2009]
➢ Also called cardiac achalasia or achalasia cardia.
➢ Pathology:
o Absence of peristalsis in the body of oesophagus.
o High resting pressure in lower oesophageal sphincter, which does not relax during swallowing.
➢ Symptoms:
o Dysphagia: More to liquids than solids (reverse of that seen in malignancy or strictures).
o Regurgitation: Regurgitation of swallowed food particularly at night.
©shovandas 107
➢ Diagnosis:
o Radiography: Barium swallow shows dilated oesophagus with narrowed rat tail lower end, sometimes also called
bird-beak appearance.
o Manometric studies: Low pressure in the body of oesophagus and high pressure at lower sphincter and failure of
the sphincter to relax.
o Endoscopy: To exclude benign stricture or any development of carcinoma which is a common complication of this.
➢ Treatment of choice:
o Modified Heller’s operation.
o Myotomy of the narrowed lower portion of the oesophagus is done.
o Forceful pneumatic dilatation of the lower oesophagus can be done in those unfit for surgery.

02. Oesophagoscopy. [2008]


➢ Rigid oesophegoscopy:
o Indication:
▪ Diagnostic: To investigate cause for:
✓ Dysphagia: Cancer oesophagus, cardiac achalasia, strictures, oesophagitis, diverticulae, etc.
✓ Retrosternal burning: Reflux oesophagitis or hiatus hernia.
✓ Hematemesis: Oesophageal varices.
✓ Secondaries neck with unknown primary (as a part of panendoscopy).
▪ Therapeutic:
✓ Removal of a foreign body.
✓ Dilatation in case of oesophageal strictures or cardiac achalasia.
✓ Endoscopic removal of benign lesions, e.g. fibroma, papilloma, cysts, etc.
✓ Insertion of Souttar’s or Mousseau-Barbin tube in palliative treatment of oesophageal carcinoma.
✓ Injection of oesophageal varices.
o Contraindication: Trismus, disease of cervical spine, receding mandible, aneurysm of aorta & advanced heart, liver
or kidney disease.
o Anaesthesia: General anaesthesia, local anaesthesia in selected individuals.
o Position: Patient lies supine, head is elevated by 10-15 cm, neck flexed on chest and head extended at atlantoocci-
pital joint to attain the axes of mouth, pharynx and oesophagus in a straight line to pass the rigid tube easily.
o Procedure:
▪ Oesophagoscope is lubricated and Introduced to the back of the tongue and advanced gently →
▪ Epiglottis, endotracheal tube and arytenoids are seen one by one →
▪ Slow but sustained pressure is given to open the cricopharyngeal sphincter →
▪ During crossing the narrowing, 25 cm from the incisors, aortic pulsation can be seen →
▪ Head and shoulders remain below the level of the table, head being slightly higher than the shoulders and
moved slightly to the right →
▪ Scope pointing to the left ASIS, cardia is identified having redder, more velvety, rugose mucosa →
▪ Oesophageal wall should be inspected again when the oesophagoscope is withdrawn.
o Complications:
▪ Injury to lips, teeth, arytenoids, pharyngeal mucosa.
▪ Perforation of oesophagus, most often at the site of Killian’s dehiscence.
▪ Compression of trachea.
➢ Flexible fibreoptic oesophagoscopy:
o Advantage over rigid oesophagoscopy:
▪ Outdoor procedure.
▪ Does not require general anaesthesia, done under local anaesthesia.
▪ Can be used in patients with abnormalities of spine or jaw.
▪ The oesophagus, stomach and duodenum can all be examined in one sitting.
▪ Good illumination and magnification helps in the accurate diagnosis.
▪ Precision biopsies can be taken.
o Procedure: The patient lies in left lateral position and fibrescope is passed through a plastic mouth prop into the ph-
arynx, postcricoid area and oesophagus, insufflating air as endoscope is advanced , to open the lumen of oesophagus.
➢ Transnasal oesophagoscopy:
o Fibreoptic oesophagoscopy performed through nose.
o Air can be inflated through it to distend the walls of oesophagus to look for any lesion in its mucosal folds.
o Oesophagus can be examined up to gastric fundus.
o Advantages:
▪ Tracheoesophageal puncture can be performed for oesophageal speech in laryngectomized patient.
▪ Laryngeal biopsy can be taken.

©shovandas 108
03. Oesophagial constructions. [2005]
➢ Normally, there are four sites of anatomical constrictions /narrowing in the oesophagus. The distance of each constriction is
measured from the upper incisor teeth.
➢ Constructions:
o First constriction:
▪ Cervical construction.
▪ Location: At the pharyngo-oesophageal junction.
▪ Distance from upper incisor teeth: 9 cm (6 inches).
▪ Vertebral level: C6.
o Second constriction:
▪ Aortic construction.
▪ Location: Where it is crossed by the arch of aorta.
▪ Distance from upper incisor teeth: 22.5 cm (9 inches).
▪ Vertebral level: T4.
o Third constriction:
▪ Bronchial construction.
▪ Location: Where it is crossed by the left principal bronchus.
▪ Distance from upper incisor teeth: 27.5 cm (11 inches).
▪ Vertebral level: T6.
o Fourth constriction:
▪ Diaphragmatic construction.
▪ Location: Where it pierces the diaphragm.
▪ Distance from upper incisor teeth: 40 cm (15 inches).
▪ Vertebral level: T10.
➢ Clinical significance:
o These are the sites where swallowed foreign bodies may stuck in the
oesophagus.
o These are the sites where strictures develop after ingestion of caustic
substances.
o These sites have predilection for the carcinoma of the oesophagus.
o These are sites through which it may be difficult to pass oesophagoscope /gastric tube.

04. Cardiac notch. [1996]


➢ The right margin of the oesophagus is continuous with the lesser curvature of the stomach, while the left margin joins
the greater curvature at an acute angle, termed the cardiac notch.
➢ Anatomy:
o Relations:
▪ Anterior:
✓ Posterior surface of the left lobe of the liver.
✓ Left gastric nerve.
▪ Posterior:
✓ Left crus of diaphragm.
✓ Right gastric nerve.
o Only part of oesophagus that is covered with serous membrane, the peritoneum.
o Diaphragmatic constriction and lower oesophagial sphincter are present near this region.
o Arterial supply:
▪ Left gastric artery.
▪ Left inferior phrenic artery.
o Venous drainage:
▪ Hemiazygos vein, a tributary of inferior vena cava.
▪ Left gastric vein, a tributary of portal vein.
▪ Site of portocaval anastomosis.
o Lymphatic drainage: Left gastric lymph nodes.
o Nerve supply:
▪ Parasympathetic: recurrent laryngeal nerves and oesophageal plexuses formed by vagus nerves.
▪ Sympathetic: T4 and T5.
➢ Clinical significance:
o Most affected site of oesophagus in GERD and achalasia cardia.
o Most common site for oesophagial carcinoma.

©shovandas 109
05. Dysphagia lusoria. [1996]
➢ An abnormal condition characterized by difficulty in swallowing caused by an aberrant right subclavian artery.
➢ Pathology:
o During development of aortic arch, sometimes the right subclavian artery arises from thoracic aorta →
o Passes in front of or behind the oesophagus →
o Compression of oesophagus by subclavian artery.
➢ Symptoms:
o Although most cases of this anomaly are asymptomatic, symptoms may appear when a ring completely encircles the
oesophagus.
o Extrinsic compression of the oesophagus leads to dysphagia.
o Dysphagia by freak of nature.
➢ Diagnosis:
o Oesophagogram: Revealed a notching of the upper oesophagus from an extrinsic compression.
o Arteriogram: Confirmed the aberrant origin of the right subclavian artery.
➢ Treatment: Surgical repair is performed. Reconstruction or ligation of aberrant right subclavian artery by sternotomy /by
neck approach.

06. Coin in oesophagus. [2017]


➢ Coin in oesophagus is one of the most common incidence of foreign body obstruction.
➢ Aetiology:
o Age: Children are most often affected. Nearly 80% are below 5 years.
o Carelessness.
➢ Site of lodgement: Commonest site is at or just below the cricopharyngeal sphincter. Flat objects like coins are held up at the
sphincter.
➢ Clinical features:
o Symptoms:
▪ History of initial choking or gagging.
▪ Discomfort or pain:
✓ Located just above the clavicle on the right or left of trachea.
✓ Discomfort increases on attempts to swallow.
✓ Local discomfort may point to the site of coin in cervical oesophagus.
▪ Dysphagia: Obstruction may be partial or total. Partial obstruction becomes total with time due to edema.
▪ Drooling of saliva: In cases of total obstruction. Saliva may be aspirated causing pneumonitis.
▪ Respiratory distress:
✓ The coin compresses posterior wall of trachea causing respiratory obstruction especially in child.
✓ Laryngeal oedema can develop.
▪ Substernal or epigastric pain: Due to oesophageal spasm or incipient perforation.
o Signs:
▪ Tenderness in the lower part of neck on the right or left of trachea.
▪ Pooling of secretions in the pyriform fossa on indirect laryngoscopy. They do not disappear on swallowing.
▪ Sometimes the coin may be seen protruding from the oesophageal opening in the postcricoid region.
➢ Investigation: As coin in radio-opaque posteroanterior and lateral view X-ray of neck and similar view X-ray of the chest in-
cluding abdomen are taken.
➢ Management:
o Endoscopic removal: Oesophagoscopy under GA is done. Both rigid and flexible scopes can be used to remove coin
from the oesophagus.
o A hypopharyngeal speculum resembling a laryngoscope with long blade is less traumatic & more convenient to use
for coin lodged near the upper sphincter.
o Cervical oesophagotomy is done in complicated cases.

©shovandas 110
Chapter 06

Recent advances

©shovandas 111
©shovandas 112
Group – a
No questions.

Group – b
No questions.

Group – c
No questions.

Group – D
No questions.

©shovandas 113
©shovandas 114
Chapter 07

clinical methods
and
neck masses

©shovandas 115
©shovandas 116
Group – a
No questions.

Group – b
No questions.

Group – c
No questions.

Group – D
No questions.

©shovandas 117
©shovandas 118
Chapter 08

Operative surgery

©shovandas 119
©shovandas 120
Group – A
All are given in respective chapters.

Group – B
All are given in respective chapters.

Group – c
All are given in respective chapters.

Group –d
All are given in respective chapters.

©shovandas 121
©shovandas 122
The End

©shovandas 123
©shovandas 124

You might also like